◆ わからない問題はここに書いてね 33 ◆

このエントリーをはてなブックマークに追加
1132人目のメルセンヌ数さん
   / ̄   ̄ ヽ
  / ,,w━━━.、)   / ̄ ̄ ̄ ̄ ̄ ̄ ̄ ̄ ̄ ̄ ̄ ̄ ̄ ̄ ̄ ̄ ̄
  ! .fw/f_」」_|_|_i_)  | .ここは分からない問題について質問するさくらちゃんスレですわ
  ヽ|:::(6||f;j' ,fj'||)  .| スレッドや業務連絡,記号の書き方例は >>2-13 辺りに。
 ∠|::i:!::|:|、_ワノ:i、 <  ローマ数字や丸付き数字などの機種依存文字はお勧め出来ませんわ
  .|::|< |::|ヽーノ`l:i;ヽ \_________________
  .ノ:ノ' i:::l `只´|:|i)::)
 (::(:i  |:::|ノ ) j:j|:(

   (⌒, -- 、⌒)     / ̄ ̄ ̄ ̄ ̄ ̄ ̄ ̄ ̄ ̄ ̄ ̄ ̄ ̄ ̄ ̄
  _ .Y      Y  _ < 自分でどこまで考えたのか、途中でもいいから
 ミ  | ・  . ・| / 彡  | 書いてくれればこっちも答えやすくて助かるわー
   @ゝ.  ^  ノ@    | 質問者も解答者もくれぐれもトラブルは起こさんといてなー
                \________________

【前のスレッド】
◆ わからない問題はここに書いてね 32 ◆
http://science.2ch.net/test/read.cgi/math/1021721809/l50
【過去のスレッド】
◆ わからない問題はここに書いてね1〜32 ◆
01 http://cheese.2ch.net/math/kako/967/967755172.html
02 http://cheese.2ch.net/math/kako/970/970795775.html
03 http://cheese.2ch.net/math/kako/974/974911042.html
04 http://cheese.2ch.net/math/kako/978/978209589.html
05 http://cheese.2ch.net/math/kako/981/981372834.html
06 http://cheese.2ch.net/math/kako/985/985594205.html
07 http://cheese.2ch.net/math/kako/988/988952592.html
08 http://cheese.2ch.net/math/kako/991/991223596.html
09 http://cheese.2ch.net/math/kako/993/993571403.html
10 http://cheese.2ch.net/math/kako/995/995448453.html
11 http://cheese.2ch.net/math/kako/997/997329928.html
12 http://cheese.2ch.net/math/kako/999/999689496.html
13 http://cheese.2ch.net/math/kako/1001/10013/1001342715.html
14 http://cheese.2ch.net/math/kako/1002/10028/1002893257.html
15 http://cheese.2ch.net/math/kako/1004/10041/1004171159.html
16 http://cheese.2ch.net/math/kako/1005/10057/1005735838.html
17 http://cheese.2ch.net/math/kako/1006/10068/1006859798.html
18 http://cheese.2ch.net/math/kako/1007/10078/1007834117.html
19 http://cheese.2ch.net/math/kako/1009/10091/1009102965.html
20 http://cheese.2ch.net/math/kako/1010/10107/1010708150.html
21 http://cheese.2ch.net/math/kako/1011/10116/1011689052.html
22 http://cheese.2ch.net/math/kako/1012/10125/1012535858.html
23 http://cheese.2ch.net/math/kako/1014/10146/1014673280.html
24 http://cheese.2ch.net/math/kako/1014/10146/1014673280.html
25 http://cheese.2ch.net/math/kako/1015/10158/1015866030.html
26 http://cheese.2ch.net/math/kako/1016/10165/1016541847.html
27 http://cheese.2ch.net/math/kako/1017/10175/1017511624.html
28 http://natto.2ch.net/math/kako/1018/10183/1018304190.html
29 http://natto.2ch.net/math/kako/1019/10193/1019394107.html
30 http://science.2ch.net/test/read.cgi/math/1020310032/
31 http://science.2ch.net/test/read.cgi/math/1021001363/
32 http://science.2ch.net/test/read.cgi/math/1021721809/
【関連スレッド】
雑談はここに書け!【4】
http://science.2ch.net/test/read.cgi/math/1021808853/
くだらねぇ問題スレ ver.3.1415926535897
http://science.2ch.net/test/read.cgi/math/1022008901/
【掲示板での数学記号の書き方例】
■数の表記
●スカラー:a,b,c,...,z, A,B,C,...,Z, α,β,γ,...,ω, Α,Β,Γ,...,Ω, ... (← ギリシャ文字はその読み方で変換
可.)
●ベクトル:V=[V[1],V[2],...], |V>, V↑, vector(V) (← 混同しない場合はスカラーと同じ記号でいい.通
常は縦ベクトルとして扱う.)
●テンソル(上下付き1成分表示):T^[i,j,k...]_[p,q,r,...], T[i,j,k,...;p,q,r,...]
●行列(1成分表示):M[i,j], I[i,j]=δ_[i,j]
●行列(全成分表示):M=[[M[1,1],M[2,1],...],[M[1,2],M[2,2],...],...], I=[[1,0,0,...]',[0,1,0,...],...] (← 行(また
は列ごと)に表示する.)

■演算・符号の表記
●足し算:a+b
●引き算:a-b
●掛け算:a*b, ab (← 通常"*"を使い,"x"は使わない.)
●割り算・分数:a/b, a/(b+c), a/(bc) (← 通常"/"を使い,"÷"は使わない.)
●割り算分数2:(a+b)/(c+d),a+(b/c),(a/b)+c(←括弧を用い分子分母を他の項と区別できるように表
現する。)
●複号:a±b=a士b, a干b (← "±"は「きごう」で変換可.他に漢字の"士""干"なども利用できる.)
●内積・外積・3重積:a・b, axb, a・(bxc)=(axb)・c=det([a,b,c]), ax(bxc)

■関数・数列の表記
●関数:f(x), f[x]
●数列:a(n), a[n], a_n
●平方根:√(a+b)=(a+b)^(1/2) (← "√"は「るーと」で変換可.)
○累乗根:[n]√(a+b)=(a+b)^(1/n) 《《新しく追加しました》》
●指数・指数関数:a^b, x^(n+1), exp(x+y)=e^(x+y) (← "^"を使う."exp"はeの指数.)
●対数・対数関数:log_{a}(b), log(x/2)=log_{10}(x/2), ln(x/2)=log_{e}(x/2) (← 底を省略する場合,
"log"は常用対数,"ln"は自然対数.)
●三角比・三角関数:sin(a), cos(x+y), tan(x/2)
●行列式・トレース:|A|=det(A), tr(A)
●絶対値:|x|
●ガウス記号:[x] (← 関数の変数表示などと混同しないように注意.)
●共役複素数:z~
●転置行列・随伴行列:M', M† (← "†"は「きごう」で変換可.)
●階乗:n!=n*(n-1)*(n-2)*...*2*1, n!!=n*(n-2)*(n-4)*...
●順列・組合せ:P[n,k]=nPk, C[n.k]=nCk, Π[n,k]=nΠk, H[n,k]=nHk (← "Π"は「ぱい」で変換可.)

■微積分・極限の表記
●微分・偏微分:dy/dx=y', ∂y/∂x=y,x (← "∂"は「きごう」で変換可.)
●ベクトル微分:∇f=grad(f), ∇・A=div(A),∇xA=rot(A), (∇^2)f=Δf (← "∇"は「きごう」,"Δ"は
「でるた」で変換可.)
●積分:∫[0,1]f(x)dx=F(x)|_[x=0,1], ∫[y=0,x]f(x,y)dy, ∬_[D]f(x,y)dxdy, ?_[C]f(r)dl (← "∫"は「い
んてぐらる」,"∬?"は「きごう」で変換可.)
●数列和・数列積:Σ_[k=1,n]a(k), Π_[k=1,n]a(k) (← "Σ"は「しぐま」,"Π"は「ぱい」で変換可.)
●極限:lim_[x→∞]f(x) (← "∞"は「むげんだい」で変換可.)

■その他
●図形:"△"は「さんかく」,"∠"は「かく」,"⊥"は「すいちょく」,"≡"は「ごうどう」,"∽"は「きごう」で変
換可.
●論理・集合:"⇔⇒∀∃∧∨¬∈∋⊆⊇⊂⊃∪∩"は「きごう」で変換可.
●等号・不等号:"≠≒≦≧≪≫"は「きごう」で変換可.

※ ここで挙げた表記法は1例であり,標準的な表記法からそうでないものまで含まれているので,後者の
場合使う時にあらかじめことわっておいたほうがいい.
※ 関数等の変数表示や式の括弧は,括弧()だけでなく[]{}を適当に組み合わせると見やすい場合がある.
※ 上記のほとんどの数学記号や上記以外の数学記号は大体「きごう」で順次変換できる.
【一般的な記号の使用例】
a:係数、数列 b:係数、重心
c:定数、積分定数 d:微分、次数、次元、距離、外微分、外積、公差
e:自然対数の底、単位元、分岐指数、基底、離心率 f:関数、多項式、基底
g:関数、多項式、群の元、種数、計量、重心 h:高さ、関数、多項式、群の元、類数、微小量
i:添え字、虚数単位、埋めこみ、内部積 j:添え字、埋めこみ、j-不変量、四元数体の基底
k:添え字、四元数体の基底、比例係数 l:添え字、直線、素数
m:添え字、次元、Lebesgue測度 n:添え字、次元、自然数
o:原点 p:素数、射影
q:素数、exp(2πiτ) r:半径、公比
s:パラメタ、弧長パラメタ t:パラメタ
u:ベクトル v:ベクトル
w:回転数 x:変数
y:変数 z:変数(特に複素数変数)

A:行列、環、加群、affine空間、面積
B:行列、開球、Borel集合、二項分布
C:複素数体、連続関数全体の集合、組み合わせ、曲線、積分定数、Cantorの3進集合、チェイン複

D:関数の定義域、微分作用素、判別式、閉球、領域、二面体群、Diniのderivative、全行列環
E:単位行列、楕円曲線、ベクトル束、単数群、辺の数
F:原始関数、体、写像、ホモトピー、面の数
G:群、位相群、Lie群
H:Hilbert空間、Hermite多項式、部分群、homology群、四元数体、上半平面、Sobolev空間、重複
組み合わせ
I:区間、単位行列、イデアル
J:Bessel関数、ヤコビアン、イデアル、Jacobson根基
K:体、K群、多項式環、単体複体、Gauss曲率
L:体、下三角行列、Laguerre多項式、L関数、Lipschitz連続関数全体の集合、関数空間L^p、線
型和全体
M:体、加群、全行列環、多様体
N:自然数全体の集合、ノルム、正規部分群、多様体
O:原点、開集合、整数環、直交群、軌道、エルミート演算子
P:条件、素イデアル、Legendre多項式、順列、1点、射影空間、確率測度
Q:有理数体、二次形式
R:半径、実数体、環、可換環、単数規準、曲率テンソル、Ricciテンソル
S: 級数の和、球面、部分環、特異チェイン複体、対称群、面積、共分散行列
T:トーラス、トレース、線形変換
U:上三角行列、unitary行列、unitary群、開集合、単数群
V:ベクトル空間、頂点の数、体積
W:Sobolev空間、線形部分空間
X:集合、位相空間、胞複体、CW複体、確率変数、ベクトル場
Y:集合、位相空間、ベクトル場、球面調和関数 Z:有理整数環、中心
【一般的な記号の使用例】
α:定数、方程式の解 β:定数、方程式の解
γ:定数、Euler定数、曲線 δ:微小量、Diracのdelta関数、Kroneckerのdelta
ε:任意の正数、実二次体の基本単数、Levi-Civitaの記号
ζ:変数、zeta関数、1の冪根
η:変数 θ:角度
ι:埋めこみ κ:曲率
λ:定数、測度、固有値、Z_p拡大の不変量、モジュラー関数
μ:定数、測度、Z_p拡大の不変量、Mobiusの関数
ν:測度、付値、Z_p拡大の不変量
ξ:変数 ο:Landauの記号
π:円周率、射影、素元、基本群
ρ:rank、相関係数
σ:標準偏差、置換、σ関数、単体、σ代数
τ:置換、群の元、捩率 υ:
φ:空集合、写像、Eulerの関数
χ:Euler標数、特性関数、階段関数  ψ:写像
ω:character、1の3乗根、微分形式

Β:beta関数  Γ:gamma関数、SL(2、R)の離散部分群、Christoffelの記号
Δ:微小変化、対角線集合、対角線写像、weight12のcusp form、単位円板、ラプラシアン、行列式
Λ:作用域、添え字集合、対角行列 Π:積記号
Σ:和記号、素体、(共)分散行列 Ο:Landauの記号
Φ:写像 Ψ:写像
Ω:代数的平方、拡大体、領域
【業務連絡】
■900を超えたら新スレに移行準備.
■旧スレ側 → 終了宣言,新スレへの誘導.
■新スレ側 → 開始宣言と目次,旧スレのリンク,掲示板での数学記号の書き方例,
  業務連絡・その他,旧スレ側の残り問題の移動.
■数学板の要望スレで数学板の注意書き(リンク先)の変更依頼.
■単独の質問スレは,このスレか「くだらんスレ」に誘導して下さい.
■誤って過去スレに新たに書き込まれた質問は,最新スレに誘導して下さい.
【数学板削除依頼スレ】
http://kaba.2ch.net/test/read.cgi/saku/986384122/ (レス削除)
http://kaba.2ch.net/test/read.cgi/saku/987829968/ (スレッド削除)
【ローカルルール等リンク先更新総合スレッド2】
http://kaba.2ch.net/test/read.cgi/accuse/1012720188/l50
★__________________________.
|              │
│ はにゃ?ん     |
| γ∞γ~       |
│人w/ 从从) )   │
│ ヽ | |┬ イ |〃  │
│ `wハ~ . ノ)    │
│  /  `「 .     │
| 数学板さくらスレ  |
|_________________________│

〃二二ヽ
| |77777〉
| | ゚д゚ノ|  サクラチャンノハタケイヨウデスワ
|⊂   つ
━━━━━━━━━━━━━━━━━━━━━━━━━━━━━━━━━━━━━

                     移転完了しましたわ♪
              ◆ わからない問題はここに書いてね 33 ◆
         いよいよ始まりますわ それではみなさま心置きなくどうぞ

━━━━━━━━━━━━━━━━━━━━━━━━━━━━━━━━━━━━━
9なっしゅ:02/05/25 14:57
A+B=C
A−B=C
A×B=C
A÷B=C

が、成り立つA.B.Cの数字ってあるかな?
>9
A+B=C
A−B=C
よりB=0
A÷B が定義できない

>1-8
スレ建て乙
12132人目の素数さん:02/05/25 15:05
>>1さんおつかれさま。

因数分解で、

x^2+xy+2x+y+1
=x^2+2x+1+(x+1)y
=(x+1)^2+(x+1)y
=(x+1)(x+y+1)

で、最後が、(x+1)(x+y+1) なぜこうなるのかがわかりません。教えてください。
(x+y)^3y になるのではないのですか?
>12
(x+1)^2 と (x+1)y を掛け算すると (x+y)^3y になるけど
(x+1)^2 と (x+1)y を足し算しても (x+y)^3y にはならんだろ

14132人目の素数さん:02/05/25 15:10
>12
ネタでなくマジで?やばい
足し算と掛け算の区別がついてない。それでもへんだ。
15132人目の素数さん:02/05/25 15:12
>>12
x^2+xy+2x+y+1
=x^2+2x+1+(x+1)y
=(x+1)^2+(x+1)y

=(x+1)(x+1)+(x+1)y
分配法則より
=(x+1){(x+1)+y}
これを整理すると、
=(x+1)(x+y+1)
何桁の数字であっても素数か因数分解できるかを
簡単に判別する方法ってありますか?

例えば 2^32+1(4,294,967,297)が、
素数かどうかを解説してきださい
>>16 素数判定法は知らないので、だれかが解説してくれるのを待つ。
オレなら素数の表をつくっておいて、√N までの素数で割ってみる
プログラムを作るけどのね。

ちなみに、2^32+1 = 641 * 6700417 で合成数。
18132人目の素数さん:02/05/25 15:44
=(x+1){(x+1)+y}

{(x+1)+y}
すみません、ここがわかりません、その前までyは乗法だったのに、なぜ加法に変わるのでしょうか?
>18
文字の計算では分配法則が成立する。すなわち
 AB+AC=A(B+C)
という感じのことが成り立つというのはいいかな?

A=x+1、B=x+1、C=y とおくと、

(x+1)(x+1)+(x+1)y
=(x+1){(x+1)+y}
20132人目の素数さん:02/05/25 15:59
>>19
ありがとうございました!とっても良くわかりました。本当に感謝です。
白チャでつまづいてちゃだめですね・・・・・。でもがんばります。
レス下さった方々、ありがとうございました。
f:N→N f(n)=n+1
これって全射かつ単射ですか?
>21
f(m)=f(n) ⇒ m+1=n+1 ⇒m=n
よって、単射?(Y/N)

Nの最小元を0とすると、
f(n)=0 となるn∈Nは存在しない。
よって、全射?(Y/N)
23132人目の素数さん:02/05/25 18:25
>21
Nがどういう集合かによるでしょう。自然数だったら違います。
整数だったらYesかな。
普通はN(あるいはZ)は自然数のとき使いますよね。
24132人目の素数さん:02/05/25 18:38
>23
Zは整数だったか、余計なことを書いた
25132人目の素数さん:02/05/25 19:03
ある家族「父・母・娘二人・息子二人・召使い・犬」がいます。
この家族が大きな川を渡ろうとしています。
船はひとつしかありません。しかも乗れるのは二人だけで一人は運転手がいります。
運転できるのは父、母、召使いだけで、父は母がいないと娘を殺してしまい、
母は父がいないと息子を殺し、犬は召使がいないと家族を殺してしまいます。
どういけば誰も死なずに川を渡れるでしょう?何回往復してもかまいません。
ひっかけじゃないよ。犬も定員1です。
           ◯
           │
           ◯
          / \
◯ ◯
¥ /
           ◯

図がきたないですがこれって束ですか?
上の2つをとった場合最大下界がないかんじがするので束ではないと思うんですが

           │
           ◯
          /  \
         ◯   ◯
          ¥ /
           ◯

28なんどもすみません:02/05/25 19:15
           ◯
           │
           ◯
          /  \
         ◯   ◯
          ¥ /
           ◯
>>28
それは束だ。
マソコの絵か?
   A    「{A,B}の最大下界」 = 「X≦AかつX≦BとなるXで最大のもの」
  |    従ってこの場合はBになる。
   B
 / \
C   D
 \ /
   E
32DQNですみません:02/05/25 21:01
直線と円の共有点の座標の問題なのですが
(1+a^2)x^2-4ax+3=0

D/4=(2a)^2-3(1+a^2)=a^2-3
となる経緯がさっぱり分かりません。
教えてください。お願いいたします。
数値計算法の質問はここでしていいのですか?
シミュ版で聞いてもいいとも悪いとも言われません
34132人目の素数さん:02/05/25 21:35
>32
Dは、二次式の判別式のことだよね。
一般に、二次式 ax^2 + 2bx + c の判別式をDとすると、
 D/4 = b^2 - ac
となる。このことは教科書にも書いてあるとおもうけど。。
判別式の計算方法は知ってるよね? それから導かれる式だよ。
知らなければ、教科書を見ること。

で、わからないというのは、こーいうことなの?
35132人目の素数さん:02/05/25 21:35
>32
まず一般論
ax^2+bx+c=0 のとき
判別式D=b^2−4ac はいいですか?

次にxの係数が偶数のとき b=2b’と書くことにすれば
ax^2+2b’x+c=0 に対して
D=(2b’)^2−4ac
 =4b’^2−4ac
 =4{(b')^2−ac}
D/4=(b’)^2−ac これで解の判別はできます

ところで問題に戻って
(1+a^2)x^2−2*2ax+3=0 に当てはめてみてください
b’→2a,a→(1+a^2),c→3 を当てはめる
36132人目の素数さん:02/05/25 21:36
>>33
どんな内容の話?
3733:02/05/25 21:41
>>36
(x)=x^3+x-1=0の解を不動点反復法を使って求めよ。
という問題で、x=g(x)を求め反復過程が収束するかを確かめよという設問の解法がわかりません
a(n+1)=1+a(n)^2-1/a(n)
あたりでtrial
39132人目の素数さん:02/05/25 21:44
面積を等面積に二分化する
いかなる直線も共通して通る点を点Aとする
このような点が存在する図形の性質はどのようなものであるか?

例えば正四角形や円はその中心が点Aであるが
正三角形にはそのような点Aが存在しない
>>16
素数の判定方法って何種類かあって、
それもこんなところではいちいち解説できません。
もしかしたら以下の本が役に立つかもしれません。
「コンピュータと素因数分解」和田秀男著(遊星社)
# いきなりアドレマン=ルメリー法とか…。(w
4133:02/05/25 21:53
f(x)=x^3+x-1=0 fが抜けてました。念のため
>>39
すごい面白い問題tnx
平面の集合Sについて上半平面との共通部分をS+,下半平面との共通部分をS-
とする。回転変換Rによる像をR(S)と書くと、
任意のRに対して|R(S)+|=|R(S)-|となる為のSの条件を求めよという問題
のほうがまだ簡単そう。簡単といっても、一筋縄じゃいかない感じ。円環とか
あるしね。Sに面積が定義されるという条件は設定します。
>>39
凸な図形に限れば点対称なもの。
凸に限らないなら…どう言えばいいだろう?
基本的な方針としては極座標で表してから、
よーしパパ積分してから微分しちゃうぞー、ってな感じ。
44132人目の素数さん:02/05/25 23:39
>39
点Aがあるとして、xy平面の原点に置く
x軸で分ける、y軸で分ける、x=±yで分ける…
と細かく分けていくと、原点で対象な領域で面積が
等しくなくてはならない。

あとは自分でやれ
45132人目の素数さん:02/05/26 00:42
a(1)≧ 2, a(n+1)=2+3/a(n) (n≧1)と仮定する。

問1  a(n)≧ 2  (n≧1) を示せ。
問2  |a(n+1)-3|≦1/2|a(n)-3| (n≧1) を示せ。
問3 lim_[n→∞]a(n) を求めよ。

自分では少しも解りませんでした。
ここのスレ全体からみると、
Lvの低い問題っぽくて恥ずかしいんですが・・・。
よろしくお願いします。
>>43
穴がなければいいのであって、凸まで要求しなくてもいいんじゃないの?
穴があるなら、かなり勝手な形が許されそうで、簡単な特徴づけはなさ
そうだけど。

点Aを通る直線をわずかにずらしたとき、一方の面積の増加分と減少分
が同じならいいわけだよね。
>>45 こういうの、連分数というんだ。a(1) をもとにして、
a(2), a(3), ... を書き下してごらん。分母が入れ子になった
分数になるだろう?その極限値を求める問題だ。

極限値(の候補)だけなら簡単に求めることができる。まあ問3の
別解だが、a(n+1) = a(n) となる条件、つまり、x = 2+3/x を
解くと、(x-3)(x+1) = 0。x>0 だから x=3 が極限値。

この問題は x=3 を誘導尋問的に求めさせようとしているだけ。

問1 はいいだろう? 問2 は各a(n) の「3 = 極限値からの距離」
で、n が一つ進むたびに 1/2 になるといっている。頑張って、
証明してくれたまえ。このことから、数列{a(n)}は 3に収束
することがわかるので、問3はあきらか。
4846:02/05/26 00:58
あ、そうか。穴なしでは不十分だね。
4938:02/05/26 01:24
>>33
収束しない数列でした。適当書いちゃったので、修正します。
収束するのはa(n)=1.0/(1+a(n-1)^2)です。(※)
これでやってみてください。0.682328位の根が見つかりました。
xを問題の方程式の根とすると、f(x)=1.0/(1.0+x^2)-xとして
dx>0の時、f(x+dx)<0 f(x-dx)>0が成り立てば収束しそうです。
これを証明すれば、根に十分近い初期値を持つ数列(※)が根に
収束することがが証明されそうです。
50132人目の素数さん:02/05/26 02:00
みなさんには楽勝の問題でしょうが(というより常識でしょうが)
Aa+Bb=G.C.D(a,b)から
「素数pがa,bの積を割り切れば、a,bのいずれかをpは割り切る。」
を証明するというのがわかりません。
いや、馬鹿なんでほんとわかんないっす。
段階を経た証明を教えてください。お願いします。
>48
連結かつ単連結(穴なし)なら十分だと思うんだけど、
違うの?
52132人目の素数さん:02/05/26 02:18
>>50
A,Bは整数?
a,bは整数?
a,bが整数なら
Aa+Bb=G.C.D(a,b)がなくても
素数pがa,bの積を割り切れば、a,bのいずれかをpは割り切れるのよね。
問題あってる?
53オラ来る:02/05/26 02:23
えっと答えがあってるかどうか確認したいんですけど、、いいですか?
y=3at^2/1+t^3(aは定数)
を微分したときの答えって3a(2t-t^4)/(1+t^3)^2
でいいですか?
54132人目の素数さん:02/05/26 02:26
>>53
多分あってる
解いてないけど
頑張ればそれでよし
>>53
アッテマス
5650:02/05/26 02:27
>>52
そうです。(A,B,a,b整数)
Aa+Bb=G.C.D(a,b)から
「素数pがa,bの積を割り切れば、a,bのいずれかをpは割り切る。」
が証明される。と本に書いてあるんです。
「」の命題自体は背理法やらで簡単に証明できるんですが、
これを使うとなると、全く発想できないのでして。
57オラ来る:02/05/26 02:29
>>54-55さん
ありがとうございます。
すっとしました。
>>50
素数の定義を書いてみそ
59lplplplplpl:02/05/26 03:00
aの四乗+bの四乗とaの四乗-bの四乗の因数分解の解教えてください
ヨロシクお願いします。。
60132人目の素数さん:02/05/26 03:01
>>50
数学セミナー6月号に載っている。
61132人目の素数さん:02/05/26 03:02
>>59
a^4+b^4
を因数分解したいとな。

ほほう。どの範囲ですればよいのじゃ?
62コニシ:02/05/26 03:04
お願い解いて

D={0≦x≦y-1,0≦y≦π/2}←にぶんのぱい
上の範囲で∫∫sin y dxdy
を求めよ。
マルチんげーる
64コニシ:02/05/26 03:08
>>62はもう解けました。
6550:02/05/26 03:15
>>60
数学セミナー6月号に書いてある事がわからないということです。
(3)から(1)が証明される、という所の説明が欲しいのです。

重ねて誰かお願いします。
Aa+Bb=G.C.D(a,b)から
「素数pがa,bの積を割り切れば、a,bのいずれかをpは割り切る。」
を証明する(a,b,A,Bは整数)とはどういうことでしょうか。
詳しい段階を経て示してくださるといいのですが。
>>65
「素数の定義は1と自分自身以外に約数を持たない」とするのね?
 ここでは。
67132人目の素数さん:02/05/26 03:23
>>50
とりあえず、数学セミナー6月号は手元にあるよな。
それから、12ページの式(3)がどうして理解できるのかは分かるか?
6850:02/05/26 03:23
>>66
何しろ、数学セミナーの解読ですから、数学セミナーに書いてある
その通りでよいとおもいます。
6967:02/05/26 03:24
>>67
あう・・・日本語としておかしい。
(3)は理解できるよな?
って聞きたかった。
7050:02/05/26 03:29
>>67
わかりますわかります。
ユークリツド互除法を使って、余りをr=αa+βbのかたちで
(つまり線形結合で)変化させていけばいいんですよね。
それと、「」の命題から素因数分解の一意性を証明する過程も
理解できてます。後ここだけが抜けているんです。
7150:02/05/26 03:34
>>70では>>67
どうやって証明できるのかはわかるか?、と解しました。
公式の意味としてもわかっているつもりですが...
>59
a^4+b^4=(a^2+b^2)^2-2(ab)^2

a^4-b^4=(a^2-b^2)(a^2+b^2)
73lplplplplpl:02/05/26 03:38
>>72さんお答え有り難うございました。。
テスト前で・・・・・・(泣)
74132人目の素数さん:02/05/26 03:39
>>50
OK状況は理解できた。

つうわけで、こっちも考える・・・期待させてたらゴメン・・・
でもがんばる。
75132人目の素数さん:02/05/26 03:45
>>50
http://mis.edu.yamaguchi-u.ac.jp/kyoukan/watanabe/elements/book7/proposition/proposition7-30.htm
希望のものとかなり違うが一応証明らしい。

念のために数学セミナー6月号風のやつも探してくる。
>50
Aa+Bb=G.C.D(a,b)=1
のときだけ考えればよい。
両辺にaをかけて…
(Aa-1)aはpの倍数
両辺にbをかけて…
(Bb-1)bはpの倍数
a、bともにpの倍数でないとすると

AaとBbのG.C.Dが1にならない罠
7745:02/05/26 03:50
>>47
ありがとうございます。
感謝します。
gcd(a,p)=p or 1 である

gcd(a,p)=p ならば、pはaを割り切る。

gcd(a,p)=1 ならば、あるA,Bが存在して Aa+Bp=gcd(a,p)=1 とできる
両辺にbを掛ければ、b=Aab+Bbp でabはpで割り切れるので、
pはbを割り切る。
7950:02/05/26 03:58
だいたいわかりました。ありがとうございます!
でもそこで、
しいて言えばG.C.D(a,b)=1 にして一般性は大丈夫なんでしょうか。
8050:02/05/26 04:03
>>79は遅レスでした…。
>>78
いや〜。エレガントですね。
これで安心して先に進めます。今度こそありがとうございました!
81DQNですみません:02/05/26 05:08
>>34-35
ありがとうございました。感激です。
アフォで申し訳ございませんでした。
82りな:02/05/26 05:30
sin15ていくらでしたっけ?

http://members.goo.ne.jp/home/rina-1919
83132人目の素数さん:02/05/26 05:36
>>82
壁紙のねーちゃん、腹たるんでるな(w
>>78
派生問題
「特定の整数の組(a,b)についてのみ,Aa+Bb=gcd(a,b)となる整数A,Bが
存在する」
という仮定の元で同じ命題
pが素数⇔pの約数の集合={p,1}
p|ab⇔(p|a)または(p|b)
が証明できるでしょうか?
8584:02/05/26 05:40
書き方悪かった。
「2つの整数a,bについてAa+Bb=gcd(a,b)となる,整数A,Bが存在するとします。
この時、abがある素数pの倍数ならばaまたはbの少なくとも一方がpの
倍数であることを示せ。」
8684:02/05/26 05:45
「ある整数a,bについて、ある整数A,Bがあって
Aa+Bbがa,bの最大公約数であるとする。
この時、abがある素数pの倍数ならばaまたはbの少なくとも一方がpの
倍数であることを示せ。」
8784:02/05/26 05:58
次の命題は正しいか、それとも誤りか、正しい場合は
証明を、誤ってる場合は反例を挙げよ。

「1を持つ可換環Rの元a,bに対しある整数A,Bが存在して
Aa+Bbがイデアル(a,b)の生成元となっているとする。
またp∈Rが1とp以外に約元を持たないとする。

この時ab∈(p)ならばa∈(p)またはb∈(p)
(つまり(p)はRの素イデアル)」
>>87=86=85=84
(a,b)の生成するイデアルとゆーよりも
(a)∩(b)の性制限というほうが
一般化という観点からは好ましい
ように思えるが。
89132人目の素数さん:02/05/26 06:52
「点P(x,y)が曲線x^2 + y^2 - 8x - 6y + 9 = 0の上を動くとき、関数
f(x,y) =4x+3y の最大最小とそのときのPの座標をそれぞれ求めよ。」
という問題です。

解答では円のパラメータ表示で解いていたのですが、4x+3y =kとしてグラフから
線形計画法で解くやりかたを教えてください。
座標を求めるところまでお願いします。
円のパラメータ表示による解法を理解できる人が何故円の位置や
直線の傾き等を理解できずまた線形計画法という言葉を知りながら
それを使って解けないのかその理由を教えてください。
91132人目の素数さん:02/05/26 07:49
>>90
すいません、解答と値が違うので合っているのかなと思いまして。
それじゃあ解答書いてみますね。間違いを見つけたら教えてください。

x^2 + y^2 - 8x - 6y + 9 = 0の式は中心(4,3)、半径4の円で
直線の式:4x+3y =kとが円に接する条件って点と直線の距離公式を
使うんですよね?それで|4・4 + 3・3 - k|/√(4^2 + 3^2) = 4
を整理すると、|25-k|=20になって、k≦25のときk=5, k≧25のときk=45
となるのですが、ここで解答と違います。解答では最大値5, 最小値-5と
なっています。

それから、Pの座標を求めるところは直線と円が接する時がPの座標だから、
判別式でいいんですか?計算がめちゃくちゃで無理っぽいのですが。
92132人目の素数さん:02/05/26 07:50
>45
(1) a(n)>2>0ならばa(n+1)>2 きちんというなら数学的帰納法
(2) a(n+1)−3=−1+3/a(n)=−(a(n)−3)/a(n)
両辺の絶対値を考える。 a(n)>2を使えば明らか
(3) |a(n)−3|<(1/2)^(n-1)|a(1)−3|→0 収束
   a(n)→3
93132人目の素数さん:02/05/26 08:02
>91
Pの座標は判別式より、円の中心を通る垂直な直線を作って
直線と円の交点を出すほうが楽じゃないかな。
(kがわかっているなら直線同士の交点でも良い。でもそれだと2回やらないと
いけない)
>>91
最小値-5ってどうも眉唾
(x-4)^2+(y-3)^2=16
4x+3y=-5
が実数解持つかVerifyしてみ
95数学ニガテっ子:02/05/26 08:08
数Bの問題なのですが教えてください!
    @ X^3+X^2−8X−12=0
    A X^4−6X^2+8X−3=0

                  
96132人目の素数さん:02/05/26 08:14
>95
因数定理は使えるのか?
97数学ニガテっ子:02/05/26 08:17
因数定理は
 P(k)=0⇔P(x)は因数xーkをもつというやつですよね…確か。。。
98132人目の素数さん:02/05/26 08:41
適当に整数(定数項の約数)を放り込んでいけば見つかる。
−の場合も忘れないように。
2番なんか1か3しかないんだから、簡単だろ。
99132人目の素数さん:02/05/26 08:43
>>93
御返事感謝です。そのような早い解法があるんですね!
確かに簡単に求まると思います。ちょっとこの問題で試しにやってみた
のですが、値がまた解答と違ってきました。円の中心を通る垂直な直線
の式はy=3/4(x-4)+3で整理すると、y=3/4xですよね。
これと円の式を連立すると、x^2 + (9/16)x^2 -8x -(9/2)x +9 =0
となって、整理すると25/16x^2 - 25/2x + 9 =0
となって因数分解できないので、解の公式を使うとルートが残っ上、
全然違う値になるのですが、やり方はこれで合ってますよね?
ちなみにPの座標は(36/5 , 27/5),(4/5 , 3/5)です。

>>94
どうもすいませんでした。確かにおかしいですよね。解答みてみると途中の
式を答えと勘違いしてました。本当の答えは上の答えであってました。
100132人目の素数さん:02/05/26 08:45
>95
そのタイプの方程式が整数解をもつとすれば、
定数項の約数の中に整数解があると考えられる。(※)

たとえば、
X^3+X^2−8X−12=0
なら、±1、±2、±3、±4、±6、±12
の中から解を探すのがいい。

なぜ、(※)のようなことが言えるのか、ヒマな時にでも考えてみよう。
101132人目の素数さん:02/05/26 08:45
すいません
厨房ですがf(x)=・・・・・
このf(x)の意味が分かりません
教えていただけないでしょうか?
102132人目の素数さん:02/05/26 08:52
>99
解の公式の計算間違いでしょう。ちなみに分母を払えば
(5x−36)(5x−4)=0
数字が大きくなるから直線同士でやったほうが楽かも
103132人目の素数さん:02/05/26 08:54
f(θ)=sin^2θ + sin^2(θ+α) + sin^2(θ+β)がθに無関係な一定値になるよう
にα,βの値を求めなさいという問題です。ここで、sin^2(θ+α)をとくとき
なぜ加法定理で展開せずに半角の公式をつかって、
1/2-1/2(cos2θcoc2α - sin2θsin2α)のように処理するのですか?
その必然性を教えてください。

104132人目の素数さん:02/05/26 08:59
>101
xの関数(xの式)
f(x)=2x+1 のときf(3)ならx=3を代入して
f(3)=2*3+1=6+1=7
のように使う。
105132人目の素数さん:02/05/26 09:03
>103
やってみればわかる
先に加法定理で展開したらどれぐらい大変になるか。
106132人目の素数さん:02/05/26 09:04
>>104
じゃあ
f(x)=2x+1/x^2 x>0
このときのXの最小値はいくつでしょうか?
107132人目の素数さん:02/05/26 09:08
>>102
仰るとおりでした。つまらないところで間違えてました。因数分解も
できるんですね。
直線どうしだと当たり前ですが一次なるんで2つ求めなきゃ行けないぶん
楽ですね。どうもありがとうございました!!苦手意識がちょっと
無くなりました。
108132人目の素数さん:02/05/26 09:13
>106
ホントに厨房?何の問題を解いてるんだろう?
それは攻防の問題です。どの程度に教えていいかわからないので保留。
じゃあ」というところの関連がわからない。
109132人目の素数さん:02/05/26 09:13
>>105
やってみたんですが、つまりました。計算が大変なだけならいいのですが、
加法定理では解けないんですよね。今後どうしようか迷ったときに指針と
なるような必然性ってないですか?たまたまこの問題ではこうで・・・
ではなくて。加法定理で展開して2乗したらうまくいったのもあって
混乱してるんです。お願いします。
110某定時校生:02/05/26 09:19
>>106
ところでそのテの問題って、yの範囲指定はないのでしょうか(アホな質問だったらすみません)。
111132人目の素数さん:02/05/26 09:21
>109
計算を始める前に見通しを立てることです。
でも実際にはやってみないとわからないこともあって。
2乗を簡単にするのに半角もある、ということですね。
>>109=105?
だとして、θに無関係だってことは、θにどんな値を放り込んでも
それらを等しいとおけるということ
f(0)=f(π)=f(π/2)=f(π/3)=....
,................
113132人目の素数さん:02/05/26 09:24
>>108
マジで中3です
高校の内容でいいですから教えてください
114132人目の素数さん:02/05/26 09:25
>110
んー、まあ無いだろうね。
>106
f(x)=2x+1/x^2=x+x+1/(x^2) ≧ 3(相加・相乗平均の関係)
116132人目の素数さん:02/05/26 09:36
初等幾何って英語だとなんて名前ですか?
11781番目の素数さん:02/05/26 09:55
α∋χ β∋χ α≠βのとき、どんなことが言えるか。

だれか解いてみてください。 
118132人目の素数さん:02/05/26 10:02
>117
どんなことが言えるか、って言われても困るんだけど、、
α∩β∋χ
でいいのか?
11981番目の素数さん:02/05/26 10:13
>>118
どうも
>117-118
#(A∪B)≧2
とか。
121109:02/05/26 10:40
>>111
見通しですか。計算力がものをいいますね。困りました。

>>112
109≠105です。
意味はわかるのですが、その状態のままでは必要条件だから解けませんよね?
122132人目の素数さん:02/05/26 10:43
直円錐の高さをa、底面の半径をbとする。高さABをn等分し、その分点C1,C2,C3…Cn-1を
通って底面に平行な平面でこの直円錐を切る。これらの切り口の円と直円錐の底とを底面として、
各々の高さがa/nであるn個の直円柱の体積の和は、どのような式で表されるか。
また、この式を直円錐の体積を表す近似式とすると、誤差の限界はどのようになるか。

高校の積分なんですが、わかりません。誰か教えてください。
123132人目の素数さん:02/05/26 11:24
図形1:(0,0),(0,1),(1,0),(0,1)を頂点とした正方形
図形2:(1,0)を中心として、半径1の円
図形3:図形1に内接する円

図形1の内部にある図形2の弦と図形3の弦で囲まれた部分の面積を
初等的に計算する方法はあるのでしょうか。

昔の高校入試での出題らしいんですが、出した人みんなに微積分使わ
ないとでない、といわれてしまいました。
124132人目の素数さん:02/05/26 11:31
>>124
図形1って物凄い正方形だな。
125132人目の素数さん:02/05/26 11:37
>>124
図形1:(0,0),(0,1),(1,0),(1,1)を頂点とした正方形
に訂正。
失礼しました。
126数学ニガテっ子:02/05/26 11:47
整式P(x)=4x^3+ax+bがx+1で割り切れ、x−1で割ると余りが6
となるように、係数a、bの値を求めよ。
        …という問題です。教えてください!!
127132人目の素数さん:02/05/26 11:53
>>126
[剰余の定理]
整式P(x)を x-α で割ったときのあまりは
P(α) である。

これヒントにしてみそ。
128132人目の素数さん:02/05/26 11:55
>>126
実際にP(x)をx+1やx−1で割って余りを求めてみたら?
129 ◆jbTzuRvU :02/05/26 11:56
http://members.tripod.co.jp/shogakusei/mondai5.htm
小学生レベルらしいですけど、全くわかりません。
答えを教えて下さい。
>129
★小学生向け問題募集★
http://science.2ch.net/test/read.cgi/math/1020248263/
131 ◆jbTzuRvU :02/05/26 12:07
>130
どうもすいません。
132132人目の素数さん:02/05/26 12:32
確率1/2で上へ、確率1/2で下へ移動するとして、今0からスタート
するとします。これを9回試行したとき、最高値が3で、かつ、7回目
試行後の値も3である確率を求めよ。

これ、お願いします。
133132:02/05/26 12:47
>>132は、上へ行ったら+1、下へ行ったら−1の意味です。すいません。
134132人目の素数さん:02/05/26 12:50
>>129-130
あ、ごめん、その問題
小学生レベルじゃなかったかもしれない・・・
13546:02/05/26 12:51
>>51
「渦巻き」みたいな図形のときダメ。
13651:02/05/26 12:53
>135
なるほど。納得。
>>122
上からk番目の円柱の体積は、π*(kb/n)^2*(a/n) 。
∴ 和=納k=1,n]π*(kb/n)^2*(a/n)=πab^2/n^3*納k=1,n]k^2

この和は、円錐を外側から囲んでいる図形の体積。

一方、円錐の内側に入るようにn-1個の直円柱を入れると
上からk番目の円柱の体積は、π*(kb/n)^2*(a/n) 。
さっきのものとは、k=n が入るか入らないかの違いだけ。
誤差は、「外側−内側」以下なんだから、π*b^2*(a/n) より
小さいといえる。
138132番目の素数さん:02/05/26 13:38
xy平面上に4つの点O(0,0),A(-2,4),B(2,4),C(0,8)と
放物線y=x^2…(i)がある。また,BC上に点Dをとり,
ADと(i)との交点をEとする。このとき,AD=DEを満たす
点Dの座標を求めよ。ただし,AとEは異なる点とする。
139AKIRA:02/05/26 13:40
x^3-3x+2を因数分解せよって問題なんですが
因数定理を使って
P(x)=x^3-3x+2と置いてまではいいんですが
P(x)のxにkを入れて=0にしP(x)は因数x−kを持つとするのですが
このxに入れるkの出し方が分かりません
教えてくださいヨロシクです。。。


140132人目の素数さん:02/05/26 13:42
>>139
>このxに入れるkの出し方が分かりません
それを求める方法は、はっきり言って「ない」。
「これかな?」と思う数値を代入して
実際に0になるかどうかを調べていくしかない。

141132人目の素数さん:02/05/26 13:44
y=xじじょう+2x+1
まぁこれが解けたらお前等天才だな。あのヒルベルト問題だぞ
>139
x^3-3x+2 = (x^2+bx+c)(x-k)
と因数分解できる。右辺を展開して定数項を調べると
-ckになっている。つまり、2=-ck

よって、kとして考えるべき数は
定数項(この場合は+2)の約数
(この場合+1と+2と-1と-2の四つ)。
143140:02/05/26 13:47
>>139
なお少しヒント。
(最高次の係数が1の)P(x)がx-k を因数にもつなら、
そのとき k はP(x)の定数項(今の場合は2)の約数でないといけない。

つまり今の場合は、k の候補として
1,-1,2,-2
を調べればよい。
>>142-143
ケコーン と。
145AKIRA:02/05/26 13:55
>>140->>143さん有難うございます。おかげでできました(嬉)
またよろしくです。。
146AKIRA:02/05/26 14:10
(1−3i(虚数))^3をくずしてください。
なんどやっても答えがあわなくて
147助けてください:02/05/26 14:35
なぜ大学という所は習ってもない問題を平気でレポートという名の宿題にするんだろう?
しかも教科書のどこに書いてあるかもさっぱり分からないときた。
そんなわけで、次の悪い奴らを倒してください。お願いします。
[1] 4次正方行列のi行j列成分が次式で与えられるとき、行列を具体的に表せ。
但し、δij=1(i=j),0(i≠j)である。
1,(-1)^i+j 2,i+j 3,δij-δi,j+1 4,iδij
(3番のijと4番の右側のijは右下に小さく)
[2] Aをn次正方行列、Pを正則なn次正方行列とする。このとき、任意の自然数kに対して、
(P^-1 AP)^k=P^-1 A^-k Pが成立することを示せ。
[3] 次の等式を証明せよ。
tan(arccosx)=√1-x^2/x(x≠0)
一応ちゃんと真面目に考えたけどわかりませんでした。
大学の勉強はいったいどのように勉強するんでしょうか?
148132人目の素数さん:02/05/26 14:37
>>146

 (1-3i)^3
=1^3 + {1^2 * (-3i)} + (1 * (-3i)^2) + {(-3i)^3}
=1-3i-9+27i
=-8+24i

この際、虚数を扱っているので符号の向きに注意。


(-3i)^2は、
(-3i)^2= (-3)^2 * i^2 より負の数になる。
149132人目の素数さん:02/05/26 14:40
[1]書いてあるとおりに書けばいいだけだろう?
[2]書いてあるとおりに計算すればいいだろう。
[3]arccosの定義を調べればいいだろう。
>>147
>なぜ大学という所は習ってもない問題を平気でレポートという名の宿題にするんだろう?
習ってないから、「自分でいろいろ調べて来なさい」ということで
レポートを課すんだろ。

[1]行列の「i行j列成分」とは何なのかを教科書で確認せよ。
 本によっては「(i,j)成分」と書いてあるかもしれない。
[2]kに関する帰納法を使え。
[3]arccosがcosの逆関数ということと
tan x=(sin x)/(cos x) ということ、および(sin x)^2 =1-(cos x)^2 ということ
が分かっていれば示せるはず。
151AKIRA:02/05/26 15:00
>>148さんありがとです符号間違いでした
>>148
あの三乗の(a-b)^3の公式
はa^3-3a^2b+3ab^2-b^3じゃないんですか??
>>152
(a-b)^3 = {a+(-b)}^3
154132人目の素数さん:02/05/26 15:25
問)直角三角形ABC(∠A=90°)の斜辺BCに点Pをとり、PからABに
下ろした垂線とABの交点をQ,PからACに下ろした垂線とACの交点を
Rとする時、三角形PQRの面積が最大になる点Pの位置を求めよ。

解)CP=x、BC=a、AB=c、AC=bとする時、
  x:PR=a:cよりPR=cx/a
  (a−x):PQ=a:bよりPQ=b(a−x)/a
  よって三角形PQRの面積S=1/2*cx/a*b(a−x)/a
  〜省略〜 よってx=a/2つまり点PがBCの中点にある時
  面積が最大となる。

これ、あってますか?
あの、>>45の方の問題の模範解答をお願いできませんか
僕は、授業でその問題に当たってるんです(同じ学校かな?)
>>47さんの書いてることを見ても理解できません
明日、黒板に書かなくちゃならない羽目になってるんです・・・
救済をお願いします・・・・・・
>>155
まずわかるとこまで自分で書けや
>>146
 (1-3i)^3
=1^3 + 3{1^2 * (-3i)} + 3(1 * (-3i)^2) + {(-3i)^3}
=1-9i+27i^2-27i^3
=1-9i-27+27i
=-26+18i
>>148公式違ってる
>>103
f(0)=f(π/2)
f(π/2)=f(π/6)
sin(α)=A
sin(β)=B
とでも置いてやったらA,Bって求まらない?
159132人目の素数さん:02/05/26 15:51
点p(x,y)が
lxl+lyl≦1
x+y=X xy=Y
のときq(X,Y)の動く範囲をXY座標に図示せよ
という問題でx,yが実数解として存在するのを示さなければいけないのは何でですか。
なんでX^2-4Y≧0も条件として必要なのですか?
x,yは最初に決めた点だから、必ず存在するはずだと思うのですが
160132人目の素数さん:02/05/26 15:53
点p(x,y)が
lxl+lyl≦1
x+y=X xy=Y
のときq(X,Y)の動く範囲をXY座標に図示せよ
という問題でx,yが実数解として存在するのを示さなければいけないのは何でですか。
なんでX^2-4Y≧0も条件として必要なのですか?
x,yは最初に決めた点だから、必ず存在するはずだと思うのですが
161132人目の素数さん:02/05/26 16:27
>147
大学は原則自分で学ぶところだからね。図書館と、教えあえる友達は大事だよ。
その程度のレポートでぶちぶち言ってたら先が思いやられるよ。 
[3] t=arccosx と置けば x=cost、それでもわからなければ右辺から
式変形してみる。
162132人目の素数さん:02/05/26 16:42
点(,x , y)が、原点Oを中心とする半径1の円周上を動くとき、
x^2 + 4xy - 2y^2の最大最小を求めよという問題なのですが、
この問題を線形計画法で解くのは無理ですよね。
それは4xyの項が入っているからですか?
4xyがなかったら上の解法で解けますよね?
163147:02/05/26 16:42
[1]がどうにも分かりそうにないんですが・・・
[2][3]はなんとかなりそう・・・かな?
164132人目の素数さん:02/05/26 16:49
>>163
その前にあんた行列って分かっている?
高校出たの?
165数学激ドキュソ:02/05/26 16:52
すいません、二次関数の問題でどうしてもわからないモノが
あるので誰か教えてくださると嬉しいですm(__)m


O≦x≦3におけるP=(x2-2x-1)2+2(x2-2x-1)の最大値は_、
最小値は__である。
166165:02/05/26 16:54
あ〜すいません、二乗が紛らわしかったですね・・・。
訂正。↓


O≦x≦3におけるP=(x^2-2x-1)^2+2(x^2-2x-1)の最大値は_、
最小値は__である。

です。よろしくお願いしますm(__)m
167132人目の素数さん:02/05/26 17:00
(1) X=x^2-2x-1とおいて、Xの範囲を求める。

(2) (1)で求めたXの範囲で、P=X^2 +2Xの最大値を求める。
168132人目の素数さん:02/05/26 17:00
>>162 係数ついてるから無理?
169132人目の素数さん:02/05/26 17:01
160の回答は?
>>147
4次正方行列
って言葉の意味と
i行j列成分
って言葉の意味は分かってるの?
>169
マルチぽ(略

172160:02/05/26 17:06
うえええええええええええええええん
こたえてよおおおおおおおおおおおお
>>163
[1]どうせ1つの行列は16マスしかないんだから、1つ1つ
計算していけばいいだろう。たとえば2行3列成分だったら、

(1) (-1)^(2+3) = -1
(2) 2+3 = 5
(3) δ(2,3)-δ(2,3+1) = 0
(4) 3δ(2,3) = 3
174147:02/05/26 17:08
縦4横4の行列。
i行のj列にある数?
175132人目の素数さん:02/05/26 17:09
>>174
それわかるなら、後は指示通りに書くだけでしょ?
176147:02/05/26 17:13
???その指示の意味が分からないんですけど・・・
すいません・・
>147
2番目の問題だけ書くと、
「i行j列成分が (-1)^i+j」 というのは

1行1列成分が (-1)^1+1
1行2列成分が (-1)^1+2
1行3列成分が (-1)^1+3
1行4列成分が (-1)^1+4
2行1列成分が (-1)^2+1
2行2列成分が (-1)^2+2

4行4列成分が (-1)^4+4

という意味。
178177:02/05/26 17:22
1番目の問題か。
179160:02/05/26 17:22
自力でわかったからもういいよ
180132人目の素数さん:02/05/26 17:24
ぷっ。分かっていない模様。
181173:02/05/26 17:24
(-1)^i+j というのを、脊髄反射的に
(-1)^(i+j)のことだと思いこんでいた。
182147:02/05/26 17:25
すみません。やっと分かりました。いわれたら確かに簡単なことですね。
183160:02/05/26 17:27
x.yを定めればX.Yは定まるが、逆は必ずしも成り立たないってこと
>>183
それが分かったことと、お前の提示した疑問の解決では大きな隔たりがあるけど?
185132人目の素数さん:02/05/26 17:32
186160:02/05/26 17:34
>>185 それ僕じゃないです
でも素直に教えて欲しかった(;_;)
>>186
“素直”って何?
>>186
その問題は俺にとってはなかなか歯応えある問題で、
正直言うと今も考えてる。一応数学科なんだけどな。
>187
他人の命令に盲目的に従順に従うこと
190160:02/05/26 17:43
命令なんてしてませんが。
お願いしただけです
つぎのかたどーぞ
192132人目の素数さん:02/05/26 17:49
>>160 俺はx+y=X、x-y=Zとおいて問題文からx,yを消しちゃうけどね
>160
分かりやすいように説明する為に考えてる
ってこともあるからもう少し長く待ってね
>193
そういう問題じゃないだろ。
マルチポストだからマズかったんであって。
196160:02/05/26 18:02
マルチについては謝罪m(__)m
もう一つの必要条件は最初の式を2乗すれば出ます
>>196
マルチするときはコテに注意しろ。
198数学ニガテっ子:02/05/26 18:37
2bの範囲の問題で
二次方程式x^2−10x+k=0において、2つの解の比が2:3であるとき、定数kの値と
2つの解を求めよ。…という問題です。教えてください。
        
>>198
2つの解をa、3aとおく。すると解と係数の関係より…
200199:02/05/26 18:42
失礼、書き間違えた。適当に直してね。
201132人目の素数さん:02/05/26 18:45
a,3/2aとおいて代入するだけ
202132人目の素数さん:02/05/26 18:49
>>200
手抜くなよ。w

解は、2a, 3aと置ける。
解と係数の関係より、2a + 3a = 10, (2a)*(3a)=k
あとは解いてね。
y=x^2-2x (x>1)
の逆関数の導関数を求めよ
という問題で、dx/dy=1/dy/dxの公式を使ってみたのですが
答えがあいません。何故でしょうか?
204132人目の素数さん:02/05/26 19:26
すいません、sin22.5°やcos22.5°というのは
二倍角を使って解くのですよね?
忘れてしまって。。。お願いします、教えてください。
>204
yes.
もちろん使わなくても出るけど、
使うのが普通。
206165:02/05/26 19:38
ベクトルの問題で

3点A(2.3.−2)、B(0.3.0)、C(0.1.−2)を頂点とするΔABCを
1つの面とする正四面体の、他の頂点Dの座標を求めよ。


っていう問題なのですが・・・誰か助けてくださいm(__)m
207132人目の素数さん:02/05/26 19:41
>>205
ていうことは...
すいませんどのような式になるか教えてください
お願いします。
208132人目の素数さん:02/05/26 19:42
やっぱり>>123(ただし図形1の最後の座標(1,1)に訂正)は初等的には不可能みたいですね;;
209205:02/05/26 19:49
>207
やっぱそれが聞きたいんだよね。(w

α=22.5°とする。
cos2α= 2 cos^2 α - 1
なので、
2 cos^2 α - 1 = 1/√2
cosαを求めて、そこからsinαも求める。
符号に注意
>206
全然ベクトル使ってないけど、
Dの座標を(x,y,z)とおいて、
|AD|=|BD|=|CD|=|AB|
を解けば終わり。

どうしてもベクトル使いたければ、
三角形ABCの重心をGとすると、
ベクトルGDはベクトルAB,ACに直交し、
長さがベクトルABの√(2/3)倍になることを利用。
など。
211132人目の素数さん:02/05/26 19:57
>>209
cos22.5°は(√3+1)/2
sin22.5°は(√3-1)/2
でよろしいのでしょうか?
212209:02/05/26 20:01
>211
(√3+1)/2 > 1
ですが何か?

って計算ぐらい自分で何とかしろ。
213132人目の素数さん:02/05/26 20:04
>>212
え?違うのですか?
>213
何でコサインが1超えるんだよ。
215132人目の素数さん:02/05/26 20:12
cos22.5°は(√3-1)/2
sin22.5°は??
216132人目の素数さん:02/05/26 20:28
αを1の5乗根とする。β=α+1/αのとき、β^2+βの値を求めよ。

という問題です。どなたかお願いします。
217132人目の素数さん:02/05/26 20:40
>>216
β^2+βをαの式で表してみ?
>>216
こねくり回している内に答えが出てしまうような問題だけど、
これを解くことによって何か得るものがあるのだろうか

なんつて
>>216
β^2+β を αの式で書いてみそ。
そしてその分子に着目。

>218
正五角形が作図可能であることの
証明になってますが何か?
>>216
α=a,β=bと書き直す

a^5=1
(a-1)(a^4+a^3+a^2+a+1)=0
a=1のとき(a^4+a^3+a^2+a+1)=5
a≠1のとき(a^4+a^3+a^2+a+1)=0

b^2+b
=(a+1/a)^2+(a+1/a)
=a^2+2+(1/a^2)+a+(1/a)
=a^2+a+1+(1/a)+(1/a^2)+1
=(1/a^2)(a^4+a^3+a^2+a+1)+1
=6,1
222132人目の素数さん:02/05/26 20:49
本当にお願いします。
cos22.5°はいくつなんですか
お願いします。
>222
2 cos^2 α - 1 = 1/√2
を計算間違いせずに解け。
224132人目の素数さん:02/05/26 20:52
√(2+√2)/2になるのですが。。。
>224
そういう何がいいたいんだか分からない書きかたして、
いったい何が楽しいんだ?
聞きたいことがあるならちゃんと聞け。
>>51(元の問題は>>39)
例えば、幅のあるS字形を考えてごらん。
中心線を点対称から歪めても、太さで調整できそうじゃない?
227132人目の素数さん:02/05/26 20:58
出なおしてきたまえ
228132人目の素数さん:02/05/26 20:59
>>221
> α=a,β=bと書き直す
ここのところがよくわかりません。
詳しく教えてくさい。お願いします。
229221ではないが:02/05/26 21:01
>228
αとかβとかをキーボードで打つのがめんどくさいので、
代わりにaとかbとかを使います、
って宣言してるだけ。
23051:02/05/26 21:02
>226
さんくすです。
231132人目の素数さん:02/05/26 21:02
>>225
http://jbbs.shitaraba.com/computer/1348/
こちらで詳しく説明いただけませんか。。。
232225:02/05/26 21:04
>>231
激しくブラクラ。(w
233132人目の素数さん:02/05/26 21:05
√の開閉法を教えてください!
>>233
何を聞いてるのかよくわかりません。
スマソ
>123
亀レスですまんが、
一応初等的に(積分を使わずに)出せるぞ。
ただ、答えは逆三角関数を使わないと
表せないと思う。
それは積分を使っても分かる。

参考のために(初等的な方法での)答えを書くと、
t, t' を
sin t = (-1 + √7 )/4
sin t' = (5-√7)/8
の解とすると
面積は
7 π/16 - √7/8 - t/4 - t'
になる。
236233:02/05/26 21:20
>234
http://www.tbs.co.jp/company/seishonen-houkoku-tokuban.html
「うたばん」の2時間特別番組である「とくばん」3月26日の放送で、石橋貴明氏が
Zoneメンバーにニックネームを付けるというくだりの中で、
MIZUHOに対して「ちんちんの先」という
ニックネームを付けたことに関して、TBS側が謝罪を行った。
237233:02/05/26 21:30
物理の答えなんかで、√257とか出てこんなの開けねーよ!って時に無理やりやる方法
らしいです。授業で夢うつつながら先生がやってるの見ました。当方、高2です。
238132人目の素数さん:02/05/26 21:37
239132人目の素数さん:02/05/26 21:39
>>237
何かの本で見たことあるけどかなり複雑で手間だったと思う。
それよりは16~2=256なので√257≒16とした方が早い。
240233:02/05/26 21:42
なるほど字が違ってたのか…どおりで変なおっさんの日記しか見れないと思った。
どーもです。
241233:02/05/26 21:44
あ、239さんもありがとうございます。

ところでこれって証明できるんですか?
242132人目の素数さん:02/05/26 21:46
線形代数なんですけど、4次正方行列の固有方程式って、
どうやって出せばいいのでしょうか。
サラスの方法は使えないですよね?
お願いします。
243132人目の素数さん:02/05/26 21:51
余因子(w
244132人目の素数さん:02/05/26 21:56
>>242
ていうか、何も理解できてないようにみえます。
きっちりやり直したほうがいいよ。
245239じゃないけど:02/05/26 21:58
>240
開平法は単に一桁ずつ足していく方法。
一桁足すごとに二乗して元の数を超えない
最大の数を求めていくもの。
だから別に複雑なアルゴリズムではないよ。
246132人目の素数さん:02/05/26 22:05
ほんとですか
247239じゃないけど:02/05/26 22:13
>246は漏れに対して?

もちろん、一桁決めるのに最低2回は
面倒な計算をしなくてはならないけどね。
例えば、
257 だったら
16.0 の次の桁を決めるには
16.03^2 と 16.04^2 を計算する必要アリ。
もっとも、16.0^2 に対する補正をするだけだけど。

それを筆算で効率よくやるのが開平法。
ちなみに開立法というのもある。
というわけで n 乗根の計算への拡張も自明だろ。
248>:02/05/26 22:18
>239
物理の問題だったら
 開平計算法を知ってるかどうかがことの本質ではないから
 電卓で間にあわせるべき。。

あるいは (1+x)^(1/2) の
テーラ展開の1次の項で打ち切ってもOKじゃないかな
( (1+x)^y = 1 + yx で近似 xが十分小さいとき)


(257)^(1/2) = (256+1)^(1/2) =
= 256^(1/2) * (1 + 1/256) ^(1/2)
--> 16 * (1 + 1/512) としちゃえ
         16.03125
実際の257の平方根 16.0312954.....
小数点2桁ぐらいでいいならこれで十分



249233:02/05/26 22:32
>( (1+x)^y = 1 + yx で近似 xが十分小さいとき
十分とは?具体的にお願いします。

>小数点2桁ぐらいでいいならこれで十分
何桁までこれで十分なんですか?
誤差と精度の説明を省いた近似に意味はあるのか?
251132人目の素数さん:02/05/26 22:36
円周率が3.14ですむ>>250
例えば√257だったら
適当な数、例えば16当たりを一つ用意それを覚える。

※今覚えている数を二乗し257を足し、覚えている数の2倍で割る。
その半分を新たに覚える。
そのさっきまで覚えていた数を忘れてしまう。
 
※の操作を5回くらい繰り返すと良い近似値が得られる

16->(16*16+257)/32=513/32
513/32->((513/32)^2+257)/(2*513/32)=((257+(513/32)^2)*16)/513
=(257*32*32+513*513)/(513*32*2)
=(263168+263169)/32832=526337/32832
................................
(513/32)^2=257.0009765625
(526337/32832)^2=257.000000000927695226261451766735
253132人目の素数さん:02/05/27 00:00
a≠0 とする。関数 y=ax^2+bx (0≦x≦1) の最大値が 16 、 最小値が -9 のとき、
定数 a,b の値を求めよ。
という問題です。

y=ax^2+bx
=a(x+b/2a)^2-b^2/4a
と平方完成して、a と -b/2a について場合分けすればいいんですか?
>>253
いいんじゃない?
|2x/{(x^2+1)^2}|<1となることの証明ってどうやればいいのですか?
256132人目の素数さん:02/05/27 00:09
>>254
場合分けをどうすれば良いのかよくわかんないんです。
>>256
最大値が16、最小値が-9になるような状況を考えてみる。

軸が[0,1]区間にない時とある時、上に凸の時と下に凸の時、ってね。
>>255
すなおにf(x)=2x/{(x^2+1)^2} とおいて、
微分して増減を調べればどうでしょ。
>>255
2x≦1+x^2≦(1+x^2)^2
等号が同時に成り立つxは存在しない。
260259:02/05/27 00:32
もう少し説明。
-(1+x^2)^2≦-(1+x^2)≦2x
やはり等号は同時には成り立たない。

以上より、-(1+x^2)^2<2x<(1+x^2)^2
∴|2x|<(1+x^2)^2
x^4+x^2+(x-1)^2>0 ⇔ 2x<(1+x^2)^2
x^4+x^2+(x+1)^2>0 ⇔ -(1+x^2)^2<2x
263253=256:02/05/27 01:00
わけわかんなくなってきたんで、解説お願いします。
264132人目の素数さん:02/05/27 01:11
>>235
わかりました
ということは、高校入試に出た、というのは勘違いですね
ありがとうございました
>264
昔、というのがどのくらい昔かによる。
本当に昔は中学校で三角比をやっていたのでは?
旧制高等学校の入試か?
267132人目の素数さん:02/05/27 01:37
あのさ〜楕円の方程式でP(α、β)を通る接線の方程式の
公式ってどうやったっけ?
268265:02/05/27 01:50
>266
そんなに前になるのか・・・
うーむ。
269132人目の素数さん:02/05/27 02:58
N-1
煤@1/(z^2k + z^3k) (但しz^n=1)
k=1

解いてもらえませんか?何時間か考えたけど、僕にはできませんでした。
1/(z^2k + z^3k) は、zの2k乗 プラス zの3k乗 が分母で、分子が1です。

どうかよろしくおねがいします。


270269:02/05/27 03:02
シグマの書き方、ルールと相違がありました、ごめんなさい。
271269:02/05/27 03:08
あ、ごめんなさい。Nはnの書き間違いです。

n-1
煤@1/(z^2k + z^3k) (但しz^n=1)
k=1

です。お願いします


z=1の時とそうでない時は同じには扱えないことはわかるかな?
普通こういう問題の場合zが実数か否かは書いてある筈だと思うが
ま、書いてなくてもz=1の場合は明らかだと思うが。
そうでない場合が問題だよね。
そうで無い場合、z,z^2,z^3,...,z^(n-1)はみなすべて異なり
共にz^n=1を満たす。
Σ[k=1,n-1]は方程式z^n=1の1以外の根すべてに渡る和と置きかえられる
わけだ。以降このΣをΣ'と書く
Σ'1/(z^2+z^3)を求めよとのことだよね。
1/(z^2+z^3)=z^(n-2)/(1+z)
1/(1+z)は実はzの多項式で表される。
これを使ってやることがポイントだ。
あと解と係数の関係。さぁ、ガムバレ
273272:02/05/27 03:19
そうでない場合、z,z^2,z^3,...,z^(n-1)はみな異なり
というのはウソ(n=8)
i,-1,-i,1,i,-1.-i,1
上の話はnが素数の時
合成数の場合はちょっと違ってくる。
>>272
これは直感なんだけど、nが

(1) 6の倍数のとき
(2) 偶数で3の倍数でないとき
(3) 偶数でなく3の倍数のとき
(4) 偶数でも3の倍数でもないとき

に場合分け、になってくるんじゃないかと。
275269:02/05/27 03:33
>>272
早々のレス、ありがとうございます。励みになります。

>z=1の時とそうでない時は同じには扱えないことはわかるかな?
そうですね。わかります。

>そうで無い場合、z,z^2,z^3,...,z^(n-1)はみなすべて異なり
>共にz^n=1を満たす。
これも素直に飲み込んでしまったんですが・・・。
うーん。素数の場合と合成数の場合は違うんですか・・・。
ごめんなさい。273のn=8の例も理解できていません。

もう少し詳しく教えていただけたら幸いです。
276272:02/05/27 03:33
合成数の場合も、素数である場合が大きなヒントとなるので、
nを最初は素数とすべし。n=3,5,7当たりで実験
n=3
z=ωもしくはω^2の場合
1+z+z^2=0
z^2+z^3=1+z^2=-z
z^4+z^6=z+1=-z^2
1/(z^2+z^3)+1/(z^4+z^6)=-1/z-1/z^2=-(1/z+1/z^2)=-(z+1)/z^2
=1
277269:02/05/27 03:35
>>274
レスありがとうございます。

確かに、場合分けが必要なのかもしれません。
僕も、偶数と奇数で少し値の変化の法則に違いが
あるとは思うんですが、そのように分けてみる方法は
思いつきませんでした。参考にさせていただきます。
n=2,z=-1の時はどうなるの?
>>278
定義できると思うの?
280269:02/05/27 03:43
>>272
あ、なるほど。
ためしにやってみると、n=5のとき値は2とでました。

偶数奇数ではなくて、素数かそうでないか、を基準に
すればいいわけですね。
やってみます。
ありがとうございます。

しかし、
>Σ[k=1,n-1]は方程式z^n=1の1以外の根すべてに渡る和と置きかえられる
>わけだ。以降このΣをΣ'と書く
>Σ'1/(z^2+z^3)を求めよとのことだよね。

のところがやはり理解できませんでした。
よければもう少し説明もらえますか?お願いします
281269:02/05/27 03:52
うーん。これはちょっと難しいんだよね。
pを素数とする時、z≠1でなければ、z,z^2,z^3,...,z^(p-1)はすべて異なる
ってことなんだけど。
もし複素数知ってたら、z=exp((2πik)/n)(k=1,2,...,(n-1))って書ける
んだけど。(これは一般に合成数でも可)
この形で考えてみたらどうだろ?
282269:02/05/27 03:54
z≠1でなければ⇒z≠1ならば
賢明な貴方はわかってくださるでしょうが。
283269:02/05/27 03:56
複素数知ってたら⇒指数関数による1のべき乗根の表現
度々スマソ
284269:02/05/27 04:04
>>269
恥ずかしいながら
z=exp((2πik)/n)(k=1,2,...,(n-1))
は皆目わかりません・・・。

僕が理解している(と思っている)のは
高校の文型範囲(数学1A2B)だけなんです。
厄介だとは思うんですが・・・。

実はこの問題は

z^n=1のとき、次の式の値を求めよ。

z^2(n-1)/(1+z) + z^2(n-2)/(1+z^2) + ・・・ + z^4/{1+z^(n-2)} + z^2/{1+z^(n-1)}

という問題を解いているうちにでてきた式なんです。
もしかしたら方針に間違いがあったのかもしれませんので、
一応記載しておきますね。
285269:02/05/27 04:06


>>274
の間違いでした。
286269:02/05/27 04:08

ごめんなさい、更に間違い。。
>>272
です。
287269:02/05/27 04:15
>>284
あ、いやいいんだ。>>281で「ちょっと難しい」といった部分、自分で
考えもらうのに便利な道具として、もし知ってたらそれで考えてもらう
のがいいかなと思って聞いただけ。この問題解くのには直接関係無い
でしょう。
>>284の問題の形にせよ、n=2,z=-1の時はナンセンスになるよね。
n=4 z=-1,z=iの時だってダメになる。
nが偶数の時はほぼ完全に定義できないようなzがある。(具体的には-1)
nが素数の時は、>>269で述べた事実が成り立つんで、きれいな値
(zの取り方に関係しない)が出てきそうだけど、それ以外の奇数の合成数
の場合はどうかな?
もうそろそろ落ちます。あまり役に立たなかったみたいで申し訳無い
けど、がんばってください。
288269:02/05/27 04:20
>>272
長い間ありがとうございました。

なるほど。だいたいわかってきました。
素数からせめていきます。

大きなヒントをありがとうございました。
もうしばらく粘って頑張ってみます。
289272:02/05/27 04:20
281-283,287は272のレスで、質問者269の名前を間違って使ってました。
269の自作自演と感じられた方はそれは誤解ですので。
290132人目の素数さん:02/05/27 04:34
あのさ、1/(z^2k + z^3k)って、部分分数分解できるよね。

1/z^2k - 1/z^k + 1/(z^k + 1)

この、前2項のΣは簡単に求まる。
問題は第3項のΣなんだが…

この方法でどうにかならないものだろうか。
291132人目の素数さん:02/05/27 04:36
サイズがkの部分集合ってどういうことですか?
>>290

 Σ1/(1+z^k)
= Σz^n/(z^n+z^k)
= Σz^(n-k)/(z^(n-k)+1)

(足す順番を変えて)

= Σz^k/(1+z^k)

よって、Σ1/(1+z^k) = (n-1)/2
293290:02/05/27 04:56
>>292
わかった。逆順に並べて、辺々足すわけね。
すると各項の和が、どれも1になる、と。
解決したじゃん。

269本人はどこいったんだ。
294269:02/05/27 04:58
>>290
>>292
部分分数に分解できるんですか!
これは大きいですね。

じゃあ、これがすべてあっていれば答えは求まる
はずですよね、ありがとうございます。

しかし、>>272さんが言ってた「素数」の事は
この解法から出たこたえに反映するのかな。
やってみます。
ありがとうございます。
295269:02/05/27 05:02
>>293
すこし落ちて問題に取り組んでいました。
分解は試みたんですが僕ではどうにもなりませんでした。

どういう方法で分解後の形を導くんですか?
下らない質問で恐縮ですが今後のために教えてください。
マイナスとマイナスをかけたら+になる理由
297290:02/05/27 05:05
>>295
部分分数のことかい?
298269:02/05/27 05:08
>>297
はいそうです。
1/(z^2k + z^3k)を部分分数にどうやってしたのか、
僕にはできなかったのでその思考過程を聞きたいと
思っていますが、、

299269:02/05/27 05:12
>>292

> Σ1/(1+z^k)
>= Σz^n/(z^n+z^k)
>= Σz^(n-k)/(z^(n-k)+1)

ちょっと疑問に思ったのですが、
上の式変形、
 Σ1/(1+z^k)
= Σz^n/(z^n+z^(n+k))
にはならないんですか?

勘違いだったらごめんなさい。
300269:02/05/27 05:15
>>292

あ!
そうか、z^n=1ですもんね。

失礼しました。
301290:02/05/27 05:16
>>298
まず z^2k + z^3k は、z^2k(1 + z^k)とできるわけだ。

1/(x^2(x+1))が分解できることの類推で、こいつもできそうだ、と思う。

そこで A/z^2k - B/(z^k(1+z^k)) とおいてやってみると、A=B=1でできる。

次に後ろの 1/(z^k(1+z^k)) を同様に分解する。こんな感じかな。
302269:02/05/27 05:20
そうか、なるほど。

やはり既知の事実をうまく応用することが大事なんですね。

>1/(x^2(x+1))が分解できることの類推で、こいつもできそうだ、と思う。

これがすごくキーですね。

ありがとうございました。
303290:02/05/27 05:29
おれも最初は272氏と同様の方向で考えてたんだよね。
(実は>>274=俺)
でもあまりにも複雑になりすぎるから、方針転換したんだ。
そうしたらたまたまうまくいっただけ。

1のn乗根を用いた場合分けの考え方でいっても、
結局同じ結論に到達するのだろう。(じゃなきゃ困る)
304269:02/05/27 05:37
>>303
なるほど。
実は僕も素数の方、あれだけヒントもらったのに手も足もでなかった
ので、もう一度このスレを覗いたんです。

今、結果を出していますが、素数から攻める方法もやってみて
比較してみます。僕は文系の人間ですが、数学は一つのもの
にいろんなアプローチの可能性があるので、決して得意では
ありませんが、昔から好きです。文系の方法としても多く学ぶ
ところがあると思っています。

駄文、申し訳ありません。
ありがとうございました。
305質問です:02/05/27 06:48
P=P1+(P0-P1)*exp(-t/γ)
の計算はどうやってするんですか?
expの意味がいまいちわかってません。
例えば、P=1000 P1=1 P0=2000 t=0.38 でγを求めたいんですが・・・
わかる方がいたら、教えてください。
306132人目の素数さん:02/05/27 06:58
exp(-t/γ)=なんちゃら
の形に変形してログを取れ
307数学ニガテっ子:02/05/27 06:59
教えてください!
 複素数と方程式の解というところの問題なのですが…
【問】縦12cm、横18cmの長方形の厚紙の四隅から、合同な正方形を切り取った残りで
ふたのない直方体の箱を作り、箱の深さは2cm以上、容積は160cm^3にしたい。切り取る
正方形の辺の長さを求めよ。・…という問題ですよろしくお願いします
308132人目の素数さん:02/05/27 07:09
文字に大小関係を導入するときって、「・・・をx,y(x≦y)とおく」のように自分が
導入した文字にしかできないのですか?例えば問題文にすでに書いてある文字に
大小関係を導入するのは行けないのですか?
>308
「x≦yとしてよい」
とでも書けば?
310132人目の素数さん:02/05/27 07:33
正確に書こうと思えば、
「・・・なのでx≧yとしても一般性を失わない」
かな
311132人目の素数さん:02/05/27 07:43
>>309>>310
なるほど、そう書けば問題文中の文字に制限を加えても良いんですね。
勉強になります!!
312132人目の素数さん:02/05/27 07:45
点(,x , y)が、原点Oを中心とする半径1の円周上を動くとき、
x^2 + 4xy - 2y^2の最大最小を求めよという問題なのですが、
この問題を線形計画法で解くのは無理ですよね。
それは4xyの項が入っているからですか?
4xyがなかったら上の解法で解けますよね?
313132人目の素数さん:02/05/27 09:02
素数は無限である事を示せって問題なんですが

有限と仮定しそれらをP(1).P(2)....P(n)とする。
N=P(1)P(2)....P(n)+1とすると
Nは素数
よって矛盾

って教わったのですがいきなりNは素数ってやるのは
駄目なのではないでしょうか?
一先ずNを1以上の整数として議論しないとなんか変な気がします。
314132人目の素数さん:02/05/27 09:19
>313
この方法でも問題ないです。
Nは、P(1)、・・・P(n)のどれで割っても1余るから、合成数ではない。
よって素数。
315132人目の素数さん:02/05/27 09:44
>307
切り取る正方形の辺の長さをx(cm)とおく。すると直方体の箱の3辺の長さは
それぞれ、x, 12-2x, 18-2x(cm)となる。箱の容積が160cm^3なので、
次の式が成り立つ。
x(12-2x)(18-2x)=160  整理して、x^3-15x^2+54x-40=0・・・@
@式の左辺のxに4を代入すると0になる。よって、@の左辺は(x-4)を
因数にもつので、@の左辺を(x-4)で割り算して、次式を得る。
(x-1)(x-4)(x-10)=0
これを解いて、x=1,4,10
問題文より、xは2以上。また、もとの長方形の縦の長さ12cmの半分の6cm
よりも大きくxをとることはできないので・・・以下略。
316132人目の素数さん:02/05/27 10:36
この2問が解りません・・・どう計算したらいいでしょうか。

次の物体(質量M,一様な密度ρ)のZ軸まわりの慣性モーメントを
求めなさい。

(1)各辺の長さが2a、2b、2cの直方体(中心は原点O)

(2)半径aの半球(原点Oは半径aの球の中心と同じ)
>316
どちらもz軸に垂直な平面で
スライスして積分。
(2)は極座標表示を使う

>>312 2次式の最大化問題を「線形」計画法で解こうというのは
面白い発想だね。そう、この問題なら xy の項さえなければ
X = x^2 と Y = y^2 の一次式になるから、X,Y >= 0 の付帯
条件をつければLPで解けそうだね。

まあ、表記の問題なら x = cos(t), y = sin(t) とおけば
目的関数は cos(2t) - 2sin(2t) となるから、tで微分して
極値をだすほうが簡単と思うが。
319318:02/05/27 11:04
おっと、目的関数は cos(2t) + 2sin(2t)。まあオレの
書くのを信用するヤツもいないものだが。
>>318
線形計画法というよりも、「幾何学的」解法のことを言ってると
思われ。要するに図形的イメージで解けないかと。
321132人目の素数さん:02/05/27 11:31
>>318>>320
お返事どうもっす。解答では円のパラメーター表示をつかって最後は
三角関数の合成でフィニッシュしてました。一次で線形計画法って
いわないのかな?ともかく320さんのいうように幾何的に解けないのかな
と思って。でも、考えたんすけど、幾何的にも無理ですよね?
x^2 - 2y^2ってどんな図形かわからないし。(当方文型)
普通に解きます。
>>321
x^2-2y^2=k
は正弦曲線を表す(ウソ

適当に回転すればxy=uという形に直せる筈なんだけど、ここではそんな
こと関係無い。x^2-2y^2+4xy=kも適当に回転させれば、x^2-2y^2=kという
タイプになる筈。(3:4:5の直角三角形に関係する角度の回転)
もちろん制約条件であるx^2+y^2=1は回転によって変わらない。
y=±√(x^2-k)/2だから|x|<√kの範囲ではyは定義されない。
1>√k>0でなければ円と交点を持ちようがない。これはもとの問題
の解決に非常に役に立つ気がする。
323 :02/05/27 13:11
厨房質問ごめん!
円柱の体積の公式を至急教えてください!
底辺かける高さ割る5
なんちゃって
円の底面積かける鷹さ
円柱の高さをh、円の半径をr、円周率をπと置くとする場合、
円柱の体積Vは、V=2π(r^2)h  ただしr^2はrの二乗のこと
326ありがとう :02/05/27 13:25
えー!?混乱してきた。
体積=π×半径×半径×高さ
で合ってる?
327132人目の素数さん:02/05/27 13:28
>>322
すいません、言ってる意味がわかりません。(当方文型)
ホント申し訳ない。
>>326
体積=(円の)面積×高さ なんだけど、面積=π×半径×半径 であってたっけ。
だったたら合ってるとおもうよ。
329132人目の素数さん:02/05/27 13:29
例えばy=3(x+3)^2 + 6(x+3)の最小値を求める問題があって、x+3=tと
おきかえると、y=3t^2 + 6t になると思うのですが、y=(tの式)になって
ますけど、この書き方は良いのですか?y=(xの式)が普通だと思うんですけど。
さらに、グラフを書くとき、横軸をt縦軸をyと書いても良いんですか?
330132人目の素数さん:02/05/27 13:30
>>314
ネタなのかどうか知らんが、嘘は言うな。
それと、aとbの方程式をグラフで書くとき、縦軸をb,横軸をaにすると
思うんですが、これは慣習ですか?横軸をb、縦軸をaとしちゃったら
まずいですよね?これはそうしなきゃいけないレベルの話ですよね?
>>329
この場合、x+3=tという等式の意味は、y=(tの式)の図を左へ3ずらすと
y=(xの式)になる、ということになります。グラフの書き方は、縦軸が左辺(この場合はy)でいいのでは?
333132人目の素数さん:02/05/27 13:33
>>312
>4xyがなかったら上の解法で解けますよね?

そんときは x^2 - 2y^2 = 3x^2 - 2(x^2 + y^2) = 3x^2 - 2 だから自明

>この問題を線形計画法で解くのは無理ですよね。

線形計画法を知らないので何とも言えないが、u=x-2y、v=2x+y とおくと
u^2 + v^2 = 5 のときの (-3u^2 + 2v^2)/5 の最大最小を求める問題になる
334132人目の素数さん:02/05/27 13:39
n=整数で0より大きいとき
n~2と(n+1)~2の間に必ず素数が存在するって聞いたんですけど
本当ですか?
335314:02/05/27 13:46
>>330
ネタでもないし、嘘を書いたつもりはない。
間違っているなら、どこがおかしいのかを指摘してもらいたい。
336132人目の素数さん:02/05/27 13:55
>>335
>Nは、P(1)、・・・P(n)のどれで割っても1余るから、合成数ではない。

これは嘘。

(2・3・5・7・11・13) +1 = 30031 = 59・509
>>333
まさにその変換u=x-2y v=2x+yが回転変換(単位的じゃないけどね)
求める式を変換すると双曲線になる。でも、この場合X=x^2 Y=Y^2と
考えれば
X>0,Y>0 X+Y=5のもとでの (-3X+2Y)/5の値の範囲を知る問題
つまり線形計画法を使うことが出来る。
平行移動・回転して放物線、双曲線、楕円等になる場合でも同じやり方
出来ることが多いでしょうね。
>>334
n=0,1はダメでしょう。
だけどn>=2なら本当でしょう多分。
(n+1)^2-n^2=2n+1
n>=2の時
n<p<2nとなるpがある
という有名な定理があったっけ。
339314:02/05/27 14:03
>>336
>>313の文章をよく読め。
「素数が有限と仮定し、それをP(1)、・・・P(n)とする。」
これを前提に議論をしている。

(2・3・5・7・11・13) +1 = 30031 = 59・509
この例で説明すると、素数は2,3,5,7,11,13の全てだとすると、30031
は2,3,5,7,11,13のどの因数も含まないので、合成数ではない、すなわち素数。
よって「素数は2,3,5,7,11,13の全て」という前提に矛盾する。
と、こうなる。




340132人目の素数さん:02/05/27 14:21
>>339
わかった。うそつき呼ばわりしたことは謝る。

ただ、>>314の書き方だと、>>313に誤解を与える可能性が
あるので、最初から>>339のように書いて欲しかった。
341132人目の素数さん:02/05/27 14:25
>>340
だったら君が「うそを言うな」なんていう前に
訂正してあげたらどうだい。
342314:02/05/27 14:31
>>340
了解。以後、気をつける。
>>340
余計ややこしくしてることに気付いてない奴
ですか?
>>343
ごねるなって。
345132人目の素数さん:02/05/27 14:57
不等号の証明の最後でcos(A-B)≦1と書いたとき、「等号成立はA=B」と
書かないといけないんですか?

346132人目の素数さん:02/05/27 14:58
それと、不等号の証明に関して何ですが、A<Bを示すとき、いきなり
(左辺−右辺)>0をしなくても、ある程度、同値変形してから、
(左辺−右辺)>0としても良いのですか?
そのほうがやりやすいですよね?
>>345
A-B=2πだったらどうしますか?

>>346
OK
348132人目の素数さん:02/05/27 15:30
昔の東大入試で6の25乗は何桁かという問題が出て2log6の5までは出来たのですが
ココから先がわかりません。教えてください。
349132人目の素数さん:02/05/27 15:37
>>348
>2log6の5までは出来たのですが
どうしてこんなのが出てくるんだ?
350132人目の素数さん:02/05/27 15:46
まちがってたらごめんなさい。まったくわからないもので。
351132人目の素数さん:02/05/27 15:46
>349
6^25=10^xとおいたのかな?でもこれだと2log6の5がでてこない・・・。
352132人目の素数さん:02/05/27 15:50
もしよろしければ解答例ご教授願います。あと申し訳ありませんが
屋根みたいな記号の意味がわかりません。なにぶん稚拙で申し訳あ
りません。
353132人目の素数さん:02/05/27 15:52
>352
6^25は6の25乗という意味です。
ちょっと考えて見ます。
354132人目の素数さん:02/05/27 15:54
6^25=10^x とおけば、
「xの整数部分」+1
が求める答。

6^25=10^x の両辺の常用対数をとると
 25log_{10}(6) = x
となる。だからあとはlog_{10}(6) を求めればよい。

ところで問題文にlog_{10}(2)やlog_{10}(3) の値は
与えられていないのか?
355132人目の素数さん:02/05/27 15:54
352様》ありがとうございます。
356132人目の素数さん:02/05/27 16:00
354様》昔の(昭和48年ころの参考書)問題で、友人からもらった
プリントには問題しか書いてません。推測ですがそのころの学生はlogの
値を覚えていたと解釈してよいのでしょうか。
357132人目の素数さん:02/05/27 16:03
今時の受験生だって、
log_{10}(2) がおよそ0.3010
log_{10}(3) がおよそ0.4771
だってことくらいは「知ってる」だろ。
しかしそれが問題文に与えられていないなら
答案でその近似値を使うときは
自力で「導く」必要がある。
>>357
知ってるのか?
数学やってる奴は、覚えるの大嫌い、考えるの大好きだと思ってるんだけど。

>>356
実際に計算するんじゃないかな?
(((6^3)^2)2)^2 ×6
0 < log_{10}(6) < 1

10 log _{10} (6) = log_{10} (60466176)

so we can obtain

7 < 10 log _{10} (6) < 8,

this means

0.7 < 10 log _{10} (6) < 0.8 .

360132人目の素数さん:02/05/27 16:12
6の10乗の計算はしたくないなあ。
>359
wonderful!!
362132人目の素数さん:02/05/27 16:13
>>347
A-B=2πだったらどうしますか?
cos360=1ですけど、cos≦1に含まれていると思うのですが、等号成立を
示さなきゃいけないんですか?
363132人目の素数さん:02/05/27 16:15
>>359
( ゚д゚)ポカーン
364132人目の素数さん:02/05/27 16:16
>>332
グラフでは縦軸y横軸tと書いて良いと言われましたが、y=(xの式)が普通だと
思うんですけどy=(tの式)と書いても良いのですか?
365132人目の素数さん:02/05/27 16:21
>>359
それじゃきまらんね。もう一桁さげないと。
すると直接計算と大差ない・・・(ノ゚∀゚)ノ ┫-・'
どうやって求めるべきか考えてる間に
直截もとまりそうなんだけど。
367132人目の素数さん:02/05/27 16:24
和積の公式についてなのですが、
sinA + sinB=2sin(A+B)/2・sin(A-B)/2 の「A-B」って普通A-B>0になる
ようにA>Bの状態で割り振るんだと思いますが、逆も成り立ちますよね?
その成り立つ理由を示してくれませんか。なぜ成り立つのかわかりません。
6^25=(2^25)*(3^25)
=2^16*2^8*(3^8)^3*3*2
=65536*256*6561^3*6
=65536*256*282429536481*6
=28430288029929701376

男ならこうやれ。
>>367
sinA+sinB = 2sin( (A+B)/2 ) ・ cos( (A-B)/2 )

だそうです。

370132人目の素数さん:02/05/27 16:31
>>368
普通に、
log_{10}(2) がおよそ0.30
log_{10}(3) がおよそ0.47
を証明して解けばいいだろ。
371132人目の素数さん:02/05/27 16:31
6^25=(2^25)*(3^25)
=2^16*2^8*(3^8)^3*3*2
=65536*256*6561^3*6

わたしはこの辺で限界だ。
>>368
何で余計にややこしくしてるんだ?
計算間違いする可能性も高くなるし、かえってリスクが高くなる。
↑直接計算する方法のことね。
直接計算の場合
(((6^3)^2)2)^2 ×6
が1番シンプルだと思うんだけど。
376132人目の素数さん:02/05/27 16:37
>>370
そうすると結局
前もってlog_{10}(2),log_{10}(3)の近似値を
ある程度知っておかないとダメってことに。
ダメダ、
やっぱり計算してられないな。
378132人目の素数さん:02/05/27 16:39
ところでこれほんとに東大の入試だったの。
>>378
んなわきゃない
6^25=(((6^3)^2)^2)^2 ×6

(6^3)^2=46656
4×10^4 < (6^3)^2 < 5×10^4

((4×10^4)^2)^2 ×6=1.536×10^19
((5×10^4)^2)^2 ×6=3.75×10^19
6^25=(6^5)^5

6^5=7776
7000 < 6^5 < 8000

7000^5=1.6807 ×10^19
8000^5=3.2768 ×10^19

の方がシンプルでした。
382132人目の素数さん:02/05/27 17:36
それでも6^4 , 7^5 , 8^5 の計算は必要か。
でもまあこれくらいなら
計算する気になる・・・かなぁ?
383132人目の素数さん:02/05/27 18:01
中3です。因数分解してください。
x^4+3x^3-39x^2-47x+210
384132人目の素数さん:02/05/27 18:19
プリーズヘルプミー。
この問題解いて下さい〜!
「次の連立方程式が表す領域を図示し、その面積を求めよ」
x^2+(y-2)^2<=2, y>=x^2+2
図示はわかるので、面積を求めて下さい@@
(ちなみにムシってもいいっす
385383さんへ:02/05/27 18:21
(x-2)(x+3)(x-5)(x+7)
やりかた……x=2を代入すると0になるからx−2で割れる。
こういうのを繰り返せばできる。
33歳のおっちゃんにはしんどい計算でした。
386384:02/05/27 18:24
すみません、「連立方程式」→「連立不等式」ですm(__)m
387132人目の素数さん:02/05/27 18:26
現在高校生ですが大学行ったら何の数学教えてもらえるんですか?
位相幾何学とか興味あるんですけど高校数学の知識とあと何の知識が必要なんですか?
388132人目の素数さん:02/05/27 18:27
あと解析幾何学もお願いします
389中学三年:02/05/27 18:33
数が素数かどうか簡単に求める方式ってありますか?
(千以下の数字)
>>385 ありがとうございました。(m_m)
>>389
1000以下なら、素数表丸暗記しる。
>>384
補助線引いてみ
393154:02/05/27 18:43
>>154はあってますか??
誰か答えてください。お願いします!
394389さんへ:02/05/27 18:44
2000以下の素数表

http://www.geocities.co.jp/Playtown-Dice/7855/prime_numbers.html

1000までだったら168個しかないから覚えようと思えばおぼえられるよ
395中学三年:02/05/27 18:46
ありがとうございます。今から頑張って暗記してみます。
396132人目の素数さん:02/05/27 18:47
>>395>>391にだまされていると思うのは俺だけだろうか。
>>384
2つの交点を直線で結んで、
図形を2分割すると簡単に求まるよ。

上の面積は、円から内接する正方形を引いたのの1/4。
下は放物線を水平に切った面積。(こっちは積分する)
398394:02/05/27 18:57
395さんへ

31までの素数で割り切れるかどうかをためす、という方法もある。
本当は素数より素因数分解しにくい数の方を覚えておいたほうが役立つ。
たとえば221=13×17、481=37×13、667=23×29など

あ、でも素数を覚えておくのは悪くないよ。頭の中で数字遊びができる。
私はそれで数学が好きになりました。
399132人目の素数さん:02/05/27 19:04
154さんへ
あってますよ。
400132人目のカテキョさん:02/05/27 20:00
漏れ「2点A,Bの距離ってのは、線分ABの長さのことだべさ」
生徒「距離って、離れ具合のことでしょ?」
漏れ「そうだべさ」
生徒「線分A,Bが、一番短いの?」
漏れ「勿論」
生徒「ど〜してそ〜いえるのさ〜」
漏れ「そ、それは大人の世界の決め事で、お子様に理解できることではないのさ!」
生徒「ふ〜ん。(疑いの目)」
漏れ「まあ、大人になれば分かるさ(遠い目)」
漏れ(ふっ、ヤレヤレ。どうにか逃げ切ったぜ)

こんな私めに、
「(2次元空間における)2点A,B間の最短ルートは、線分ABである」という
証明法をご教授下さいませ。

ちなみに当方大学の教育学部で理科やってまして、生徒は中学生♂。
そのくらいのレベルでおながいします。

折れ線なら「(三角形の一辺)<(他の二辺の和)」で
何とかできそうですが、曲線を含むともう何が何やら。
テレホタイムにまた来ます。
401某定時校生:02/05/27 20:12
>>400
「点Aを中心として、点Bをとおる円」で説明をされたような記憶が。
402数学ニガテっ子:02/05/27 20:40
教えてください!
問:次の整式を複素数の範囲で因数分解せよ。
【1】x^4−4
【2】x^4+3x^2−4
            …という問題です!よろしくおねがいします! 
403132人目の素数さん:02/05/27 20:42
誰かこの問題の計算過程を教えてください。

1997^1997を9で割ったときの余りを求めよ。

とりあえず(1998-1)^1997にして、二項定理を利用するそうです。
答えは8になるとのこと。
お願いします。
404132人目の素数さん:02/05/27 20:48
>403
1998は9で割り切れる
>>403
(-1)^1997 = -1
406132人目の素数さん:02/05/27 20:51
>>404
1998=9*222だからですよね。
それは分かってるんですけど・・・。
あまりの求め方が分かんないんですよね。
>>402
x^2=Aとおいてみてください。
x^2=(x^2)^2=A^2
となります。
そうすると各問いはAの2次方程式になります。
それを因数分解したあとAをx^2に置き換えてください。
408132人目の素数さん:02/05/27 20:55
和積の公式についてなのですが、
sinA+sinB = 2sin( (A+B)/2 ) ・ cos( (A-B)/2 )
の「A-B」って普通A-B>0になる
ようにA>Bの状態で割り振るんだと思いますが、逆も成り立ちますよね?
その成り立つ理由を示してくれませんか。なぜ成り立つのかわかりません。
>>406
1998=x
とおくと
(1998-1)^1997=(x-1)^1997
これを展開すると・・。
あとは>>405さんの言うとおり。
>>408
cos( (A-B)/2 )=cos( (B-A)/2 )
だから。

411132人目の素数さん:02/05/27 20:57
グラフでは縦軸y横軸tと書いて良いと言われましたが、y=(xの式)が普通だと
思うんですけどy=(tの式)と書いても良いのですか?
>>408
cos(A-B) = cos(B-A)
>>411
いいです。
414132人目の素数さん:02/05/27 21:01
記述問題なんですが、どうすれば満点な答えがでるのでしょうか?

(1)xに関する2次方程式x^2-(m-7)x+m=0 の解がともに正の整数であるとき、定数mの値を求めよ。

(2)ax^2-bx+c<0の解が1<x<3であり、ax^2-bx+c<6x-14 の解が 2<x<5 であるとき、a,b,cの値を求めよ。


回答(1)m=15,18   (2)a=2 b=8 c=6

ですが、満点の記述回答とはどんな感じになるのですか?

教えてください。
415132人目の素数さん:02/05/27 21:02
>>337
>まさにその変換u=x-2y v=2x+yが回転変換(単位的じゃないけどね)
求める式を変換すると双曲線になる。でも、この場合X=x^2 Y=Y^2と
考えれば

すいません、回転変換ってなんですか?
それと、双曲線もよくわかりません。
1A2Bの範囲で解けないのですか?
416132人目の素数さん:02/05/27 21:03
>>412
こんにちは。それは加法定理からですか?
417132人目の素数さん:02/05/27 21:03
>406
(1998−1)^nの一般項はnCr1998^r*(−1)^(n-r)
と表せる。1998がある部分は9で割り切れるから、問題の余りは
r=0のときだけ。つまり(−1)^n=−1 (nが奇数だから)
−1≡8 (mod9)
418132人目の素数さん:02/05/27 21:05
>>413
お返事どうもっす!!すいません、普通はf(t)と書き直すのですか?
y=(tの式)は見たことがないので、普通の書き方はどうなるんですか。
>>416
y=cosx
のグラフを考えてみましょう。
あまり公式に縛られずに、イメージすることも大事ですよ。
420132人目の素数さん:02/05/27 21:07
>416
cos(-θ)=cosθ
加法定理からでも言えるけど
421132人目の素数さん:02/05/27 21:08
>>419
グラフですか。私cos(A-B)のグラフは書けません。Aが180みたいなやつだと
わかるのですが。どう書くんですか?
422403=406:02/05/27 21:08
>>417
すいません、
最後の1行がよく分からないです。
modってなんですか?
>>418
y=f(t)
でおかしくないですよ。
ただ
y=f(x)
のxをtを使った式で置き換えたときなどは
g(t)などとするべきです。
424132人目の素数さん:02/05/27 21:09
>>420
それって加法定理ですか???cos(A-B)の形じゃないですよね。
425132人目の素数さん:02/05/27 21:11
記述問題なんですが、どうすれば満点な答えがでるのでしょうか?

(1)xに関する2次方程式x^2-(m-7)x+m=0 の解がともに正の整数であるとき、定数mの値を求めよ。

(2)ax^2-bx+c<0の解が1<x<3であり、ax^2-bx+c<6x-14 の解が 2<x<5 であるとき、a,b,cの値を求めよ。


回答(1)m=15,18   (2)a=2 b=8 c=6

ですが、満点の記述回答とはどんな感じになるのですか?

教えてください。


>>424
>加法定理「からでも」言えるけど

>>421
y=cosx
を書いてみましょう。
cosx=cos(-x)
がわかると思います。
x=A-Bを考えてみましょう。
>>425
満点といわれると自信がないです。
スマソ。
もう少し待ってみては?

(めんどくさいし)
428132人目の素数さん:02/05/27 21:13
>422
mod9 ってのは9で割ったときの余りを考える、見たいな意味
≡(合同)は同じ仲間になるよってな意味
例 19≡10≡1 (mod9)
>>425
あなたの解答をまず書いて、添削してもらった方が良さそうです。
430132人目の素数さん:02/05/27 21:15
>>423
御返事有り難うございます!

>y=f(t)でおかしくないですよ。

これってはじめから、y=f(t)だったらOKということですか?でも
はしめからy=f(t)なんてないですよね?たいてい初めはy=f(x)
で始まると思うし・・・。
>>425
そういう要求をするなら、
まずはチミの答えを書くんだ。
432132人目の素数さん:02/05/27 21:19
>>426
グラフ書いてみるとわかりました!!なるほど。ところで、
>x=A-Bを考えてみましょう。
あぁそういうことですか。A-Bをひとかたまりとみたらいいんですね。
どうも助かりました。
433132人目の素数さん:02/05/27 21:20
(1)2つの解をα、β(α≦β)とすると、α+β=m−7、αβ=m

これからmを消去して(αー1)(βー1)=8


ここまでです・・・・・。以降謎・・・・・



434403=406:02/05/27 21:24
>>428
ありがとうございます、よく分かりました。
でも、ちょっとあっさり過ぎて心配なので、
別解で答え8は求められませんか?
>>430
例えば、ある運動してる物体の、t秒後の原点からの距離をyとします。
yはtの関数で表すことが出来ますよね。

y=f(t)
おかしくないです。
436132人目の素数さん:02/05/27 21:32
もっと簡単に
1997^1997≡(−1)^1997≡-1≡8 (mod9)
数論ではよく使う。ほとんど = と同じ感覚で使えるのがいい。
ただ入試なんかで使うと、ほめてくれるか、バツにされるか採点者次第だろうね
>>434
じゃあ別解。

(x^1997)+1 は、x+1 を因数に持つ。
従って(1997^1997)+1 は1997+1で割り切れる。
だから9でも割り切れる。

本質的には同じだけど。
>>437
すばらしい
439403=406:02/05/27 21:38
<<437
やっと満足することができました、どうもスイマセンでしたm(__)m
<<436さんもありがとうございましたm(__)m
440132人目の素数さん:02/05/27 21:43
1÷100=100
これに1本、直線を加えて等式を成立させよ

例)5+5+5=550→545+5=550
>>440
1÷100=1%
>>440
いいともでやってたね。
443132人目の素数さん:02/05/27 22:01
>>435
t秒後ですか・・・確かにおかしくないですね。物理っぽい問題ですが。
どうも有り難うございました!!!!
444132人目の素数さん:02/05/27 22:02
aとbの方程式をグラフで書くとき、縦軸をb,横軸をaにすると思うんですが、
これは慣習ですか?横軸をb、縦軸をaとしちゃったらおかしいですよね?
これはそうしなきゃいけないレベルの話ですか?
445132人目の素数さん:02/05/27 22:05
>>444
そうする必要はありません。
446132人目の素数さん:02/05/27 22:33
四面体OABCについて、↑a=↑OA、↑b=↑OB、↑c=↑OCとする。
辺BCの中点をD、辺ACを2:1の比に内分する点をEとする。
(1)↑OD、↑OEを↑a、↑b、↑cを用いて表せ。
(2)ADとBEの交点をFとする。↑OFを↑a、↑b、↑cを用いて表せ。
(3)ΔOBCの重心をGとする。線分AGを6:1の比に内分する点をHとするとき、
3点O、H、Fは同じ直線上にあることを示せ。


(1)は判るんですが(2)がわかりません。
(2)が判れば(3)は解けるんだろうけど・・・。
親切な方、どうか教えてくださいm(__)m
>440
1÷1%=100

>446
直線AD, BEをそれぞれパラメータ表示。
Fは直線AD上の点であり、しかも
直線BE上の点である。
448132人目の素数さん:02/05/27 23:02
次の関数の導関数を求めよ。
f(x)=x√(x^2+a)+alog{x+√(x^2+a)}

いくらやってもできなかったので質問いたします。
どうかお願いいたします。
449132人目の素数さん:02/05/27 23:04
>>445
すいません、それじゃあなぜ、bが横軸、aが縦軸のグラフは存在
しないのですか?一回も目にしたことはありませんが。
450132人目の素数さん:02/05/27 23:06
>>448答えが聞きたいのか過程が聞きたいのかどっちだ
451132人目の素数さん:02/05/27 23:08
>>449存在するよ。自分で紙に書けばいいじゃん。挑戦的な奴だな。
>>400 「ABを結ぶ最短距離が直線であること」を子供にわかる
ように説明しろだと? あほか、ネタでなければ、ただ子供に
なめられてるだけだ。もしこれが実話なら、数学的に厳密に説
明したて理解されるわけでもなかろう。

ネズミでもハエでも、エサをみつければまっすぐ飛んでいくぞ。
こんなことは動物的直感で、それを理屈で証明するほうがひね
くれているというものだ。もし子供が理解していないとすれば、
困難は原理的なものではなく、線分ABという抽象的表現がエサ
をめざして走っていく自分と重ならないだけだ。
453にげっとずざ:02/05/27 23:11
>>449
別にいいけど、試験でやると見間違えられるから諸刃の剣。
あとで、
        ∧_∧∩      / ̄ ̄ ̄ ̄ ̄ ̄ ̄
       ( ´∀`)/    <先生!採点まちがってます!  
  __ / /   /       | 
  \ ⊂ノ ̄ ̄ ̄ ̄ ̄\   \_______
  ||\           \
  ||\|| ̄ ̄ ̄ ̄ ̄ ̄ ̄||
  ||  || ̄ ̄ ̄ ̄ ̄ ̄ ̄||
     .||              ||

とかやるのもUzeeeeeeし。
454132人目のカテキョさん:02/05/27 23:16
>>400です。

>>401の某定時校生さんに返信頂きましたが、
詳しくお教え願えますか。

また、他の方々にも、返信して頂ければ嬉しいのですが。
455132人目の素数さん:02/05/27 23:18
>>451
>自分で紙に書けばいいじゃん。
そういう話じゃなくて・・・。

>>453
別に決まっていないんですか。じゃあ何でみんなそうするんですか?
453さんが縦軸をb横軸をaとしたのはなぜですか?また、なぜ逆で
かかなかったのでしょうか?しつこくてすいません。
>>455
abc・・・xyz
の順番に合わせてるだけだと思います。
bの値の変化に伴ってaの値が変化するなら
aが縦軸の方がいいと思います。
457にげっとずざ:02/05/27 23:25
>>455
>別に決まっていないんですか。じゃあ何でみんなそうするんですか?
さあ?
いいと思えば別に逆でもいいんでない?

>453さんが縦軸をb横軸をaとしたのはなぜですか?
漏れ程度には常識を備えた人だったから。

>また、なぜ逆で
>かかなかったのでしょうか
>>453がへそ曲がりじゃなかたから。

>しつこくてすいません。
わかっているならいいです。
458132人目の素数さん:02/05/27 23:28
>>456
初めて納得できました!!そういうことだったのですか。
>bの値の変化に伴ってaの値が変化するならaが縦軸の方がいいと思います。
そうですね、ごくたまにそういう問題もありますね。
とても役立ちました!!!
>455
じゃあ何でみんなそうするんですか?

自分でも最初に書いたとおり、
『慣習だから』
が答え。

当然、b軸を横にa軸を縦に書いた方が
見やすければそうするべきだが、それならそれ以前に
aとbを逆に取った方がいいわけで・・・

>>455
曲線が入るとややこしいということですが、曲線を短い線分の折れ線
と近似して説明するのではダメですか。
461132人目の素数さん:02/05/27 23:29
>400
ピンと糸を使って実験したらどう?
462???:02/05/27 23:31
記述問題なんですが、どうすれば問題なしの答えがでるのでしょうか?

(1)xに関する2次方程式x^2-(m-7)x+m=0 の解がともに正の整数であるとき、定数mの値を求めよ。

(2)ax^2-bx+c<0の解が1<x<3であり、ax^2-bx+c<6x-14 の解が 2<x<5 であるとき、a,b,cの値を求めよ。


>400
曲線を折れ線で近似して三角形の論法を使えば?
464にげっとずざ:02/05/27 23:35
>>461
でも、「ピンと糸を張ったもの」と「最短経路」を同じモノとみなせるの?
というか、それについてリア厨も納得な説明をすることができる?
ヴァカな漏れには辛いな…
466132人目の素数さん:02/05/27 23:36
>>459
お返事どうもっす。
>自分でも最初に書いたとおり、『慣習だから』が答え。
やはりそうですか!!だいたいどの問題でもb軸を縦に、a軸を横にとること
を要求しているし、逆にすれば、2次間数や3次間数の形が変わってきて
見にくくなりますよね。誠意あるお返事どうもありがとうございました。
467にげっとずざ:02/05/27 23:37
>>462
問題なしの答え???
468???:02/05/27 23:37
433は自分で解いたところまでです。



記述問題なんですが、どうすれば満点な答えがでるのでしょうか?

(1)xに関する2次方程式x^2-(m-7)x+m=0 の解がともに正の整数であるとき、定数mの値を求めよ。

(2)ax^2-bx+c<0の解が1<x<3であり、ax^2-bx+c<6x-14 の解が 2<x<5 であるとき、a,b,cの値を求めよ。


469132人目の素数さん:02/05/27 23:38
>>462
『問題なしの答え』とはどういう意味だ?

とりあえず、問題の答えは
(1)
x=(1/2) * (m-7 ± √(m^2-18m+49))
っていう形になるのでとりあえず、両方の値が正って言うことでmの条件を
定めて、そしたらその範囲で√(m^2-18m+49)が整数になるものを求めればよい。

(2)
前半部分から
ax^2-bx+c=a(x-1)(x-3)
a>0
が分かるから、後半部分も同じようにすればよい。

とゆーこと
470???:02/05/27 23:40
問題なし=減点なしってことです・・
471質問です:02/05/27 23:42
  鋭角三角形ABCにおいて、AC=1, ∠ABC=90°, ∠BCA=θのとき、
(1) θのとりうる値の範囲を求めよ.
(2)以下略

という問題なのですが、(1)がわかりません。
鈍角三角形の性質はどのようなものがあるのでしょうか。
よろしくおねがいします。
472132人目の素数さん:02/05/27 23:45
>>469
(2)は整数解とは限らないみたいだけど(w
473471:02/05/27 23:46
すみません、訂正です。
一行目 (誤) 鋭角三角形→(正) 鈍角三角形
474132人目の素数さん:02/05/27 23:46
>>471
>鋭角三角形ABCにおいて、AC=1, ∠ABC=90°, ∠BCA=θ・・・
問題文ちゃんと書き写してますか?これって直角三角形じゃん。
475132人目のカテキョさん:02/05/27 23:47
>>460>>461
レスありがとうございます。

>>461さんの、いただきです。
中学生相手としてはベストだと思います。
厳密な証明ではないけれど、それで十分ですね。

では、厳密な証明方法を考えてみたいと思います。
厳密な証明をしたい!という時は>>460さんの方法でしょうが、
「曲線>折れ線>線分」 を言うわけですか?
曲線を折れ線で近似すると、折れ線はそれぞれが線分で、
またその線分と曲線のどちらが長いか…という問題が
ずっと続くってことはないですか? 数学苦手ですまそ。
476132人目の素数さん:02/05/27 23:47
>>462
(1)は、変数を分けて
y=x^2+7x
y=m(x-1)
の交点を考えることによりm>9を出しておいて
あとは√(m^2-18m+49)が整数条件
ただしm:整数(証明略)
477471:02/05/27 23:47
さらに失礼しました。
一行目 鈍角三角形ABCにおいて、AC=1, ∠ABC=30°, ∠BCA=θ
よろしくおねがいします
>>471
0 <θ< 90°
479132人目の素数さん:02/05/27 23:49
>>471

∠ABC=90°なら「鋭角三角形」じゃないだろう。
480132人目の素数さん:02/05/27 23:49
>>471
そもそも、直角三角形は鋭角三角形なのか?
もう覚えていない・・・

まー、どっちにしても直径をACとする半円を考えて、その半円の周上に点Bが
あるようなモデルを考えると、三角形ABCがどんな図形になるのかが考えやすくなると思う。
481132人目の素数さん:02/05/27 23:50
lim_[n→∞][n]√(n)の極限を求めよ。

挟み込みを使うようなんですがいまいちわかりませぬ。
どうかよろしくお願いします。
>>477
90°を超える角があるのが鈍角3角形
483132人目の素数さん:02/05/27 23:52
>>477
60°<θ<90°
じゃない?
484132人目の素数さん:02/05/27 23:53
θ+∠A=150
このうちどちらかが、90より大きい必要がある。
i)θが鈍角のとき、90<θ<150
ii)θが鈍角でないとき、90<∠A<150だから、0<θ<60
485483:02/05/27 23:53
すまん 鋭角と勘違いした。この答えは無し
486132人目の素数さん:02/05/28 00:01
0<θ<90の範囲で、cos4θ・cos2θ>0が成立するθに範囲を求める問題なのですが、
cos4θ>0かつcos2θ>0のときと、cos4θ<0かつcos2θ<0とすると、本の解答と
違ってくるのですが、どうしてダメなのでしょうか?解答では、cos4θを2倍角
を使って、2(cos2θ)^2 - 1と書き直して、cos2θで統一して、2(cos2θ)^2 - 1
を因数分解して、最終的には、2(cos2θ- 1/√2)(cos2θ -1/√2)cos2θ>0と
なって、3次関数のグラフから0<θ <22.5, 45<θ <67.5となっていました。

cos4θとcos2θは独立な関係にないからですか?でも4θと2θは別個のものですよね?
487132人目の素数さん:02/05/28 00:02
>462
上の方でも書かれていたが、まず自分の答えを書いて添削してもらう、
それか単に教えて下さい、だけのほうが
完璧な解答を、といわれてもなあ。

一応やって三日
(1)2つの解をa,bとおくと、解と係数の関係から
a+b=m−7,ab=m
2つの式からmを消去すると
a+b=ab−7
ab−a−b=7
(a−1)(b−1)=8
a,bは正の整数だから、a−1,b−1も正の整数で(0でないのは明らか)
a<=bとしても一般性を失わないから
a−1=1,b−1=8 またはa−1=2,b−1=4
これを解いてa=2,b=9 またはa=3,b=5
m=abより m=18 またはm=15

α,βのほうがそれらしいかも知れないが、掲示板では面倒

(2)1<x<3を解とする2次不等式は
   a(x−1)(x−3)<0 ただしa>0
この左辺の展開式をax^2-bx+c と係数比較してb=4a,c=3a
またax^2-bx+c<6x-14 よりax^2-(b+6)x+c+14<0
2<x<5を解に持つ2次不等式
a(x−2)(x−5)<0 の展開式と比較して
b+6=7a,c+14=10a
これらの関係式を解いてa=2,b=8,c=6
これはすべての条件式を満たしている
488452:02/05/28 00:05
>>475 何度でもいうけど、そもそもこんなこと聞かれるほうが
子供になめられとるんだ。それをうまく説明すればわかっても
らえるだろう、なんて考えるのが教育学部の悪弊だ。わかる
やつは教えなくてもわかる。わからんやつは教えてもわからん。
中間はないんだ。

厳密な証明? どこまで厳密かしらんが、√(1+y'^2)を
オイラー方程式で解くといい。
>>475
正確には、平面上の2点が函数 f(x) 上にあるとして、
f をいろいろ変えて(a,f(a))と(b,f(b))の距離

L = ∫[a,b]√(1+(f')^2)dx

が最短になるfを考える。

この手の問題には変分原理の方法がそっくり適用できる。
詳しくは省略するが、結論はLを最小にするようなfは
微分方程式 f '' (x) = 0 を満たしていることがわかる。
>486
ダメなわけがない。

単に、貴方が
cos4θ>0かつcos2θ>0 または cos4θ<0かつcos2θ<0
という条件から答えを導くまでの過程で
計算間違いをしてるだけだと思われ。
491471:02/05/28 00:06
ありがとうございました
>>488
しつこい。
493489:02/05/28 00:14
488の下2行とかぶった。

>>488の言い分は、やや言葉は乱暴だが、うなずける。
この手の質問をする子供の意図は、そのほとんどが、
大人を困らせること。

何を答えても、ニヤつきながら「それはなぜ?」と
永遠に聞き返してくることだろう。
494132人目のカテキョさん:02/05/28 00:19
>>400です。自力でやってみました。
「線分AB<折れ線AB≒曲線AB」を目指しました。添削キボン。


2点A,Bに対し、「A,Bを両端とする曲線AB」と「線分AB」がある。

ここで曲線AB上に点x0=A,x1,x2,x3,…,xn=Bをとり、
x0からxnまで順に線分で繋いでいく。これを「折れ線AB」とする。

すると、点x0,x1,…,xnの数を増やしまくれば、
折れ線ABの長さは曲線ABの長さに近づくと思われる。

ところで「線分AB<折れ線AB」(∵三角形の一辺<他の二辺の和)だから
線分AB<曲線AB

これでいいですか?

>>488
まあ、教えなくても分かるってのは自力で調べてってことでしょうし、
その「調べる」の手段として家庭教師に聞くってのがあってもいいかと。
あと、自分でも気になったもので。
昔好きだったのに、理科ばっかで数学あまりしてないな…と。
オイラー…調べてみます。
>>489
すまんがもう少し詳しく解きほぐしておくれ。
変分原理と、その適用について。
URLへのvectorでもいいから。
>>494
ダメダメです。
497488:02/05/28 00:38
>>494 言葉の悪かったのは謝るが、子供(オレの場合は大学生)
に教えてて、つくづく嫌になった。わかるやつはわかる。わから
んやつはわからん。わかるヤツは、「自分で調べてわかる」の
じゃない。教えなくても、調べなくても、道すじさえ示唆すれば
勝手にわかっちゃうんだ。

このごろ思っているのは、教育とは、教えることじゃない。教え
ないことだ。勝手に悟らせることだ。「あの先生、教えてくれな
いから、ちょっと考えてみた。そしたら、わかっちゃった」と
なれば理想的だと思う。

変分法は、ちょっとここには書ききれないだろう。「変分法」
「変分原理」「オイラーの微分方程式」などで検索してみる
といい。数学より、力学の教科書にくわしいだろう。それほ
ど難しいことではない。2点を結ぶ最短経路が直線である、周囲
一定で最大面積の図形は円である、などのあたりまえのことが、
形式的に出てくるところに、すごさがある。
>>497
大学の先生ですか。
私のようなのがお手を煩わせているのだと思います。
検索の仕方まで教えて頂いて。カタジケナイ
義務教育と大学教育では違う気もしたり・・・
考え方は違えど、より良い教育を目指しているということで。
ってのはダメですか?だめだろな。
>497-498
二点を(0,0), (p,0) (p>0)
とし、曲線を
(x,y)=(f(t),g(t)), f(0)=0, g(0)=0, f(T)=p, g(T)=0
とパラメータ表示すれば

L = ∫[0,T]√((f')^2+(g')^2)dt
が線分のときに最小になるというのは、
変分法使わなくてもほぼ高校範囲で
出てきますよ。念のため。
>499
補足。
要するに

∫[0,T]√((f')^2+(g')^2)dt
≧∫[0,T]√((f')^2)dt
=∫[0,T] | f' |dt
≧∫[0,T] f' dt
= p

です。
>>499
なんか簡単そうなのがキタ―――!
おおおっ!ちょっとやってみます。
考えるのに時間がかかるのはご愛嬌w
>>500 の方法は、(0,0)→(p,0)を結ぶ曲線のうち、最短の
ものでも長さは p以上と言っているのであって、曲線の形
を決めるものではないような。それが直線であり、それに
限るという証明は別途しなければならないような。
>>502
そのとおり。
504499:02/05/28 01:43
>502
そういうこと。
もちろん、線分のときが長さpなのは一瞬で出てくるので、
問題は『それに限る』の部分だけ。

『それに限る』を示すには、
次の補題
「a<b, 区間 [a,b]で 関数f,gは連続で、f(x)≦g(x) であり、
区間 [a,b]内のある一点 c でf(c)<g(c) とする。
このとき、∫[a,b]f(x)dx < ∫[a,b]g(x)dx が成立」
を使うのが典型的なやり方ですが、ここは
高校範囲じゃないわけで。

まぁともかく、高校範囲でも、線分が最短となることだけは
示されているわけです。
(他に最短があるかどうかは別として)
505132人目の素数さん:02/05/28 01:43
>>502
L=p となる曲線は明らかに存在する。(直線)

一方、500で等号が成立するためには、
少なくとも g'(t)≡0 である必要がある。

それはx軸に平行な直線である。
507にげっとずざ:02/05/28 01:46
>505
RAIDネタのこと?
>>505
719で答え出てんじゃん。
1台の時に対する故障倍率は
1と3の間の値を取る。
509320:02/05/28 02:09
>>506 それでもまだ x軸上を行きつ戻りつする「曲線」
は排除できないから…
>>509
500の、

∫[0,T] | f' |dt ≧ ∫[0,T] f' dt

の部分で等号が成立するためには、
f' が常に0以上でなければならない。

つまりx座標が減少する曲線は、
L=p を満たさない。
>>510 たいへんよくわかりました。どうもありがとう。
つまり、2点0-(p,0)を結ぶ曲線は長さ p が最低で、
直線の場合それを与え、それに限るというわけだ。

ところで、2点の「距離」として pより小さいものは
ない、という証明は、どうやるの?これは公理?
512132人目の素数さん:02/05/28 02:35
>>511
2点(a,b) (c,d)間の「距離」を

√{(a-c)^2 + (b-d)^2}

で定める、というのは、ユークリッド平面の
定義の一部になっているはずだよ。確か。
513471:02/05/28 02:37
できました。ありがとうございました。
>512
いや、そう言ったら身も蓋もない。

>511
変分で求めたのが p なのだから
(本当は多少微妙だが)長さの「最低値」は p なのでは?
まだ「証明」が必要?
515512:02/05/28 02:43
>>514
でも、>>499

L = ∫[0,T]√((f')^2+(g')^2)dt

を出す過程で、すでにユークリッド距離を
使ってると思うんだけど。
516511:02/05/28 02:45
>>514 変分法のずるいのは、これで求まるのは積分値に
停留点を与える曲線、というだけで、極大とも極小とも
(ましてや最大とも最小とも)言っていないんだ。最大
最小の確認が必要なら、おそらくここで議論したような
話が復活すると思う。
517514:02/05/28 02:45
>512
シッ。言っちゃダメ。
518512:02/05/28 02:53
(・*・)……キョクセンノ ナガサッテ ナンナノカ
       ヨクワカラナク ナッテキチャータ
>511
極値が一つであること、
無限遠をとおってくれば明らかに極値での値よりも
大きくなることを示せばよいのでは?
>512
たいてい空間には計量という2階テンソルがある。
それをつかって、曲線について線積分をとったものが
長さ。
だから計量によるんだなぁ。
ユークリッド空間の場合は、>512 でよい。
521512:02/05/28 03:05
(・o・)
そうか、距離と長さを混同してたからいけなかったんだ。

2点間の距離がはじめからpと決まっていたとしても、
その2点を結ぶ、長さpの連続曲線があるとは限らないんだよね。

ましてやそれが直線であることなんて、全然わからないわけだ。
>>520サンクス
522132人目の素数さん:02/05/28 03:11

















( ゚д゚)
523272:02/05/28 05:20
もう見てないかも知れないが>>269
後の人が解いてくれたみたいだね。
僕が考えていた方法とは少し違うみたいだが。
nが偶数の時は、z^k=-1となる元の存在を考えなくてはならず、
問題の関数はすべてのz^n=1を満たす複素数に対しては定義されない。
nが奇数(特に奇素数)の場合はそういうことが無い。
1/(Z^k+1)がZ^kの多項式と実は等しいことが判ればあの問題は解けたも
同然だというのはわかるよね。(本質的に等比級数の和を組み合わせた
ものだから)
補足としてこれを示しておこう。
W=Z^kとおく
S=1-W+W^2-W^3+...+W^(n-1)とおく(n-1は偶数であることに注意)
wS=W-W^2-W^4+...+W^n
W^n=1だからwS=-S+2が成り立つ。
(1+w)S=2よりS=2/(1+w)
即ち1/(1+W)=S/2
(1/z^2k)(1/(1+z^k))=1/2(1/w^2)(1-W+W^2-W^3+...+W^(n-1))
=1/2(1/w^2-1/w+1-w+w^2+...+w^(n-3))
w=z^kに戻して
もう一度和を取れば、結局(n-1)/2になる筈。
272では大体こういう解答を導こうと思ってたんですが。
もっと簡単な方法があったようですね。
応用問題として暇な時に
z^n=1
Σ[k=1,n-1]1/(i(z^2k)+z^3k)当たりでもチャレンジしてみてはどうでしょうか
こちらもすべてのn,zに対して定義されるとは限りませんが。
なお、僭越ながら忠告しますが、「文系だから...、苦手」とか言うのは
文系の人に失礼な面があります。論理的に言って文系は数学が苦手という命題は正しく
ありません。
また、高校生の段階で、文系・理系という言葉を単に大学の志望学部の系統の大雑把な
分類という意味以外に拡張して使うことは好ましくないと思います。
524132人目の素数さん:02/05/28 05:56
>>523
>なお、僭越ながら忠告しますが、「文系だから...、苦手」とか言うのは
>文系の人に失礼な面があります。論理的に言って文系は数学が苦手という命題は正しく
>ありません。
>また、高校生の段階で、文系・理系という言葉を単に大学の志望学部の系統の大雑把な
>分類という意味以外に拡張して使うことは好ましくないと思います。

数学が苦手だと文系に行ってしまうのは最近の傾向でもあるのだし、
そう突っかかっても仕方あるまい
まぁ、最近は理系でも全然という人が増えてきてはいるが
文系程ではない
525言外に見え隠れする:02/05/28 06:12
[[えっと初心者なんでわかりやすく丁寧に教えやがれゴルァァァ]]
526132人目の素数さん:02/05/28 06:19
文系
527132人目の素数さん:02/05/28 07:06
>>312(別解)
(x^2 + 4xy - 2y^2)/(x^2 + y^2) = k
を満たす(x,y)≠(0,0)が存在する為の k の条件をもとめればいい。

(分母を払って)

x^2 + 4xy - 2y^2 = k(x^2 + y^2)
を満たす(x,y)≠(0,0)が存在する為の k の条件をもとめればいい。

(両辺を x^2 でわって)

1 + 4t - 2t^2 = k(1 + t^2)
を満たす t が存在する為の k の条件をもとめればいい。

tの方程式
(k+2)t^2 - 4t + k-1 = 0
が実数解をもつ為の条件が求める答。

↑穴だらけの「解答」なんで細部は自分でなんとかして。


>>415
>1A2Bの範囲で解けないのですか?

一般に、使う道具を制限すればするほど解法は技巧的で
わかりにくいものになるとおもう。
・・・っていうか、「1A2Bの範囲」って知らんのよね。
528132人目の素数さん:02/05/28 09:03
xyz>0(x>y>z)をとくと、x>0,y>0,z>0 またはx>0,y<0,z<0ですよね。
でもxxx>0をとくと、x>0になり、上の答えと違ってくるんですが、なぜですか?
上と下との違いは何ですか?基本的なことだけど、ずっと悩んでました。
よろしくお願いします。
529132人目の素数さん:02/05/28 09:14
>528
>xyz>0(x>y>z)をとくと

↑の式でyとzをxに入れ替えると、
xxx>0(x>x>x)となってしまう。
そもそも比較に意味はないような気が。
530132人目の素数さん:02/05/28 09:16
>>529
そうですか。私が聞きたいのは、2つでも3つでもいいのですが、
積の状態があったときに、それが同じ文字の場合と、違う文字の場合では
なぜ答えが違ってくるのかと言うことです!!!
これはなぜですか?
文字が違うから。
不等号を≧に直せば意味あるかも。
533132人目の素数さん:02/05/28 09:25
(前略)つぶれてしまうからです(後略)
534132人目の素数さん:02/05/28 09:34
>>532
そうですか。ならこうします。
xyz>0(x≧y≧z)をとくと、x>0,y>0,z>0 またはx>0,y<0,z<0ですよね。
でもxxx>0をとくと、x>0になり、上の答えと違ってくるんですが、なぜですか?

>>532
ちゃんと説明してください。
xyz≧0(x≧y≧z)をとくと、x≧0,y≧0,z≧0 またはx≧0,y≦0,z≦0ですよね。
でもxxx≧0をとくと、x≧0になり、上の答えと違ってくるんですが、なぜですか?
536132人目の素数さん:02/05/28 09:52
>xyz>0(x≧y≧z)をとくと、x>0,y>0,z>0 またはx>0,y<0,z<0ですよね。
>でもxxx>0をとくと、x>0になり、上の答えと違ってくるんですが、なぜですか?

なにが疑問なのかよくわからんが、
xxx>0の場合はxyz>0のx=y=zの場合なので、x>0,y<0,z<0にはならないので、
答えは違ってないと思うが。
537132人目の素数さん:02/05/28 10:00
>>536
>なにが疑問なのかよくわからんが
あれほど、書いたのに・・・。
答えが違ってくるのは、3つの文字が独立か独立でないかのちがいですか?
>>535
結果は違っても過程は同じです。

xyz≧0(x≧y≧z)をとくと、x≧0,y≧0,z≧0 またはx≧0,y≦0,z≦0ですよね。
       ↓       ↓       ↓       ↓
xxx≧0(x≧x≧x)をとくと、x≧0,x≧0,x≧0 またはx≧0,x≦0,x≦0


x≧0,y≧0,z≧0という3つの条件が
x≧0,x≧0,x≧0となってx≧0という条件1つに重なった。

x≧0,y≦0,z≦0という3つの条件が
x≧0,x≦0,x≦0となってx=0につぶれてしまった。

    x≧0,x≧0,x≧0 またはx≧0,x≦0,x≦0
 ⇔ x≧0またはx=0
 ⇔ x≧0
違って見えるだけで同じなんだよ。

x>0,y>0,z>0・・・(A)
x>0,y<0,z<0・・・(B)

(A)または(B)が答えなわけだ。

この問題では(A)かつ(B)を満たすx,y,zが存在しないから
x=y=zという特殊な場合には
(A)と(B)のどちらか一方だけを満たす。それが(A)だ。
それだけ。
540132人目の素数さん:02/05/28 10:18
あれほど、書いたのに・・・。
あれほど、書いたのに・・・。
あれほど、書いたのに・・・。
あれほど、書いたのに・・・。
あれほど、書いたのに・・・。
あれほど、書いたのに・・・。
あれほど、書いたのに・・・。
あれほど、書いたのに・・・。
あれほど、書いたのに・・・。
あれほど、書いたのに・・・。
あれほど、書いたのに・・・。
あれほど、書いたのに・・・。
あれほど、書いたのに・・・。
あれほど、書いたのに・・・。
あれほど、書いたのに・・・。
あれほど、書いたのに・・・。
あれほど、書いたのに・・・。
あれほど、書いたのに・・・。
あれほど、書いたのに・・・。
あれほど、書いたのに・・・。
あれほど、書いたのに・・・。
あれほど、書いたのに・・・。
あれほど、書いたのに・・・。
あれほど、書いたのに・・・。
あれほど、書いたのに・・・。
あれほど、書いたのに・・・。
あれほど、書いたのに・・・。


ハァ??俺様にわかるよーに出直してこいと逆ギレに271828ペリカ
わかったけどもう来ねーYO!!と逆ギレに314159ペリカ
542132人目の素数さん:02/05/28 10:46
>>538>>539
お返事どうも有り難うございます!大変よくわかりました。
そこで、ちょっと考えたんですけど、(x+3)(x-3)<0はグラフから解くのに、
(x-3)(y+3)<0は538のようにx-3<0,y+3>0 または x-3>0,y+3<0のように
解きますよね?このように解き方が違うのはなぜなんですか?
543132人目の素数さん:02/05/28 10:52
同じ
544132人目の素数さん:02/05/28 11:36
>>542をめぐる一連のやりとりに笑わせてもらいました。
545132人目の素数さん:02/05/28 11:39
顔を洗って出直してきたまへ
546542:02/05/28 11:46
cos4θ・cos2θ>0はタイプ1かタイプ2のどちらに属するのですか?
どちらのタイプも同じと言うことはわかりましたけど、解き方は歴然と
違いますよね?
547132人目の素数さん:02/05/28 11:50
>542
何がいいたいのか、よくわからない。タイプ1とは?
548132人目の素数さん:02/05/28 11:53
549132人目の素数さん:02/05/28 11:56
>>542
とりあえず、>>544-545の意味を考えることをおすすめする。

んで、>>542の解答。

>、(x+3)(x-3)<0はグラフから解くのに、
>(x-3)(y+3)<0は538のようにx-3<0,y+3>0 または x-3>0,y+3<0のように
>解きますよね?

単にy=(x+3)(x-3)のグラフが書きやすいだけなんだけど。
実際にはグラフを書いて解答しているわけではない。 理由としてはグラフが
描けてしまったときには既に
(x+3)(x-3)>0
の範囲が求まってしまっているから、グラフを書く過程を考えれば
まず、(x+3)(x-3)=0となる場所がどこで・・・
とかやるはずだろ、だから、結局(x+3)(x-3)>0を解くときにグラフを利用しているのではなく
グラフを書いた時点で範囲が分かっていないとおかしい。

ということで、別に解き方が違うわけではない。
550542:02/05/28 12:03
>>547
タイプ1, グラフから視覚的に解く解法
タイプ2, 正負に注意してして式から解く方法
タイプ1はタイプ2からでも解けることが>>538からわかりました。
しかし、タイプ2はタイプ1からでは解けませんよね?
また、cos4θ・cos2θ>0は特殊な例でタイプ2からでも解けるし、
2倍角の公式を使えば、3次間数のグラフからでも解けますね。
>>542 >>550
そのようなパターン化にこだわって何の意味があるかわからんが。
例えば(x-3)(y+3)<0 を考えるときだって、
xyz空間で z=(x-3)(y+3) のグラフを考えれば
君のいう「パターン1」の解法もできるよ。
ただグラフ(この場合「曲面」)を書いて調べるのが面倒だけど。
552132人目の素数さん:02/05/28 12:44
>542
>549で言っているように、タイプ1で解くということは、本質的にタイプ2で
解いていることと同じなのです。だから、この種の問題はタイプ2の方法で解く
ものと理解すればよいのでは?
>>550 だったら cos4θ・cos2θ>0 はタイプ3、
「グラフから式の正負に注意して解く法」がいいぞ。

どうせθは90度までの角度の間だろう。その範囲で
同じグラフ上に y=cos4θ と y=cos2θ のθ2本の曲線
を書くんだ。その間に前者は 4θ だからcos の全周期、
後者は半周期を描く。問題の式は両者をかけたものが
正となる範囲だから、個々の値のプラスマイナスを
見れば一目瞭然だろう。
554132人目の素数さん:02/05/28 13:04
あれほど、書いたのに・・・。
555542:02/05/28 13:47
>>550
お返事どうもです。私もパターン分けはあまり好きじゃ無いんですが、整理しないと
よくわからなかったので、パターンで分けただけです。

>例えば(x-3)(y+3)<0 を考えるときだって、xyz空間で z=(x-3)(y+3) のグラフを
考えれば君のいう「パターン1」の解法もできるよ。
むりやり同じように解くと難しい話になるんですね。すいません、
よくわかりませんでした。

>>552
タイプ1をタイプ2と“同じようにも解ける”という話で、解法は歴然と違いますよね?
タイプ1とタイプ2のどこが一緒なんですか??そう思えないんですが。
タイプ1かタイプ2のどちらで解くのかという問題は、結局グラフがイメージできるか、
できないかという問題ですか?549さんの言っていることは全くわかりませんでした。
しつこくてすいません。物わかりが悪くて・・。
556名無数:02/05/28 13:48
二浪の悲しみの質問です‥(´Д⊂ヽウウッ

漸化式の破産確率のへんで使うヤツなんですけど、
An+2+pAn+1+qAn=0
は、X^2+pX+q=0
の二解αβを用いて
An=●α^n+◆β^n
と表せれる(●、◆はAnが満たす物を上式に代入して求める)ってなるのはなんでっすかっ??!
高校レベルの数学の頭で理解できるよーに証明なり説明してもらえませんでしょーか?!
だれか助けて!!
557542:02/05/28 13:49
>>553
解答ではなぜかタイプ1で解いていました。私はタイプ2で解いたので
混乱しましたが、タイプ2のほうがやはり速かったんですね。どうも。
558132人目の素数さん:02/05/28 13:50
f(x)=sin(1/x)は開区間(0,1)において有界連続であるが、
一様連続ではないことを示せ。

一様連続って出てくるとさっぱりです。どうやるんでしょうか?
559132人目の素数さん:02/05/28 13:51
556様。レスさせていただきます。

あなたが今まで経験なさって来た全てのことは
この世での修行です。
何一つ無駄なことはありません。
神仏への感謝の気持ちを常に持つことにより、
人間界で言われている、いわゆる運気は上がります。
私の見たところ、あなたの前世は修行僧、(山にこもって荒行をし、
里に降りて托鉢をする、)子供に慕われた無骨な男性です。

あなたは、現在自分の大切に思っている人の代わりに
数々の苦難を受けています。(上記の一つ前の前世もそうでした。)

写経をし、一日一回(出来れば気づいたとき何度でも)
自分を守ってくれる存在に手を合わせましょう。
気持ちが穏やかになり、平和な生活が送れるでしょう。
あなたに乗り越えられない出来事は決して起こりません。
自分を信じて精進なさって下さい。やがて光があなたを包みます。

力はそれほど強くないのですが、3体ほどの神様が見えます。
彼らは力を合わせてあなたを守ります。
人のいいというか、感じの良い神様です。(おじさんです。)
あなたが勇気を持って己を信じ、生きていくことこそ、
彼らにとって最大の栄養になります。
あなたは美しい。あなたは多くの人に愛されています。
心を強く持ってお励みくださいませ。
心をこめて、合掌。
560556:02/05/28 14:03
ウウッ
合唱!!!
(>_<)
∩∩|
>556
An+2 + p An+1 + q An = 0
をなんとか等比数列に変形したいのだ。
つまり、
An+2 - a An+1 = b (An+1 - a An)   (1)
の形。そうすれば
An+2 - a An+1 = b^(n+1) (A1 - a A0) (*1)
とできるだろ?
(1)を展開して整理すると
An+2 - (a + b) An+1 + ab An = 0. (2)
これをもとの前科式と比べると、

a+b = -p , ab = q.
これは2次方程式の解と係数の関係だから、
a,b は x^2 + p x + q = 0 の解。 (3)

つまり、x^2 + p x + q = 0 を解けば、 (1) に変形するための a,b を
求めることができる。

で、実は (2) は (1) 以外にも
An+2 - b An+1 = a (An+1 - b An) = a^(n+1) (A1 - bA0) (*2)
とも変形できるから、
(*1) と (*2) を An+2 と An+1 の連立方程式として
解けば
An = 何か a^n + 何か b^n と求まるというわけだ。

がんがれや。
>>556
漸化式 A(n+2)+pA(n+1)+qA(n)=0 ……☆は
 A(n+2)-αA(n+1) = β{A(n+1)-αA(n)} ……(a)
と変形できる(実際(a)を変形すれば☆に戻る)。
また
 A(n+2)-βA(n+1) = α{A(n+1)-βA(n)} ……(b)
とも変形できる。
(a)からA(n+1)-αA(n)=(ナントカ)β^(n-1)
(b)からA(n+1)-βA(n)=(カントカ)α^(n-1)
となる。

あとはがんばれ。
563名無数:02/05/28 14:10
なるほど☆彡
561さん562さん、ついでに599さん
ありがとーです!!
がんばりまくります(>_<)
>>556 次のようにも理解できるよ(ただ、数学的には少し乱暴)。
U と I という「演算子」を定義する。U は「昇項演算子」と
呼ぶべきもので、それを An に作用させると U An = An+1
となると考える。2回作用させると UU An = U^2 An = An+2
というわけだ。Iは「恒等演算子」 IAn = An.

もとの漸化式を U, I で書けば、U^2An + pUAn + qI An = 0.
つまり、

 (U^2 + pU + qI)An = 0.

この演算子の部分だが、x^2+px+q = 0 の解をα, βとすれば、
 (U-αI)(U-βI) = 0 と書けるわけだ。

数列の一般項だが、An=0 というくだらない解を除けば、
(U-αI)An = 0 あるいは (U-βI)An = 0 が答となる。
前者は、An+1 = αAn ということだから、An = α^n Ao.
後者からは An = β^n Ao が解となる。どちらも、もとの
条件式を満たすので、あとは初期条件に合うように適当に
ブレンドしてやればよい。

2次方程式の解と数列の解が対応するのは、上のように演算子
という立場から同じ式がでてくるからだ。ちなみに、(常)微分
方程式も同様に演算子で解くとができ、やはり n次方程式の
解と対応する。
565132人目の素数さん :02/05/28 14:26
√-1 = i とおくとなぜ計算がうまくいくのですか?
(-1)^(1/3)=iじゃだめなんかい?
566132人目の素数さん:02/05/28 14:37
>565
(-1)^(1/3)って、-1じゃん。
>>565 こう考えたらどうかな。複素数というのは、それまで実数の
1次元数直線で考えていた数学を、2次元の複素平面に拡張する手法。
つまり、z = x + i y として、実数部が x軸を、虚数部が y軸を
あらわす。x軸と y軸は直交しているのが望ましいが、それを保証す
るのが √(-1) = i の定義だ。

つまり、数直線上の 1 と マイナス1は原点を中心に 180度回転した
位置関係にあるが、1をその半分だけ(√)まわしたのが i という
わけだ。j = (-1)^(1/3) (まぎらわしいので j と書くよ) を
虚数の単位として、w = x + y j でも平面を覆えるが、この場合の
y軸は x軸を 60度まわした斜交座標なので、距離の計算などに不便
があるというわけだ。
>>566 お気づきかと思うが、w = (-1)^(1/3) は
w^3 + 1 = 0 という3次方程式の解なので、答えは 3つある。
-1 はその一つ。おそらく >565 はその他2 つ、 ω = (1+√3i)/2
ないし ω^2 を意図していたのだろう。
569132人目の素数さん:02/05/28 15:14
>>567
曲線座標は定義できるのですか?
570まおまお:02/05/28 15:36
>>558
「有界」と「連続」は問題ないっすよね。

ひねった言い方をすると、「コンパクト集合上で定義された連続関数は
一様連続である」ということを知識として知っていれば、任意のε>0に
対してf(x) = sin (1/x)が閉区間[ε,1]上で一様連続であることが分かるハズ
です。だから、「一様連続でない」ことを言うためには、0付近の様相が
ポイントになる、ということが推察されるでしょう。

f(x) = 0 を与えるような点は、x = 1 / (nπ)です。n をどんどん大きくして
やれば、x はどんどん0に近付いていきます。
同様に、f(x) = 1 or -1 を与えるような点は‥‥‥ってな具合に考えていき
ましょう。
>>565
ωを、方程式 x^3=-1 のx=-1以外の解、すなわち方程式 x^2-x+1=0 の解と置く。
そして、i と実数から複素数という数の体系を作ったように、
ωと実数から数の体系を作ってみる。
ところがそうやって作った体系は、ω=1/2+√3/2i←→i=-1+2/√3ωという置き換えで複素数と対応できる。
つまり、結局普通の複素数を作ったのと同じこと。
>>569 複素数の平面の性質を調べるのに、直交座標以外のものを
使うより、直交座標の平面を適当な関数で曲線座標に変換してし
まうほうが楽だ。たとえば z = x+iy を w = exp(z) で変換する
と、極座標のようなものが得られる。あとは極座標のほうの図形を
逆変換で直交座標に写すなどして性質を調べると、便利なことが
ある。「正則関数」「等角写像」などのキーワードで調べてごらん。
>>571 のコメントに、さらにちょっとコメントする。x^2-x+1 = 0
は係数はすべて実数(整数)の 2次方程式だが、その解は実数には
含まれない。したがって、その解の一つ ω を実数につけ加え、
x + yω 形式の数を考えることは、実数の一つの拡張となる。この
ようなものを「ガウスの整数」と呼び、整数論と深いつながりが
ある。複素数は、実数に x^2+1=0 という方程式の解 (これこそが
±i) をつけ加えたものである。

複素数の面白いこと(そして肝要なこと)は、係数を一般の複素数
とした xの多次方程式(代数方程式) の解は、すべて複素数内で
求められることで、体系を複素数に取ると、代数方程式はその
中で解けてしまう。実数係数の方程式には実数解を持たないものが
あり、結果として虚数・複素数という発展があったわけだが、
複素数は、もうこの先なし。どんずまりだ。これが代数を複素数
で考える、最大の理由だ。
574132人目の素数さん:02/05/28 16:41
両辺を0で割ることはダメだと思うんですが、両辺をに0を書けることは良いのですか?
575学部1年教養数学:02/05/28 16:42
x^2+1/(x^3-27)を積分せよという問題

これを部分分数に分けてみて

∫(10/27)/x-3 dx + ∫(17x/27 + 7/9)/x^2+3x+9 dx

というところまでは導けたのですが
ここからどうすればいいのか分かりません。
いろんな本を読む限りではarctanを上手く利用するようですが・・・。
何故、駄目だなんて、悲しいこと言うんですか?
やる前から「駄目だ」って思ってたら、本当に駄目人間になっちゃうよ!!
577575:02/05/28 16:44
あ、質問する立場なので当たり前なのですが
回答の催促はしませんので気長に待ちます。
暇なときでよろしいのでぜひ。
578574:02/05/28 16:52
私も、質問する立場なので当たり前なのですが
回答の催促はしませんので気長に待ちます。
永遠にまっとれ
580132人目の素数さん:02/05/28 16:55
複素数以外のものすごい数はないのか?
おしえてけろ。
>>575
まず x^2/(x^3-27) + 1/(x^3-27) と分けるんだ。
すると第1項の不定積分は (1/3)log|x^3-27| になる。
あとは後半だが・・・
582132人目の素数さん:02/05/28 16:57
>>575
arctanをうまく使う。。ということはこのことは知らないのかな?

x=tanyとおいて

dy/dx=1/(dx/dy)
=1/(1/cos^2 y)
=cos^2 y
=1/(1+tan^2 y)
=1/(1+x^2)

ということなので、
∫1/(1+x^2) dx = arctan x +積分定数。

これを利用しろって事だろ。
一番スゴい素数は3だそうだが
584132人目の素数さん:02/05/28 16:59
>>579
いじわる
585575:02/05/28 17:00
あ、いけね。大事なこと書くの忘れてた。
この問題、部分分数に分けて計算しろという条件がついているんです。
別にいじわるをしているつもりはない
永遠に待つという行為を通じて、0で割ることの意味が見えてくるのだ
587575:02/05/28 17:05
>>582
そうですね。
とりあえずarctanの定義は知ってますが
それの実践的な使い方がなかなかちょっと。
588132人目の素数さん:02/05/28 17:07
>>586
あなた本気ですか?教えてくれないならイイモン!プンプン
>575
ま、とりあえず
x + 3/2 = (3 √3)/2 X tan t
として x を消去してみな。
590132人目の素数さん:02/05/28 17:13
閉集合の補集合が開集合となる。

これを証明したいのですがどうやってやればいいでしょうか?
591575:02/05/28 17:14
うーむ、ダメだ・・・。
せっかくいろんなアドバイスを頂いているのにまだ全然わからん。
どうやら自分の努力が足りなかったようです、
これからまた大学行ってもっと調べて出直してまいります。
というわけで夜までレスできませんので悪しからず。
>>575 arctan(x)' = 1/(1+x^2) だから、積分をなんとかその
形に持ち込め、ということだろう? アンタの部分分数による
積分の第2項は、分子が xの 1次式だから、この中から

∫(2x+3)/(x^2+3x+9)dx = log(x^2+3x+9)

となる部分をくくりだすと、∫1/(x^2+3x+9)dx の形が残る。
arctan はここから出るんだ。

分母を平方完成させて、x^2+3x+(3/2)^2 - (3/2)^2+9 として
ごらん。変形すると(15/2)^2・((2/15)x+1/5)^2 + 1)となる。
どうだい、arctan が見えてきたかい?
593592:02/05/28 17:24
あらま、分母の変形ちょっとまちがえた。
(4/27)(((3√3/2)x + 9/4)^2 + 1) かな? とにかく、方針は
これでいい。
594575:02/05/28 17:27
まだいますよー。
もう準備終わったので今度こそ本当にすぐ家を出ますけど。
>>592-593
重要なヒントありがたく思います。
問題検討の参考にさせていただきます。
595132人目の素数さん:02/05/28 18:13
ありがちな問題ですが、明確で簡潔な解答がなかなか得られないので投稿します。

地球上でボールを投げると引力により放物線を描き、やがて落下します。
最高の高度に達した時、速度はゼロになります。
そのゼロになる瞬間がよくわからないのです。
速度が1、0.1・・・0.00000001・・・0.00000000000000000000000000000000000000000001
と徐々に速度が落ちていきます。
しかし小数点以下にゼロが1兆個続いたあとに1が付いていた場合、それはゼロではありません。
こうして際限なくゼロが増えたとしても、その後に1が付けばそれはまだゼロではありません。
(小数点以下にゼロが無量大数個続いた後に1が付いてもそれはゼロではないはず。)

それ(速度)がゼロになる瞬間はいつなのでしょうか?
もしゼロになる瞬間があったのなら、小数点以下のゼロの数は有限個だと言えるのではないでしょうか。
投稿ですか(w
>>595 いや、良いところに気が付いたね。そうなんだよ。空間が
連続だとすると、ボールはいつまでも停止できないんだよ。先般ゼノン
という人がそれを指摘して、数学者に猛省をうながしたのは、記憶に
新しい。

だから、空間は不連続なんだ。電子の大きさくらいの、格子構造だろ
うと言われている。

「連続」というのは、数学の概念上だけのことで、実空間の一種の
近似でしかないんだ。

デムパ警報発令中。
>>595
「ゼノンのパラドックス」で検索しよう。
記憶に新しいですか(w
600384:02/05/28 18:26
>>397
お返事ありがとうございます。さっそくやってみます。っていうかこのスレすごい
勢い^^;
601高3:02/05/28 18:33
こんにちは。
ここはすごくよいインターネットですね。
次の問題が全くわかりません。
方針やヒント、考え方などを。と思ったのですが、
できたら解答していただけるとうれしいです。
途中は省いていただいてもたぶんだいじょうぶです。

問題-1を解とする実数係数のxの方程式
x^3+ax^2+bx+c=0
の他2つの解をα、βとする。
複素数平面上で、1、α、βが、面積6√3である1つの正六角形の異なる3頂点になっている。
a、b、cを求めよ。

よろしくお願いします。
インターネットですか(W
603高3:02/05/28 18:36
上の問題の解は1です。-1じゃないです
604高3:02/05/28 18:46
>>602
はい。ここって、インターネット。ですよね?
605132人目の素数さん:02/05/28 18:47
なんかネタっぽいがとりあえずヒント。
面積6√3の正六角形一辺の長さは2
もうひとつヒント。実数係数の3次方程式なら、複素解を
答にもつなら、それは 2つあり、互いに複素共役。
>601
とりあえず六角形の一辺の長さは
2

解の一つが 1 なので、1 + a + b + c = 0.
このとき、方程式は因数分解できて、
(x-1) ( x^2 + (a+1) x + a + b + 1) = 0.
解と係数の関係から、
α+ β = - a -1
αβ = a + b + 1

で、今 αとβは一つの2次方程式の解だから
実軸に関して対称。
つまり、 六角形は実軸上に頂点を2つ持つようなもの。
αとβは例えば (0, ±√3) とかになる。
4通りあるけど。

あとはがんがれ。
608607:02/05/28 18:54
>605-606
あう。遅かった・・・
609601さんへ:02/05/28 18:55
解が2つ出てきた
(a,b,c)=(5,11,7),または(-7,11,19)

六角形の一辺の長さは2だろ
(α、β)=−2±√3iまたは4±√3iになった

これであってるかな、不安。
610高3:02/05/28 18:57
ありがとうございます。
『インターネット』はネタでしたが、問題はマジで質問しました。
どうもありがとうございます。
もう一度頑張ってみます。
あ、 -1 を解だと思ってしまった人だ。 >609
612609:02/05/28 19:00
603を見る前に書き込んでしまった。
それなら解は(a,b,c)=(3,3,−7),または(−9,27,−19)
613高3:02/05/28 19:04
>>612さん。
ごめんなさい。
でもありがとうございました。
614132人目の素数さん:02/05/28 19:09
両辺を0で割ることはダメだと思うんですが、両辺をに0を書けることは良いのですか?
615607:02/05/28 19:09
>612
ん?解は4つあると思うけど。
全部隣り合っているときとか。
616132人目の素数さん:02/05/28 19:18
問題:
任意の自然数にnに対し,θ=90/nとする。
a[n]とb[n]を次のように定める。
a[n]=sinθ*sin2θ*sin3θ*・・・・・*sin(2n-2)*sin(2n-1)θ
b[n]=sin2θ*sin4θ*sin6θ*・・・・・*sin(2n-2)θ
このとき2以上のnに対して、a[n]とb[n]の間に成立する関係式を求めよ。

関係式とは一般的にどんな物なんですか?等号で結ばれた式と言うことで
しょうか?そうでしたら、両辺を足して、
a[n]+b[n]={sinθ*sin2θ*sin3θ*・・・・・*sin(2n-2)*sin(2n-1)θ}
+ {sin2θ*sin4θ*sin6θ*・・・・・*sin(2n-2)θ}で終わりだと思うのですが。
これは問題文が不備なのでしょうか?それとも関係式の認識を間違えているの
でしょうか?それと、不等号も関係式に含まれていますか?
いっぱい聞いてすいませんが、よろしくお願いします。
617132人目の素数さん:02/05/28 19:24
>>614
かけてどうすんの?
618132人目の素数さん:02/05/28 19:26
f(x)=0の両辺にあるものを掛けると、式が綺麗になるのですが、0を
掛けちゃだめだという規則があったら場合分けしなきゃいけないな
と思いまして。
619618:02/05/28 19:27
>>617です
620132人目の素数さん:02/05/28 19:27
断定的なことはいえないが、>>616の問題はなにかの誘導の匂いがする。
もしそうであるなら問題の「本丸」をおしえてくれ。そうすれば出題者の
意図することが分かる。
そうでなくても、多分、最大限出題者を好意的に解釈するなら、
a[n]=f(b[n])であらわすとか、そんなんだろ?
ま、とにかく「本丸」がでてから、それからだね。
621612:02/05/28 19:29
あ、そうですね、俺ってだめだなあ。
じゃあ、解に
(a,b,c)=(−1,3,−3),または(−5,11,−7)を追加
622132人目の素数さん:02/05/28 19:32
>>618
そういうシチュエーションならありうるね。
多分実数か複素数の範囲で考えているのだろうから、その前提で話を
すると、0になりうるものは両辺にかけて一向に差し支えない。
ただ、そのあとでてきた式が意味を持つかは別の問題だが・・・

ただ、そんなことより、その今君が悩んでいる問題そのものを具体的に
書いたほうがよいと思うよ。一応君が悩んでいる解法に沿ってみなさん
答えてくれるだろうし、他にエレガントな解法があればそれも教えてもらえ
て勉強になるだろうし。
62399:02/05/28 19:36
-------風俗の総合商社・MTTどこでも-------

〇デリバリーヘルス〇デートクラブ〇女性専用ホストクラブ〇
〇ハードSM奴隷クラブ〇レズビアン倶楽部〇ホモ・オカマ倶楽部
〇変態痴女と遊ぶ会〇痴漢・覗き趣味の会〇変態同好会・各種!
●楽しく遊べます! 090-8002-8356番
-----------美男・美女会員など多数在籍中-----------
  http://www.mttdocomo.jp/
-----女性アルバイト随時募集・高収入(日払い)月100万円可能-----
-----レズビアン・スタッフ●ホモスタッフ●女性専用ホストスタッフ同募-----
http://www.mttdocomo.jp/
------------------------------------------------



624タスケテ:02/05/28 19:51
Aの棒がx軸で回転するとBの棒がz軸移動。
んでもAの回転がマイナスでもプラスでも、Bはプラスの移動。
関数の知識が無いおいらはこの単純な行動を表す式が判らないー!
>>616 問題文はあいまいだが、a[n] の 積項の一部に b[n] が
現れる(a[n]はb[n]で割れる)ことから、a[n]/b[n] が何か
簡単な関係になりはしないかと考えてみよう。

予想としては、a[n]/b[n] = 1/n。(あるいは a[n] = b[n]/n)

証明がんばれ。
626132人目の素数さん:02/05/28 20:04
f(x)=x^5/3 や f(x)=x^4/3
といった関数がどういった写像か見極めるためにはどうしたらいいのでしょうか
(一対一や上への写像、同相写像など)
627132人目の素数さん:02/05/28 20:41
>590
距離空間、それとも位相空間での話?
どちらにしても、任意の閉集合の補集合が閉集合であると仮定して
矛盾を導くという方針でいけると思う。
628132人目の素数さん:02/05/28 20:43
ちょっと質問なんですが、「一般解」とは普通の解と
どう違うのでしょうか?
629132人目の素数さん:02/05/28 20:44
x^2+x+1=0の解の1つをωとするとき、次の値を求めよ。
(ア)ω^4+ω^2+1  (イ)ω^20+ω^15+ω^10

という問題がわかりません。よろしくお願いします。
630132人目の素数さん:02/05/28 20:44
>626
一対一、上への写像、同相写像の定義は既知でよいの?
どの部分が分からないのかな?
631132人目の素数さん:02/05/28 20:49
>629
ヒント:
ω^2+ω+1=0、さらにこの式の両辺に(ω-1)をかけると、
ω^3-1=0。よってω^3=1。
だから、例えば、ω^4=ω*ω^3=ωとなる。
632626:02/05/28 20:50
>>630
はい わかります
始めのとっかかりがまるでわかりません・・・
633名無人 ◆TCcC3EVE :02/05/28 20:54
x^2+x+1=0の両辺にx-1を掛けてx^3=1
x=ωを解に持つのでω^3=1
ω=ω
ω^2=ω^2
ω^3=1
ω^4=ω^3*ω=ω
ω^5=ω^3*ω^2=ω^2
ω^6=ω^3*ω^3=1




これを利用する
634名無人 ◆TCcC3EVE :02/05/28 20:56
>>631
かぶったね スマソ
635132人目の素数さん:02/05/28 21:05
工房で場合の数と個数みたいな単元を教科書で勉強していたんですけど
集合A={a,b,c,d}の部分集合の個数
Aの部分集合について、a,b,c,dがこの部分集合に属している場合を○、属していない場合を×とすると
○、×を並べる重複順列の総数が求める部分集合の個数である
よって
2^4=16

この
Aの部分集合について、a,b,c,dがこの部分集合に属している場合

さっぱり意味が分からないので教えてください
636132人目の素数さん:02/05/28 21:06
>636
じゃあまず、上への写像かどうかを調べてみよう。
実数から実数への写像:f(x)=x^5/3の場合は、
任意の実数yに対して、x^5/3=yとなる実数xが存在することが示せれば、
上への写像となる。
さらにx≠yならば、x^5/3≠y^5/3が言えれば単射。上への写像でかつ
単射なら、一対一写像。
一対一写像で、fの逆写像が連続ならば同相写像。
637626:02/05/28 21:10
>>636
ありがとうございます。

x^5/3=yとなる実数xが存在することが示せれば
とはどのように示せばいいのでしょうか・・・
638629:02/05/28 21:11
>>631
>>633
どうもありがとうございました。解けました。
639132人目の素数さん:02/05/28 21:11
>>635部分集合がわからないならまず意味を調べる。
部分集合がわかるなら可能な場合を全部紙に書き出す。
640132人目の素数さん:02/05/28 21:13
>635
例えば、集合B{a,b}、集合C{a,c,d}、集合D{c}を考える。
B、C、DはそれぞれAの部分集合である。
aは部分集合Bに属している。(Bの中にaが含まれているから)
また、aは集合Cにも属している。しかし、
aは集合Dには属していない。(Dはcしか含んでいないから)
同様にbはBには属しているが、C、Dには属していない。

こんな感じでイメージができるだろうか。
641132人目の素数さん:02/05/28 21:15
>>639-640
ありがとうございます
つまり集合Aの中に集合Bがあってそれにa,b,c,dそれぞれが属してるか調べるわけですね?
642132人目の素数さん:02/05/28 21:22
>637
うーん・・・普通に、yを任意の実数としたとき、x=(3y)^(1/5)が
x^5/3=yを満たす、でいいんじゃないかと思うが・・・。
(3y)^(1/5)なる実数が存在することは明らかとしていいのか
ちょっと不安。ごめん、少し自信ない。誰か助けてくれ。
643132人目の素数さん:02/05/28 21:31
aを含む部分集合と
aを含まない部分集合は
どちらが多いか考えてみそ
>635
Aの要素a,b,c,dのそれぞれについて、
○か×かを対応させる。

たとえば、
a→○
b→×
c→×
d→○

というような感じ。
○に対応しているものを集めて集合を作る。
この場合は、{a,d}。これでAの部分集合が一丁上がり。

○と×の(順序を区別した)組み合わせに対して、
Aの部分集合がただひとつ対応している。

ということは、Aの部分集合がいくつあるかを調べるには
○と×を並べる重複順列が何通りあるかを調べればよい。


645申し訳ないが教えてくれ。:02/05/28 21:34
nを2以上の自然数とする。
x^nをx^2-2x+1で割ったときの余りを求めよ。

簡単そうなんだが、やり方を忘れてしまって・・・
>642
>うーん・・・普通に、yを任意の実数としたとき、x=(3y)^(1/5)が
>x^5/3=yを満たす、でいいんじゃないかと思うが・・・。

トートロジーっぽいね
存在を既知としている場合は(3y)^(1/5)みたいな
5乗根というものを使ってもいいのだろうけど

敢えて存在の証明からするのであれば

任意の実数aに対して

f(x)=x^5/3-a

とでも置く。これは十分大きなxと十分小さなxを使えば
符号が分かれるから、中間値の定理によりf(x)=0なるxが存在する

って感じでは?
>645
x^2 - 2x+1 = (x-1)^2

x^nを(x-1)^2で割ったときの余りをax+bとすると、
x^n = Q(x)(x - 1)^2 + ax + b
とかける。

x=1を代入すると
1 = a+b

あともうひとつ、関係式を作ろう。
x^n=(x^2-2x+1)f(x)+ax+b
とおいて
両辺微分
それぞれの式にx=1を代入して
a・bを求める
649642:02/05/28 21:43
>646
なるほど、そうですね。サンクス!
650申し訳ないが教えてくれ。:02/05/28 21:52
>>647
そこまでは分かるんですよ・・・。
最初に言っておけばよかったですね。スマソ
>>645
x=t+1と変換すると
「(t+1)^nをt^2で割った余りを求めよ」になる

n≧2でニ項展開して(t+1)^n=1+nt+n(n-1)t^2/2+・・・+t^n
よって余りは1+nt
tをxに戻してnx+(1-n)
変換せずに最初から
x^n={(x-1)+1}^n=1+n(x-1)+・・・+(x-1)^n
と展開してもよい
653証明問題で質問です:02/05/28 22:03
‘7^n -1 が6の倍数であることを証明せよ’

という問題で、合同式を使って証明する事は可能ですか?
>653
可能です。

(6+1)^n - 1
655132人目の素数さん:02/05/28 22:09
7≡1   モド6
7^n≡1   モド6
7^n-1≡0  モド6
656厨房以下:02/05/28 22:13
申し訳御座いません、仕事の都合でグラフを使うことになったのですが
当方数学とはほぼ縁の無い生活を長々と続けていたため厨房時代のことをすっかり忘れてしまいました。
y>2/3x+2というグラフなのですが、これはどのように図に表せばいいのでしょうか?
これは傾き2/3ですよね。
僅かに残っている記憶だと、「○○からx軸に3進んでy軸2下がる…」とかいう方法があったような気がするのですが。
どうかお助けください。
よろしくお願いします。
657626:02/05/28 22:13
>>646
>>642
ありがとうございました!!助かりましたTT
>656
y=(2/3)x+2
というグラフなら直線なので2点決まれば
一つに定まる。

ので、xに0とか3とかを入れて、yを求めてみる
659 :02/05/28 22:16
b(x)=exp(-1/x^2) x>0のとき
b(x)=0 x≦0のとき

このグラフはどんなグラフですかね?
660厨房以下:02/05/28 22:18
>>658
即レス有難う御座いました。
661653:02/05/28 22:21
>>654>>655
どうもありがとうございます!

一応、自分では、mod6の時のnと7^nの表を書いて証明しようとしたのですが、
mod6で考える時はnは1〜5までだけでいいんでしょうか・・?
662こんな現実が・・・・:02/05/28 22:25
>661
0〜5
664132人目の素数さん:02/05/28 22:42
放物線y=x^2-2ax+a^2+2a-3・・・@,と直線y=-a^2・・・Aは異なる2点で交わる。
ただしaは定数である。
(1)(アイ)<a<(ウ)であり、2つの交点のx座標はa+-√(エ)a^2-(オ)a+(カ)である。

(2)@,Aの2つの交点をx座標の小さい順にA,Bとする。
線分ABの長さが2√3より大きくなるのは、(キク)<a<(ケ)のときである。

(3)線分ABの長さは、a=(コサ)のとき最大値(シ)をとる。


という問題がわかりません・・・。
これでも一生懸命考えたんですが、10点をとるような数学ドキュソなので
全く意味がわかりません(汗
ご面倒かと思いますが親切な方、よろしくお願いします。m(__)m
665匿名希望:02/05/28 22:44
             __,,.-‐''''''''''''''''''''ー:-、_
          ,,.-'";:;:;:;:;:;:;:;:;:;:;:;:;:;:;:;:;:;:;:;:;:~`ヽ:、_
         ,/;:;:;:;;:-'"~~''ヾ/~~~'''''--、;:;:;;:;:;:;;:\
        /;:;:;:-'"            `ヾ;:;:;:;:;:;;ヽ、
      /;:;:彡                ヾ;:;:;:;:;:;;:ヽ
     /;;::;:三                  i;:;:ミ;:;:;:;::ヽ
     ,i";:;:彡         - ''   -  、.  ヾ;:;:;:;:;:;:;:;:;i     >>664先生
     /;:;;:;彡         ,,...:-'"  ,,,.....   ミ;:;:\;:;:ミ|
    /;:;:;:;:;:彡    - ' ''""   - ''"      ミ;;;;;;;;ヾ;;;;|     なにをいってもね、無駄ですよ。
    i!;:;/;:;:;彡             ,,;;;;;;;!!!!!:、  ヽ、;:;:;:;;::、  ええ。躾ができていませんから。
     !i;:;:;:;:;:;:;| ,,;;;!!!!;;;;::::  :::.    '"" _   ノ i  /  |
     ヾ;:彡/     _ _  :::  :::: ,,.ィゝ-''ヽー ' ヾ/ゝ |
      !;:;:;:| ゝ_,.-''"ゞ-'ゝ,;:  ::::;:;:;::ー= '"    :::ノ |
       i、/ ,.-‐>ゝ: : ' "   ::::;:;:;:      : : :;  /
       :|        ..:/   :::: '⌒ヽ       ;::‐'
       ゝ、    ..::::: ゝ - 、  ,:-‐-' `ヽ、  ..  :|
          i   ::::/::    `''''       i::::    |
         ヽ   !:::: __,,,..::;;;;;;;;;;;::::‐-っー      ,!
            ヽ  `::: `ーゞL:L:L:Ll.-'~ノ  .::   /
           \  `  `ー-、二二,.:''   .::  ,ィ"
      ,,...--;'''"";:ヾ, :..           ,;: /
   ,..:-'"        ー:、  .....,,,,,,....    ,.::'"
                ~`''''ー--―''''""
666132人目の素数さん:02/05/28 22:45
>661
7^n のnにまでmodを使うのはちょっとちゃうんじゃない?
使うのは7に対して
667653:02/05/28 22:48
>>663
あ、0もですね・・(汗

レスくれたみなさん、どうもありがとうございました。
これですっきり寝れます・・
668653:02/05/28 22:54
>>666
7に対して使ったらどうなりますか?
やっぱ655さんのやり方が一番良い方法なのかな
>624
なんのことかさっぱり分からんが。
棒って何?
何かの機械の動作か?
670 :02/05/28 22:59
>>646
f(x)=x^5/3-a
とでも置く。これは十分大きなxと十分小さなxを使えば
符号が分かれるから、中間値の定理によりf(x)=0なるxが存在する

ここをくわしく教えてください 十分小さなXと大きなXとはどうやってつかうのでしょうか
ちょっときになったもので・・
671132人目の素数さん:02/05/28 23:07
>664
放物線y=x^2-2ax+a^2+2a-3=-a^2
x^2-2ax+2a^2+2a-3=0
xの2次式と見て、判別式や解の公式は書けるかな?
672ぢだま:02/05/28 23:14
グラフ理論なのですが
点数nのグラフをV1,V2,V3に分割するとき、異なる分割の個数?
(1)制約が|V1|=4,|V2|=3,n>7のときいくつか?
(2)制約が|V1|+|V2|=n/2 (nは偶数)のときいくつか?
グラフ理論さっぱりなのでよろしくお願いします
673132人目の素数さん:02/05/28 23:17
x+1/x=5のとき、x^2+1/x^2=()、x^3+1/x^3=()である。

って問題なんですが・・・。
数Aの問題ですが、全然忘れちゃって出来ません(汗
ヒントでももらえると嬉しいですm(__)m
674 :02/05/28 23:20
f(x)=3x+5 の f^-1 とはどうなるんですか?
675132人目の素数さん:02/05/28 23:21
x+1/x=5両辺2乗してみたら?
676132人目の素数さん:02/05/28 23:24
>>674
f^-1=1/(3x+5)
これでいいのかな?
677ななし:02/05/28 23:27
x+1/x=5
なら
両辺を2乗して
x^2+2+1/x^2=25

両辺を3乗して
x^3+(x+1/x)+1/x^3=125
678646ではないですが:02/05/28 23:30
>>670
p=(3a+3)^(1/5),q=(3a-3)^(1/5)とおく。

p^5=3a+3,q^5=3a-3であるから
f(x)=(x^5/3)-aに対して
f(p)=(a+1)-a=1>0,f(q)=(a-1)-a=-1<0となる。
x=pのときfが正で、x=qのときfが負となり、fが連続であるから、
f(x)=0となるxがpとqの間に存在する。

これで納得できますか?
>676
>674 は逆関数のことを言っていると思われ。
680674:02/05/28 23:31
>>676
ありがとう〜〜
681677:02/05/28 23:31
>>674を忘れてた
682677:02/05/28 23:34
>>673の誤りだ
683679:02/05/28 23:36
いいのか!!! >680
684674:02/05/28 23:36
f^-1=(x-5)/3 でいいのかな?
685670:02/05/28 23:38
>>678
すんげ〜わかりました
ありがおう〜〜
686132人目の素数さん:02/05/28 23:40
>>672
(1)|V1|=4,|V2|=3,|V3|=n-7だから
nC4*(n-4)C3かな?
687670:02/05/28 23:44
yを任意の実数としたとき、x=(3y)^(1/5)

のところはx=y^(3/5)とはちゃう?
688132人目の素数さん:02/05/28 23:48
>671
ありがとうございます〜(涙)
今、解き終わりました・・・。ホント助かりました!!
689132人目の素数さん:02/05/29 00:07
L-7m-n=50,2L-2m+n=-8,6L+n=-36
Lとmとnの値を出したいんです
これらの解き方をご教授願います
>>689
第三式からL=-n-36。
これを第1式と第2式に代入しる。
691678:02/05/29 00:12
>>687
{(3y)^(1/5)}^5=(3y)^{(1/5)×5}=(3y)^1=3y
{y^(3/5)}^5=y^{(3/5)×5}=y^3
よって
{(3y)^(1/5)}^5≠{y^(3/5)}^5
ゆえに
(3y)^(1/5)≠y^(3/5)
692132人目の素数さん:02/05/29 00:26
lim[x->0]{e-(1+x)^(1/x)}/x = ?
を教えてください。
693132人目の素数さん:02/05/29 00:27
>689
nを消去するのが簡単そう。
辺辺引き算でもいいし、第3式より n=−6L−36 とし、前の式に代入
文字が2つになれば解けるでしょ。
694132人目の素数さん:02/05/29 00:29
>>690>>693
ほんとにありがとうございました!
なんとか回答を得れました
数学が好きになれそうです
695?H?H?H?H:02/05/29 00:30
2 2  3 1  4 3
 2    4    9
3 4  5 3  5 6
 
3 5
 ?
5 9
 
ある法則に基づいた、上の図のような組み合わせがあります。では、?の中に入る数字は何でしょう?
>695
ナイショ
697132人目の素数さん:02/05/29 00:36
>695
2次の正方行列の行列式の値
698132人目の素数さん:02/05/29 00:46
>695
なめとるンか。といいつつ反応してしまう自分が恥ずかしい。
699687:02/05/29 00:46
>>691
いや違って x^5/3
Xの五分の三乗の逆数はXの三分の五乗じゃない?
700132人目の素数さん:02/05/29 01:01
exp(-1/x^2)とはどういう意味?
指数関数ってなんですか?
どんなグラフになるんですかね?
701132人目の素数さん:02/05/29 01:10
>>700
http://www.vector.co.jp/magazine/softnews/000325/n0003253.html
俺は使ったことないけど、試しにやってみ
702:02/05/29 01:13
すみませんが、お世話になります。

@ 方程式 x−e^(−x)=0の解をcとおく。cは区間(1/2,5/8)
内にあることを示せ。また、そのニュートン法による近似値X及びXとcの誤差の
限界を求めよ。

A 次を示せ。 任意の正数 x1,x2,・・・・,xn に対し、
 (@) a<0 または a>1のとき
  
    1/n(x1^a+x2^a+・・・xn^a)≧{(x1+x2+・・・xn)/n}^a
  
 (A) 0<a<1 のとき

    1/n(x1^a+x2^a+・・・xn^a)≦{(x1+x2+・・・xn)/n}^a
  
  ここで等号は、(@)、(A)どちらの場合もx1=x2=・・・xn のとき、
  その時のみ成り立つ。 
703132人目の素数さん:02/05/29 01:16
>>620>>625
お返事どうもです。これが本丸で、問題のすべてです。
(1)(2)などの区別はありませんでした。
好意的に解釈ということはもとから解釈に幅があるあいまいな問題なんですね。
好意的解釈とはつまりsinを残すなということですか?
704132人目の素数さん:02/05/29 01:25
(√2+√3−1)
 ̄ ̄ ̄ ̄ ̄ ̄ ̄ ̄
(√2+√3+1)

をできるだけ簡単な形にするってあるんですがどうするんですか?
(最終的には分母が整数になるようです)
705307:02/05/29 01:41
初項が正、第2項と第4項の和が20、第4項と第6項の和が80である等比数列の初項から第N項までの和を求めよ。
教えてください。
706132人目の素数さん:02/05/29 01:43
級数の凝集判定法って何ですか?


>>705
初項をa、公比をrとして立式してみそ。aとrの連立方程式になるでしょ。
708132人目の素数さん:02/05/29 01:46
>>704
分母分子に(1+√2-√3)をかけてみなされ

709132人目の素数さん:02/05/29 01:48
>>704
ちょっとした工夫で計算量はかなり減らせます。
710307:02/05/29 01:50
>>707
その方程式が解けないんです。
711132人目の素数さん:02/05/29 01:54
>>705

ar^+ar^3=20・・・(1)

ar^3+ar^5=80・・・(2)

(2)式を(1)式で割ってみるとよろしい。
>>702 Aについて
f(t)=t^aとおく。
n=1の時は明らか。
n>1として
(n-1)s=x[1]+x[2]+x[3]+...+x[n-1]とおく。
0<x[1]<=x[2]<=x[3]<=...<=x[n-1],t>=0に対し
F(t)=f(x[1])+f(x[2])+....+f(x[n-1])+f(s+t)-nf(s+t/n)とおく
F'(t)=f'(s+t)-f'(s+t/n)
帰納法の仮定よりF(0)>=0 (等号はx[1]=x[2]=x[3]=...=x[n-1] この時s=x[1]=x[2]...であることに注意)
F'(0)=0はすぐわかる。

a>=1の時はf'(t)=at^(a-1)は非減少
∴f'(s+t)-f'(s+t/n)>=0
∴F(t)>=0
特にF'(t)>0(t>0)からF(t)>0(t>0)

a<0の時 F(t0)<0となるt0>0があったとする。
lim[t->∞]f(t)=0から
lim[t->∞]F(t)=f(x[1])+f(x[2])+....+f(x[n-1])>0
f(m)>0となるm>t0が存在するとして良い。
F(s)=0となるt0<sが存在する。(中間値の定理)
同様にF(0)>0でもF(u)=0となる0<u<t0が存在する。F(0)=0の時はu=0とおく
平均値の定理より、F'(t)=0となるu<t<t0がある。
これはf'(s+t)=f'(s+t/n)となることを意味するが、f'(t)=at^(a-1)は(a<0)の時、単調である
ので矛盾
従ってF(t)>=0(等号はt=0,x[1]=x[2]=x[3]=...=x[n-1]の時のみ)
0<a<1の時は
f'(t)=at^(a-1)は単調減少よりF'(t)<0(t>0) ∴F(t)<=0(等号は同様)
a=0,1の時はF(t)=0
x[n]=s+tとおくとx[n]>=x[i](i=1..n-1)であり ns+t=(n-1)s+x[n]=x[1]+x[2]+...+x[n]だから証明された。
713723:02/05/29 07:43
数学的帰納法使うって宣言しなかったけど、入れといてください。
714ぢだま:02/05/29 08:23
672の問題をお願いしますTT
715678:02/05/29 08:59
670さん、>>699で話題にしているのは

>>642
>yを任意の実数としたとき、x=(3y)^(1/5)がx^5/3=yを満たす
に関してでしょうか?

 [1] 「yを任意の実数としたとき、x=(3y)^(1/5)が(x^5)/3=yを満たす」
と解釈すればこの命題は真なのですが
 [2] 「yを任意の実数としたとき、x=(3y)^(1/5)がx^(5/3)=yを満たす」
と解釈するとこの命題は偽になります。
ですからここではx^5/3は(x^5)/3であると考えられます。

そこで私は>>670にあるf(x)=x^5/3-aを
私はf(x)={(x^5)/3}-aだと解釈して>>670に答えたのです。

しかし、質問の発端の>>626さんが「f(x)=x^5/3やf(x)=x^4/3」を
 [ア] 「f(x)=(x^5)/3やf(x)=(x^4)/3」
 [イ] 「f(x)=x^(5/3)やf(x)=x^(4/3)」
のどちらのつもりで書いたのかまでは分かりません。

ただ、特に注意がない場合、
A^b/cと書いてあれば(A^b)/cと解釈するのが規則だと思います。
>>4によれば、指数部が一文字ならA^bと書き、
そうでない場合は指数部にカッコをつけA^(bc)のように書く
ことになっているからです。

ちなみに
>>699
>Xの五分の三乗の逆数はXの三分の五乗じゃない?
は間違いです。
X^(3/5)×X^(5/3)=X^(3/5+5/3)=X^(34/15)≠1
X^(3/5)×X^(-3/5)=X^(3/5-3/5)=X^0=1
ですから{X^(3/5)}の逆数は{X^(-3/5)}です。
716132人目の素数さん:02/05/29 09:04
 すいません。ヘルプお願いします。
「1辺の長さがlの正四面体OABCがある。また、[OP]=l[OA]+m[OB]+n[OC]
とする。l+2m+3n=1が成り立つとき、点Pはいかなる図形上にあるか。」
 という問題です。内分に帰着させるのかな?と思ってやってみるのですが、
うまくいきません。どうか、よろしくお願いします。
717132人目の素数さん:02/05/29 09:34
>>716
「空間中、[OA],[OB],[OC]が一次独立で
 [OQ]=l[OA]+m[OB]+n[OC] とする。
 l+m+n=1が成り立つとき、点Qは平面ABC上の点である。」
ということは分かるのかな?このことを用いて良いのなら
(1/2)[OB]=[OD],(1/3)[OC]=[OE]となるように点D,Eを定めておいて
[OP]=l[OA]+m[OB]+n[OC]
  =l[OA]+(2m)(1/2)[OB]+(3n)(1/3)[OC]
  =l[OA]+(2m)[OD]+(3n)[OE]
と変形する。
ここで[OA],[OD],[OE]は一次独立での係数の和が1であるから
上式よりPは平面ADE上の点であることが分かる。
718717:02/05/29 09:36
>>717 訂正
[訂正前]ここで[OA],[OD],[OE]は一次独立での係数の和が1であるから
[訂正後]ここで[OA],[OD],[OE]は一次独立で、係数の和が1であるから

719678:02/05/29 09:57
>>715

>>699
>Xの五分の三乗の逆数はXの三分の五乗じゃない?

おそらく「逆関数」と打ち込むはずが
「逆数」と打ち込んでしまったんのでしょうね。

確かにf(x)=x^(3/5)に対してその逆関数はf^(-1)(x)=x^(5/3)
f^(-1)[f(x)]={f(x)}^(5/3)={x^(3/5)}^(5/3)=x^{(5/3)×(3/5)}
よってf^(-1)[f(x)]=xが成立するからこれは正しいですね。

ちなみにf(x)=(x^3)/5に対する逆関数はf^(-1)(x)=(5x)^(1/3)ですね。
f^(-1)[f(x)]={5f(x)}^(1/3)=[5{(x^3)/5}]^(1/3)=(x^3)^(1/3)=x^{3×(1/3)}
よってf^(-1)[f(x)]=xが成立するからこれも正しいですね。
720132人目の素数さん:02/05/29 10:08
 >>717
 分かりました。 
平面ベクトルでやったことあるのに、すぐ忘れてしまう...。(ウツ

 お手数かけました。どうもありがとうございました。
721132人目の素数さん:02/05/29 10:10
e^(-1/x^2)を微分するとどうなるでしょか?
722132人目の素数さん:02/05/29 10:15
>721
y=-1/(x^2)とおいてみると、e^y*(dy/dx)
723132人目の素数さん:02/05/29 10:52
なんで分数の割り算は逆にしてかけるんですか?
>>723
掛け算を逆にしたのが割り算なんで、もとにもどしているだけです。
掛け算は足し算の繰り返し、割り算は引き算の繰り返しで、
逆というのは正確には引き算の逆が足し算だという事なんですが。
725132人目の素数さん:02/05/29 11:37
なーーんとなく理解できましたが、では分子と分母を逆にするのはなぜですか?
>>275
分数という数字の表記の仕方がちょっとわかりにくくて、
小学校で最初に勉強するときには、割り算を省略して書くのに使われている気がします。
たとえば 5 ÷ (2/3)なんかだと、5÷(2÷3)といった式を、ただ分数という表記法を教えるのに
都合が言いと最初に考えた人がいて、それでたまたま答えがある計算法で「逆」というのがつかわれている感じです。

ですので、全部調べたわけではないのですが、分数(有理数)を使わないと答えが出ない計算というのは「逆」という
表現が使われている集団内では実際には行われておらず、とりあえず計算は割り算に直して計算して、
「逆」という表現が好きな人のまえでは「ふんふん逆ね、OK!」などと明るく振舞うのが好ましいのではないでしょうか。
727132人目の素数さん:02/05/29 12:12
723 です。最近ふと「思ひでぽろぽろ」でそういうエピソードがあったなぁ、と思い出して質問させてもらいました。数学は高校以来まったくやってないので…まだ理解しきれてないのでじっくり考えます。レスありがとでした。
728??????:02/05/29 12:18
2 2  3 1  4 3
 2    4    9
3 4  5 3  5 6
 
3 5
 ?
5 9
 
ある法則に基づいた、上の図のような組み合わせがあります。
では、?の中に入る数字は何でしょうかね?
答えがわかった方は書き込みましょう!
>>728
答え出てんのに何度も書くなよ。しかもマルチだし。
730132人目の素数さん:02/05/29 12:52
S={(x,y)|(x-1)^2+(y+1)^2<4)かつ(y+x^2>=1)}とする時
Sの面積を求めよ。
方針すら立ちませんどうすればいいでしょうか?
731132人目の素数さん:02/05/29 12:56
>>672
あぅ無視されてるTT
732132人目の素数さん:02/05/29 14:06
a/b+b/a=1
どうやって解くんでしょう?
733132人目の素数さん:02/05/29 14:15
8/(x-1)^2+4
のlim-∞が0になるんですがどうして-0にならないのかわかりません。
というか正負を判別するための計算方法がわかりません。教えてください
>>731
他に条件はないか?
a/b=xとでも置いてxの方程式にすると・・・

>>732
二乗してるぢゃん。
735132人目の素数さん:02/05/29 14:24
たして1、掛けて1、
さぁ何と何でしょう?
>>735
-ωと-ω^2
737トッピオ:02/05/29 14:27
代数の問題.
有限群Gの部分群H,Kに対して
H/(H∩K) と HK/Kに
自然な全単射が存在することを
示せ。

解答では、以下のようになっていました。

Hから、HK/Kへの写像fを f(h)=hK と定義する。...@
(fが全射であることは、明らかであり)

f(h1)=f(h2) ならば h2=h1*k をみたす k∈Kが存在する。...A
k=(h1)^(-1) * h2 ∈ H∩K であるから
h2∈h1(H∩K) である。
よってfは全射である。

僕がわからないのは、
H/(H∩K) と HK/Kに
自然な全単射が存在するとはどのようなことを
さすのかがわかりません。
とくに自然な全単射というのが、わかりずらいです。
そして、fをどうして@のように定義するのか、
Aでは、f(h1)=f(h2) ならば h2=h1*k をみたす k∈Kが存在する。
といっているがどうして、’k∈K’なのか?

まず、問題の全体像がわからないので、そこから
説明してください。おねがいします。
738132人目の素数さん:02/05/29 14:32
y=4/x+4/x^2 (lim→-∞)は-0になるんですが違いが分からない
739132人目の素数さん:02/05/29 14:54
Q.>>730を正しい問題文に直せ
>739
???
741132人目の素数さん:02/05/29 15:00
738お願い・・・
>>738
4/x+4/x^2=4(x+1)/x^2だな。
分母は正だが分子は負だ。これが0に近づくってぇんで-0。
743132人目の素数さん:02/05/29 15:07
じゃあ
y=-1/x(lim→∞)は-0になるんですか?
>>743
だな。
745132人目の素数さん:02/05/29 15:10
極限分野では符号ついてなかったけどな・・・
746132人目の素数さん:02/05/29 15:10
742 名前:132人目の素数さん 投稿日:02/05/29 (水) 15:04

>>738
4/x+4/x^2=4(x+1)/x^2だな。
分母は正だが分子は負だ。これが0に近づくってぇんで-0。
747132人目の素数さん:02/05/29 15:11
744 名前:132人目の素数さん 投稿日:02/05/29 (水) 15:10

>>743
だな。
748132人目の素数さん:02/05/29 15:19
>742 分子が負になるっていうのがよくわかりません。
分母分子をx^2で割ると、4(1/x+1/x^2)ですけど、
4(-0+0)になりますが、これってなんで負なんですか?
>>748
1/xと1/x^2はどっちが絶対値が大きい?
750132人目の素数さん:02/05/29 15:23
1/xです
統計の問題なんですけど、
要素数が無限大の集団があり、その集団の中の一人が流行性の病気にかかる。
時刻tの時の感染者の数をX(t)と書く(時刻t=0の時、個体数は1)。
(時刻tの時に感染者がn人いる確率はp_n(t)とする、、、がとりあえず、ここでは使わない)
感染者が死亡、もしくは回復する事はないとする。
そこで感染者がはじめてK人になる時の時刻(τとする)の期待値を求めたいのですが、
その期待値の求め方が
∫{0,∞}P(τ>t|X(0)=1)dt
になるらしいんですけど、これが何故かどうしても分かりません。
具体的な数字は端折りましたけど、計算して見ると、正しい感じの答えになるんです。
一応基本的な期待値の求め方は知っていますが、それでやると答えがおかしくなるんですよ。
どなたか解説御願いします。
752132人目の素数さん:02/05/29 15:46
750です
例えば、x-2/x^2-1で漸近線がプラマイ1なんですが、
++1と-+1で+∞と-∞に分かれてしまいます。
全く意味がわかりません。教えてください
753トッピオ:02/05/29 15:54
代数の問題.
有限群Gの部分群H,Kに対して
H/(H∩K) と HK/Kに
自然な全単射が存在することを
示せ。

解答では、以下のようになっていました。

Hから、HK/Kへの写像fを f(h)=hK と定義する。...@
(fが全射であることは、明らかであり)

f(h1)=f(h2) ならば h2=h1*k をみたす k∈Kが存在する。...A
k=(h1)^(-1) * h2 ∈ H∩K であるから
h2∈h1(H∩K) である。
よってfは全射である。

僕がわからないのは、
H/(H∩K) と HK/Kに
自然な全単射が存在するとはどのようなことを
さすのかがわかりません。
とくに自然な全単射というのが、わかりずらいです。
そして、fをどうして@のように定義するのか、
Aでは、f(h1)=f(h2) ならば h2=h1*k をみたす k∈Kが存在する。
といっているがどうして、’k∈K’なのか?

まず、問題の全体像がわからないので、そこから
説明してください。おねがいします。
754132人目の素数さん:02/05/29 16:05
>753
確認ですが、Aの下の「よってfは全射である」の部分は
「よってfは単射である」が正しいのですよね。
何故 f(h) = hK とおくかといえば、そうおけばうまくいくから。
準同型定理をにらんでおけば、結局 g(h(H cap K)) = hK という
自然に定義できる写像が同型写像を与えている。証明はこれが
well-defined であること、つまり定義となっていることに帰着
する。自然というのは簡単にいえば定義式がきれいな形で与えら
れること。

756132人目の素数さん:02/05/29 16:42
質問です。1変数xについての任意のC^n級の実関数f,gに対して、
D(f+g)=Df+Dg, D(kf)=kD(f),D(fg)=(Df)g+f(Dg)
を満たす作用素Dは微分のスカラー倍しかない。
っていえますでしょうか。(kは実数)
757132人目の素数さん:02/05/29 16:53
〜略〜固有ベクトルを求めよ。
ただし固有ベクトルの大きさは1

という問題があったとき、
固有ベクトルの行列式を求め、
さらにその行列式の絶対値で割ってやれば
良いんでしょうか?
厨房質問すみません。
758132人目の素数さん:02/05/29 16:54

なんでC^n級なんだ?
759132人目の素数さん:02/05/29 16:59
複素積分の問題について質問です。
Aを0,1,1+iを順次結んで得られる折れ線、Bを0,1+iを結ぶ線分とする
次の積分を計算せよ。
(1)∫[A]zdz(2)∫[B]zdz
(3)∫[A]|z|dz(4)∫[B]|z|dz
(5)∫[A]|z||dz|(6)∫[B]|z||dz|

(1)(2)はわかったんですけど、絶対値をどうやったらいいのか
サッパリ分かりません。どなたか教えてください
暗号
42@RUカフェ
を誰か解読してください。
ヒントは目の前にあるもの。
>>751 次のように考えたらどうだろう。f(t) を時刻tに
初めて感染者が k人になる確率密度とする。F(t) = ∫[0,t]f(t)dt
はその分布関数。Prob(τ>t) は、感染者が kになるのに
t秒以上かかった確率、つまり t秒以内にk人感染してしまう
ことの余事象だから、Prob(τ>t)=1-F(t)。
これから、∫Prob(τ<t)dt = ∫(1-F(t))dt。部分積分して、
∫(1-F(t))dt = t(1-F(t))[0,∞] + ∫tF(t)dt。
したがって、これは期待値 ∫tF(t)dt となる。ただし、
lim{t→∞}t(1-F(t)) = 0 が必要。
762761:02/05/29 17:08

∫tF(t)dt は ∫t f(t)dt の誤り。スマソ。
763132人目の素数さん:02/05/29 17:12
>>756
微分の定義はなに?
普通のd/dxの意味だったら嘘だよ
それの関数倍もあるし
>>761
おお、成る程。
lim{t→∞}t(1-F(t)) = 0 も成り立っているようです。
どうも、ありがとうございました。
しっかし聞いてしまえば簡単な事なのに、なんでこうも分からないんだろう?
かなり情けない。
765132人目の素数さん:02/05/29 17:33
xが整数のとき、x≦3⇔x<4 が成り立つわけを詳しく教えてください。
これは本当に必要十分なんですか?
766132人目の素数さん:02/05/29 17:34
>>757
でんでんちやう
きょうかしょよみなおせ
こゆうべくとるにぎょうれるしきあるか?
767トッピオ:02/05/29 18:26
>754
そのとおりです
768132人目の素数さん:02/05/29 18:32
電卓で任意の整数を入力して、
ルートキーを押し続けると、
かならず1に帰着するのはなぜですか?
証明してください。おながいします。

ガイシュツですがおながいします。
小学生でも理解できるようにおながいします。
当方、文系大学生。
769132人目の素数さん:02/05/29 18:36
f`(x)=0の解が二つ出てきたときに、どちらが極大で、どちらが極小の解なのか判別する方法はありますか。
f(x)が1/3sin3x-2sin2x+sinx(0≦x≦π)
なので判別できません。
770132人目の素数さん:02/05/29 18:38
等号成立はどのようなときに書かなければならないのですか?
相加相乗のほかにありますか?
>>359 積分の形にしてあげるから、あと計算はガムバッテね。

(積分路Aについて)
z = x + iy とおいて、
(3) = ∫xdx + i∫√(1+y^2)dy
(5) = ∫xdx + ∫√(1+y^2)dy
ただし、積分範囲はすべて[0,1]。

(積分路B)
積分路上では、z = x+ix と書けるから、dz = (1+i)dx
ないし|dz| = |1+i|dx = (√2)dx。これより、
(3) = ∫|x+ix|(1+i)dx = (√2)(1+i)∫xdx
(4) = ∫|x+ix|(√2)dx = 2∫xdx.
772132人目の素数さん:02/05/29 18:48
>>768
ネタ?
773757:02/05/29 18:51
>>766

教科書見直してきました。

例えば、
λ1に属する固有ベクトルp1=(a1,a2)
λ2に属する固有ベクトルp2=(b1,b2)
と出たとすると、
「大きさを1にする」ってのは
p1に対して((a1)^2+(a2)^2)^(-1/2)をかけ,
p2に対して((b1)^2+(b2)^2)^(-1/2)をかける
という風でいいですか?
774トッピオ:02/05/29 18:51
代数の問題.
有限群Gの部分群H,Kに対して
H/(H∩K) と HK/Kに
自然な全単射が存在することを
示せ。


この問題文の意味がよくわかりません。
H/(H∩K) と HK/Kに
自然な全単射が存在するとは、
f:H/(H∩K)→ HK/K
このようなこのなのか?
とにかく、題意がよくつかめていません
わかる方、おしえてください
775132人目の素数さん:02/05/29 18:52
Xn+1=Xn^1/2
これって解けるか?
776132人目の素数さん:02/05/29 18:57
上の漸化式変形してlim(n→∞)のとき1になればいいんだろうけど
素人だからわからん
777768:02/05/29 19:08
ネタじゃないです。
マジでお願いします。
778132人目の素数さん:02/05/29 19:11
いつのまにか糞スレになりつつあるな
779132人目の素数さん:02/05/29 19:12
768 はマルチ
780132人目の素数さん:02/05/29 19:18
まず、まともな質問ができるようになってから
出直してきたまえ
781まおまお:02/05/29 19:19
>>774
やぁ、引っ張りますね。

1. Gは有限群でなければ成り立ちませんか?
2. Kは、単にGの部分群というだけで良いのですか?(もう少し強い
条件が必要でしょう?)
782132人目の素数さん:02/05/29 19:22
マルチって何?
>>768
√キーを押しつづける問題だけど、これは整数だけじゃなく、任意の
正の数(1以下の数でも)成立する。一応、下記には1 以上の数 x > 1
からはじめた場合を書くが、0<x<1の場合は自分で考えてごらん。

x = 1+h と書くと、h > 0 だ。その範囲で 1 < √(1+h) < 1+ h/2
が証明できる (2乗すれば簡単)。 x = 1+h を電卓に入れて、一度
√キーをおすと結果は √(1+h) になるわけだが、これは1以上 1+h/2
未満の数というわけだ。2度おすと、√√(1+h) だが、これは1+h/4
より小さい。以下同様に、n回押した場合、

1 < √√…n…√(1+h) < 1 + (1/2^n)h

となるが、1 + (1/2^n)h はいくらでも 1 に近づくことから、
1 とそれにはさまれた電卓の表示も 1 に近づく(収束する)こと
になる。
>>782
マルチポスト
同じ内容の書き込みを、いくつものサイト・板・スレですること。
マナー違反だと思います。
785132人目の素数さん:02/05/29 19:34
>>784
そうですか。ありがとうございます。
786768:02/05/29 19:36
>>783
ありがとうございます。
よくわかりました。
787132人目の素数さん:02/05/29 19:48
aは0<a<1を満たす定数である。数列{a(n)}を
a(1)=a,a(n)=1−[3]√(1−a(nー1))で定義するとき、
すべての自然数nについて0<a(n)<1を示せ

この問題教えてください!
帰納法だと思うんですけど、よろしくお願いします。
>>787
[3]
って何を表してるんですか?
789132人目の素数さん:02/05/29 19:54
>>788
[3]√で、三乗根です。分かりにくくしてしまってすいません。
>>787 ( >>788 … [3]√は3乗根)
b[n] = 1-a[n] とすれば、やはり 0<b[1]<1。
漸化式は、b[n+1] = (b[n])^(1/3)。
y = x^3 のグラフを 0<x<1 の範囲で考えれば
0<y<1、だから 0<b[n]<1。これじゃ、いかんの?
791790:02/05/29 20:02
↑ グラフは y = x^3 じゃなくて y = x^(1/3) だった。
ただ、逆関数という意味では y = x^3 で考えたほうが
わかりやすいかも。どっちみち区間 (0,1) の写像が
(0,1) に落ちることを言えばよい。
>>787
n=kのとき 0<a(k)<1 とする。
∴0<1-a(k)<1 
→ 0<(1-a(k))^(1/3)<1
→ 0<1-(1-a(k))^(1/3)<1
∴0<a(k+1)<1
793787:02/05/29 20:06
すいません、>>787の問題(n=2,3…)が抜けてました。

一応自分で考えたんですが↓

[T]n=2の時
a(n)=a(2)=1−[3]√(1−a)
0<a<1より0<a(n)<1が成り立つ
[U]n=kがなりたつとすると
0<a(k)<1,a(k+1)=1−[3]√(1−a(k))より
0<a(k+1)<1となり、成り立つ

…なんか違う気がする
794132人目の素数さん:02/05/29 20:15
・xが整数のとき、x≦3⇔x<4 が成り立つわけを詳しく教えてください。
これは本当に必要十分なんですか?

・等号成立はどのようなときに書かなければならないのですか?
相加相乗のほかにありますか?
>>793
どこがおかしいと思うの?
途中式をもう少し書いたほうがいいとは思います。
計算問題ではなく証明問題なので。
>>774
基本的には755に書いたとおり。準同型定理と書いたのは言いすぎ
でそのためには781指摘の正規部分群であることが必要だが、ここ
では同型写像といってるのでなく全単射ということだから,準同型
定理の証明からとでもいえばそれでよい。
>>794
x≦3
となる整数
x<4
となる整数を書き出してみてはどうでしょうか。

2つ目の質問は1つ目と関係してますか?
798787:02/05/29 20:23
>>795
>>793の、
「a(k+1)=1−[3]√(1−a(k))より 」の部分が、なんかおかしいような。
うまく説明できないんですけど、いきなりこれらをイコールとして良いのか、と。

>>795さん=>>792さんですか?
799132人目の素数さん:02/05/29 20:26
質問です。。。

「スカラー関数 f(x,y,z)=3x^2+y^2+z^2 の等ポテンシャル面群を求めよ」

という問題です。
 
>>798
>>795>>792です。
[T]か[U]かどちらかで
もう少し過程を詳しく書いてもいいんじゃないでしょうか?

過程は同じなので片方だけでもいいかなと思います。(たぶん)

問題集にある解答例としてはこれぐらいの表記しかなかったりしますが
あくまでも略してるんだと思います。
見る相手に伝わるように、考えた方がいいと思います。
>>799 質問。

i) X^2 + y^2 + z^2 = r^2 = R^2 (Rはある定数) は
どんな図形ですか?
ii) 上式の X = √3x とした、3x^2 + y^2 + z^2 = R^2
はどんな図形ですか?
>>798
たぶん、与えられた条件を使うということと、自分で何か構成
するときとの混乱があるのではないでしょうか? いきなりも
なにも与えられたことだからイコールなのです。それは別な何か
から証明されることではありません。
>>802
ああ、そういうことか。
別のことに違和感感じてるんだと思ってた。
>>798
ひょっとして n について与えられた条件を k で使っているのが
変だってことじゃないの?
805787:02/05/29 20:40
>>800
分かりました〜。
[T]、[U]共に過程を詳しく書くことにします。

あと、証明の時点で、a(k+1)→1−[3]√(1−a(k))っていうのは言えるんですか?
1−[3]√(1−a(k))→a(k+1)は言えますよね?
806799:02/05/29 20:41
>>801
i)半径Rの球ですよね?
ii)・・・うーん・・・・
807132人目の素数さん:02/05/29 20:42
>>735
(1+√-3)/2と(1-√-3)/2
808801:02/05/29 20:44
>>806 「うーん」じゃ、ちょっと困る ^^;
要するに、球を x方向に 1/√3 に圧縮しただけのもの。
回転楕円体という。等ポテンシャル面はその表面。
>>805
その→ってなあに? = とどう違うの?
与えられた条件はすべての自然数について成り立つってことです。
だから、こころおきなくいつでも使っていいのです。→なんて
書かないで。
>>803
段々、いきなり、の意味がわかってきた。
だいたい802でいいところをついてたみたいだな。
a(k) が決ってから a(k+1) が決まるように思ってるんだ。

>>805 さんね、809にあるように自然数 k すべてについて条件式
は成立しているので、帰納的におずおず決まるんじゃないのです。
811799:02/05/29 20:55
>>808
ありがとうです〜
812787:02/05/29 20:57
>>809
すいません、かなり混乱してました。いつの間にか頭の中では
数列{a(n)}の証明をしようとしてました。

>与えられた条件はすべての自然数について成り立つってことです
>こころおきなくいつでも使っていいのです
 ↑この文章に癒されました。やっと理解できました。
お手数かけました。本当にありがとうございました。
813787:02/05/29 21:02
>>810
>a(k) が決ってから a(k+1) が決まるように思ってるんだ。
まさにその通りです。けどやっと分かりました。
皆さん本当にありがとうございました。

814れい:02/05/29 21:12
質問1.
lim[n→∞](log(n)/(n^ε))=0
をロピタルの定理で証明したいのですが
どうしたらできるかおしえてください。

質問2.
lim[n→∞] {f(n)/g(n)} が有限な極限のとき
f(n) < C*g(n) となる定数Cが存在する。
これを証明したいのですが、
わかるひとがいたらおしえてください。
815132人目の素数さん:02/05/29 21:16
内分点の公式がなぜこうなるのかわかりません。教えてください。
例) ABを3:1に分ける点、(A+3B)/4
これって比と係数が逆になってますよね?なぜですか?
816132人目の素数さん:02/05/29 21:19
>>797
お返事ありがとうございます!書き出してみたらわかりました。
一つ目の質問と二つ目の質問は関係してないです。

>>815 なぜって、あんた、「ABを3:1に分ける点」はどちらかと
いうと B に近い場所なわけだろ。だから (A+3B)/4 という式つくって、
B の影響を強く反映させるわけさ。
>>817
>>815じゃないけどわかりやすい説明だな〜。
>815
ABを3:1に分ける点をPとすると、、
AP:PB = 3:1
になるよな。ということは、
AP:PB:AB = 3:1:4
になるよな。
ということは、
AP↑=(3/4)AB↑=(3/4)OB↑−(3/4)OA↑
だよな。よって、
OP↑=OA↑+AP↑=OA↑+(3/4)OB↑−(3/4)OA↑
    =(3/4)OB↑+(1/4)OA↑
と書いたところで、>817を見ると、ウツになる、と。
972 :132人目の素数さん :02/05/28 16:50
   1 2 3
   -------
e   1 2 3
σ1  2 1 3
σ2  1 3 2
σ3  3 2 1
σ4  2 3 1
σ5  3 1 2

(ここまで前のスレのコピペです)
でσ2*σ5やる場合は

σ2(132)をσ5と同じに動かす→321=σ3
でいいの?
>>819
いや
>>817 >>819
2つあったほうがイイと思いますよ。
感覚的なことと、それに対する理屈と言うことで。
822132人目の素数さん:02/05/29 21:36
x=1/√3-1が2次方程式2x^2+ax+b=0(ただし、a,bは有理数)の解であるとき、
a=(),b=()である。
このとき、4x^3-10x^2+8x+5=()+()√()である。

という問題なんですが、わかりません・・・(汗)
親切な方、ヒントお願いします(>_<)
823815:02/05/29 21:42
>>817
ちょっと私にはわかりにくかったです。818さんはわかりやすかった
みたいですが・・・。819さんの理屈編は納得できました。
感覚編のほうも納得したいのですが。
「B の影響を強く反映させるわけさ」のところがちょっとわかりません。
>>822
x=1/√3-1・・・1)
より
(√3-1)x=1
→ (√3)x=x+1 両辺二乗して整理
x^2=(2x+1)/2・・・2)
あとは1)2)を
2x^2+ax+b=0
に代入してa・bを求める
同様に1)2)を
4x^3-10x^2+8x+5
に代入して整理すれば求まります。
等号成立はどのようなときに書かなければならないのですか?
相加相乗のほかにありますか?
>>823
感覚的に納得できなかったら
理屈だけでいいんじゃないかな?
>>825
問題の必要性による
としか言えないです。
スマソ。
828823:02/05/29 21:50
>>826
感覚的に納得できなかったらすぐ使えないじゃないですか。
他の説明はないのでしょうか。
829132人目の素数さん:02/05/29 21:51
(tanA + tanB)/2 ≧ tan(A+B)/2・・・@ を使って、次のことを示せ。
(1)(tanA+tanB+tanC+tanD)/4 ≧ tan(A+B+C+D)/4
(2)(tanA+tanB+tanC)/3 ≧ tan(A+B+C)/3
という問題なのですが、(1)は@を
繰り返し使えばいいと思うのですが、(2)はどうすればよいのですか?
830132人目の素数さん:02/05/29 21:53
自分で考えて感覚的に理解できないのなら、それまでの頭だと思いたまえ
>>828
いや、>>819だけでも充分な解答ですよ。
>>817>>はわかりやすい説明だと思うけど
わからなかったら機械的に理解しておけばいいんじゃないですか?
832132人目の素数さん:02/05/29 21:57
>>829
(tanA + tanB)/2 ≧ tan(A+B)/2・・・@
が無条件で成り立つとは思えないけど
あと、括弧の使い方がおかしいよ
833132人目の素数さん:02/05/29 22:20
>829
(1) に D=(A+B+C)/3 を代入して整理。
834132人目の素数さん:02/05/29 22:21
xの整式f(x)をx^2-4x+aで割ったときの商と余りは同じであり、
f(x)をx^3で割ったときの余りは-24x+32である。
このとき、f(x)は(ア)次式であり、a=(イ)である。
また、f(x)を因数分解すると、f(x)=(ウ)(x+(エ))(x-(オ))^(カ)となる。


という数Aの数と式の問題なんですが文系数学ドキュソなので
わかりません(汗
ヒントを頂ければ嬉しく思いますm(__)m
>823
たとえば、線分ABの長さを1メートルとして、
ABを1億対1に内分する点Pを考えるとしよう。
点AとPの間は、1ミリの10万分の1しか離れていないことになる。
肉眼では点Aと点Pは重なっていて区別できない状態にある、と言ってもいいだろう。

よって、点Pをベクトル表示すると
OP↑≒OA↑
と考えても、ほとんど問題ないだろう。
ゲンミツには等しくないわけだけどね。

直線AB上の点Pは
 OP↑=αOA↑+βOB (ただし、α+β=1)
と表すことができる。
今、OP↑はほとんどOA↑と等しいので、
α≒1、β≒0
だ。
Aに近ければ近いほど、Aのベクトル成分をより多く含むことになる。
AとBの中点では、Aの影響とBの影響が等しいので
α=β=1/2
になる。
Bにものすごく近い点では、
α≒0、β≒1
だ。
>>834
xの整式f(x)をx^2-4x+aで割ったときの商と余りは同じであり、
  ↓
f(x)=(x^2-4x+a)(bx+c)+(bx+c)   (b・cは定数)
とおいて他の条件からといてみてください。
837132人目の素数さん:02/05/29 22:28
>822
前半だけだけれど、別解
まず分母を有理化しておいて、
有理係数の方程式なら x=p+q√3が解ならp−q√3も解
解と係数の関係を使う

さらに別解
(x−p)^2=(q√3)^2
これを整理する
838132人目の素数さん:02/05/29 22:32
>828
内分点を支点にしたシーソーを考えます。このシーソーを
つりあいが取れるようにするには、どちらを何倍すればいいでしょう。
内分点とシーソーの関係は、と聞かれたら 無いです。
>>838
既に公式の覚え方
になってるね。
俺もよくどっちだっけ?って思うから(カッコ悪い)
>>817はいい解答だな〜と思ったんだけど。
840132人目の素数さん:02/05/29 22:46
>>817 のような考え方は普通じゃないのか?
じゃないと暗記になっちゃうだろ
俺いつも公式覚えてなかった。
テストのとき、公式もとめることからやらないといけないこともあった。
三角関数とかは辛かった。
雑談スマソ
>>838
けっこう関係あると思うよ、内分点とシーソー。
重み付きの平均(モーメント)という意味で。

図形の重心を求める式や、
積分∫f(x)dx の f(x)dx の部分なんかも、
重み付き平均の考え方で理解できる。
843132人目の素数さん:02/05/29 23:05
824、837さん。ありがとうございます。
マジで助かりました〜
>>820
その置換は右作用?それとも左作用?
それによって答えが変わる。

どちらにしろ、あみだくじでも使って
考えてみるといいよ。
845575:02/05/29 23:16
ワーイ解けました!ヽ(´ー`)ノ
がんばった甲斐がありました。
相談になってくれた方、ありがとう。
846575:02/05/29 23:16
相談になって→相談に乗って
>>844
わかりません。。。(右作用、左作用って言葉事態初めて聞きました)
演算表(群表)っての埋めたいんですが。。。
848828:02/05/29 23:38
>>838
お返事どうも。なぜ「つりあいが取れるよう」にしないといけないのですか?
すいません、ちょっとわかりません。

>>840
817さんののような考え方が理解できるんですね。あのぉ〜内分点の公式をちょっと
違う言葉で説明してもらえないでしょうか?

>>841
内分点の公式はどうやって理解されていたのですか?
849132人目の素数さん:02/05/29 23:40
>>692は解けないもんなんですか?
数学じゃないですけど暗号問題。
42@RUカフェ
これを解読してください。
ヒントはキーボード
0≦A≦90, 0≦B≦90において、
(tanA + tanB)/2 ≧ tan(A+B)/2・・・@ を使って、次のことを示せ。
(1)(tanA+tanB+tanC+tanD)/4 ≧ tan(A+B+C+D)/4
(2)(tanA+tanB+tanC)/3 ≧ tan(A+B+C)/3
という問題なのですが、(1)は@を繰り返し使えばいいと思うのですが、
(2)はどうすればよいのですか?第一手目について方針をお願いします。
852132人目の素数さん:02/05/29 23:47
>851
>833
ちゃんと読めよ。
853おはつ:02/05/29 23:51
だれか〜!この問題といて〜!
(x+1)/(x^4+x^2)の不定積分
たのみます〜!むつかし〜!
代数系を勉強してるんですがどうもつかみづらい話ばっかでとまどってます。
あと学校では「群」から始めて、漸化式や関数などにあてはめる計算ばかり
やってるのですが、きちんと体系的に勉強する場合、体→群→束としたほうが
いいんでしょうか?大体の教科書はこの順ですが。この順番はなにか理由があるんですか?
あと、代数系でわかったかなきゃいけない一番の骨ってなんですか?
>849
ロピタルの定理を使う
856132人目の素数さん:02/05/29 23:52
>>852
あぁどうも申し訳ない。832だけ見てました。ところで、疑問なのですが、
(1)を使うためにD=(A+B+C)/3 を代入するんですよね?
それと、勝手にDを決めてしまって良いのですか?(A+B+C)/3じゃないかも
しれませんよね。
>853
1/(x^4+x^2) = A/x^2 + B/(x^2+1)

となる定数A,Bが存在することに注意。
>>850は誰にも分かりませんか??
>>847
たとえば、σ1*σ2*σ3を [a,b,c] に作用させる場合、
σ1から順に施すのか、σ3から順に施すのかによって
結果が変わってくる。

前者を左結合といい、[a,b,c]σ1*σ2*σ3 と書く。
後者を右結合といい、σ1*σ2*σ3[a,b,c] と書く。

要するに、σ1*σ2*σ3と書かれていた場合、
(σ1*σ2)*σ3と計算するかσ1*(σ2*σ3)と
計算するかの違い。
860132人目の素数さん:02/05/29 23:59
a,b,cは定数でf(x)=2x^3+7x^2+9x+a,g(x)=bx^2+cx+3である。
(1)f(x)=(x+k)・g(x)(kは定数)が全てのxについて成り立つとき、
k=(ア)またはk=(イ)/(ウ)である。
(2)整式h(x)はh(0)=1を満たす整式で、全てのxについて
f(x)-g(x)=(x-2a)・h(x)が成り立っている。
このとき、a=(エ)で、c=(オカ)b+(キク)である。
さらに、f(x)をh(x)で割った余りがg(x)のとき、
h(x)=(ケ)x^2+(コサ)x+(シ)である。

この問題がわかりません(ーー;)
誰か親切な人、ヒントをお与え下さいm(__)m
861854:02/05/30 00:00
うーん、雑談スレにしとけばよかったか
841>>848
教科書に公式を導く過程が書かれてるでしょ。
テストの前の日は、それを見直してました。
あなたに対するレスはもう充分だと思いますよ。
>>819が理解できるんですよね?
それで充分です。
863おはつ:02/05/30 00:18
すごいヒントをありがとうございました。それにしてもはやかったねえ。
>>692>>849
マクローリン展開を利用する。
865132人目の素数さん:02/05/30 02:07
>860(1)
f(x)=(x+k)・g(x)と云うことは因数分解できると云ういみ。
・最高次係数は同じ(b=2)
・(2x^3+7x^2+9x+a)=(k+3)(bx^2+cx+3)
の2条件から2k+c=7,ck=6,a=3k
c消去でkの2次方程式がでる。
866132人目の素数さん:02/05/30 02:13
>860(2)
h(0)=1より今度は定数項に注目。x=0を代入
f(0)-g(0)=(0-2a)・h(0)よりa=1
次に(x-2a)の因子に注目してx=2a=2を代入
(2*2^3+7*2^2+9*2+a)-(b*2^2+c*2+3)=0より残りが出る
867859:02/05/30 02:14
すまん間違えた。
>>859の下3行は間違っているので忘れてくれ。
>854
普通は群やって環やって体
体は環で、環は群なのだから、群の定理は環や体でも使えるし
環の定理は体でも使えるからこの順でやる

>あと、代数系でわかったかなきゃいけない一番の骨ってなんですか?

↓これ一冊くらいの内容は把握しましょう。
http://shopping.yahoo.co.jp/shop?d=jb&id=04986612
869859:02/05/30 02:46
>>820
σ2というのは、2番目と3番目を入れ換える操作だから、
これを[a,b,c]に作用させると[a,c,b]になる。
つまりσ2[a,b,c] = [a,c,b]

またσ5というのは、1番目を2番目に、2番目を3番目に、
3番目を1番目に移す操作だから、これを[a,b,c]に作用させると
[c,a,b]になる。つまりσ5[a,b,c] = [c,a,b]

これらをアミダくじで表すと、
 σ2     σ5
a b c   a b c
┃┃┃  ┃┣┫
┃┣┫  ┣┫┃
┃┃┃  ┃┃┃
a c b   c a b
870859:02/05/30 02:54
(続き)

この図から、σ5というのは、[a,b,c]にまずσ2を作用させ、
続いてσ1を作用させる操作と等しいことがわかる。この様子を
σ5[a,b,c]=σ1*σ2[a,b,c]=[c,a,b] と表す。

これらのことから、σというのは「操作」を表していことがわかると思う。
つまり操作の対象となる相手は[a,b,c]でも[あ,い,う]でも何でもいいのだ。

そこで、「操作σ5」と、「操作σ2のあと、操作σ2」が等しい、というように
作用の対象を無視して操作だけに着目し、

σ5=σ1*σ2

と表すことにするわけだ。
これはσ2とσ1のアミダくじをたてにつないだ物が、
σ5のそれと等しいことを意味している。

では、σ2*σ5 はどうなるだろうか?(終わり)
>>799
で?
872解いてください:02/05/30 04:17
x^5+6x^4+13x^3+24x^2+12x+18=0

この方程式の実数解を求めてください。

>>872
x=-2+(-10)^(1/3)
以外にある?
>>873
どうやって解いたの?
875132人目の素数さん:02/05/30 06:03
掲示板だからしょうがないけど (-10)^(1/3) という表記は
少なくとも高校ではタブー
876132人目の素数さん:02/05/30 07:55
x^5+6x^4+13x^3+24x^2+12x+18=(x^2 + 1)(x^3 + 6x^2 + 12x + 18)
877132人目の素数さん:02/05/30 08:30
虚数はダメだろ?
878132人目の素数さん:02/05/30 09:32
-(10)^(1/3)だったらいい
…というわけでもないにょ?

>>877
三乗してマイナスになる実数は…
879132人目の素数さん:02/05/30 09:40
756質問の再掲です。。1変数xについての任意のC^n級の実関数f,gに対して、
D(f+g)=Df+Dg, D(kf)=kD(f),D(fg)=(Df)g+f(Dg)
を満たす作用素Dは微分のスカラー倍しかない。
っていえますでしょうか。(kは実数)

>758
>なんでC^n級なんだ?

C^Infinityで結構です。

>763
>微分の定義はなに?
>普通のd/dxの意味だったら嘘だよ
>それの関数倍もあるし

d/dxです。
すいません、詳しく教えてください。
880高校万歳:02/05/30 09:52
ベクトルなんですが、
|vector(AP)|^2-4/3(vector(AM),vector(AP))=0
ゆえに
|vector(AP)-2/3vector(AM)|^2=|2/3vector(AM)|^2
にどうやってなるのかわかりません。誰か教えてください。
881132人目の素数さん:02/05/30 09:57
(P-(2/3)M,P-(2/3)M)
=(P,P)-(P,(2/3)M)-((2/3)M,P)-(2/3)M,2/3)M)
=((2/3)M,(2/3)M)
882132人目の素数さん:02/05/30 10:02
(P-(2/3)M,P-(2/3)M)
=(P,P)-(P,(2/3)M)-((2/3)M,P)+((2/3)M,(2/3)M)
=((2/3)M,(2/3)M)
883132人目の素数さん:02/05/30 10:05
>>879

>>763 がいいたいのは
D=a(x)(d/dx) という作用素という意味ぢゃないの
>>859
ありがとうございます。

σ1 213
1→2
2→1
3→3
σ2 132
1→1
2→3
3→2
σ5 312
1→3
2→1
3→2

σ2*σ5は
操作σ2をして、132にして、操作σ5をして321にしてσ3(321)ですか?
885高校万歳:02/05/30 10:08
すみません。もう少し詳しく説明してくれませんか?
886ホ?ヨ?チiLSミ?ユ?チj:02/05/30 10:18
次のLS積分でその解が、1/2^{n-1}となるF(x)を探しています。
どうか、お知恵をお貸しください。

∫_0^1 x^s (1-x)^{n-s-1} dF(x)
887ぽぽんた:02/05/30 10:23
下の問題が分かりません。
連続するmこの整数のうち、ひとつがmで割り切れることを証明せよ。
だれか、教えてください。
888ホ?ヨ?チiLSミ?ユ?チj:02/05/30 10:29
886に追加です。
F(x)は、[0,1]上の確率測度です。
>>880
実数p,mに対して
p^2-2mp=0 ⇔ (p-m)^2=m^2
のように平方完成しただけ。
890高校万歳:02/05/30 10:35
880の答えだと思うんですが

(P-(2/3)M,P-(2/3)M)
=(P,P)-(P,(2/3)M)-((2/3)M,P)-(2/3)M,2/3)M)
=((2/3)M,(2/3)M)

をみてもさっぱりわかりません。もう少し詳しく教えてください。
自分まだベクトルの初心者なので
891高校万歳:02/05/30 10:40
>889

なるほど。なんとなくわかりましたが、(P,M) みたいに
かっこがついた場合のベクトルの計算のしかたがわからないのです。
どう言う意味なのかも・・・。
>>887
あきらか。
背理法でもよし
893132人目の素数さん:02/05/30 10:50
(a,b)は(実)ユークリッド内積:
a=(x1,...,x_n),b=(y1,...,y_n)ならば(a,b)=x1y1+...+x_ny_n
(a+b,c)=(a,c)+(b,c)
(λa,b)=λ(a,b)=(a,λb)(λは実数)
(a,b)=(b,a)
といった性質がある(確かめてみろ)
894886>886:02/05/30 11:32
お騒がせしました、
x=0.5に関するディラック測度でいいんですよね。
895132人目の素数さん:02/05/30 12:12
0≦A≦90, 0≦B≦90において、
(tanA + tanB)/2 ≧ tan(A+B)/2・・・@ を使って、次のことを示せ。
(1)(tanA+tanB+tanC+tanD)/4 ≧ tan(A+B+C+D)/4
(2)(tanA+tanB+tanC)/3 ≧ tan(A+B+C)/3
という問題なのですが、(1)は@を繰り返し使えばいいと思うのですが、
(2)は(1)を使うためにD=(A+B+C)/3 を代入すると思うんですが、
勝手にDを決めてしまって良いのですか?
(A+B+C)/3じゃないかもしれないですよね。
896132人目の素数さん:02/05/30 12:22
複素積分の問題について質問です。
Aを0,1,1+iを順次結んで得られる折れ線、Bを0,1+iを結ぶ線分とする
次の積分を計算せよ。
(1)∫[A]zdz(2)∫[B]zdz
(3)∫[A]|z|dz(4)∫[B]|z|dz
(5)∫[A]|z||dz|(6)∫[B]|z||dz|

(1)(2)はわかったんですけど、絶対値をどうやったらいいのか
サッパリ分かりません。どなたか教えてください
>>896

この問題は >>771 で回答ずみ。ちゃんと調べろ。
898132人目の素数さん:02/05/30 12:46
>>895
↓この続きか
>>829,>>832,>>833,>>851,>>852,>>856

(1)はどんなA,B,C,Dについて成り立つ不等式だったの?
899132人目の素数さん:02/05/30 12:51
y=sinx(0≦x≦π)をx軸回りに回転させ、
それをさらにy軸回りに回転させた体積を求めよ。

y軸回りに回転させる段階でつまずいています。
V=π∫[b,a](f(y)^2)dyの公式を使うと、
sinx^(-1)が出てくると思うんですが、
それをどう積分したらいいのか...
お願いします。
>>899
それは、y=±sin xで囲まれる「平面図形」を
y軸で回転させる場合じゃないのか?

立体図形を回転させる場合は、そういう公式を
単純に適用するわけにはいかない。
2重3重に通過される部分が出てくる可能性が
あるからね。

地道にスライスして考える以外ないんじゃなかろうか。
901教えてください:02/05/30 13:17
nCk+nCk-1=n+1Ck
の証明の仕方なんですが、
この証明方法って、
n=1の時を成り立つことを示して
n=mが成り立つとき
n=m+1の時成り立つことを示すという方針だと思うんですけど
自分でやってもシックリきません。
誰か教えてください。
>>899
>それは、y=±sin xで囲まれる「平面図形」を
>y軸で回転させる場合じゃないのか?

そのとおりだと思います。
y=0、x=π/2で切って4倍するとしても、どうすれば良いんでしょうか?
>>901
(n+1)人の中からk人選ぶとき
特定のAさんに注目して
Aさんが含まれる選び方+Aさんが含まれない選び方=k人の選び方
904132人目の素数さん:02/05/30 13:31
>>901

帰納法を使わなくても n+1Ck の意味から出てくるよ: n+1 個のうち
一つだけ固定しておく。それをAと呼ぶ。n+1 個から k 個とってできる
組み合わせは、Aを含まないか含むかのどちらかである。前者の場合はAを除いた
n個のうちからk個とる場合(nCk)で、後者の場合は、Aはすでにとるのだから、
k個のうち、残りのkー1を、A以外のものであるn個からとる組み合わせ
の数(nCk-1)である。したがってnCk+nCk-1=n+1Ck 。(証明おわり)
905132人目の素数さん:02/05/30 13:31
 お願いします。空間ベクトルの問題です。
「点A(1,-2,3)から、直線l:(x,y,z)=(2,-1,-3)+t(1,2,2)(tはパラメーター)
に、垂線を引く。この2直線の交点Pの座標を求めよ」
 という問題なのですが、
 交点を、(2+t,-1+2t,-3+2t)とおいて、APと直線lの内積が、0と考えたんだけど、
うまく答えが出ません。教えて下さい。
>>899 必要になる積分だが、(arcsin(x))' = 1/√(1-x^2)
を知っていれば ∫arcsin(x)dx = √(1-x^2) + x arcsin(x)
は部分積分ですぐ出るはず。

問題は ∫(arcsin(x))^2 dx
     = -2x + 2√(1-x^2) arcsin(x) + x(arcsin(x))^2
だが…。
907132人目の素数さん:02/05/30 13:33
winner=903
908901:02/05/30 13:35
>>903,904
ありがとうございます
しっかり復習します
>>905
それでよい。計算ミスってるだけと思われ。
910899:02/05/30 13:37
>>906
ありがとうございます!!
>>902
平面 y=k でスライスするか、
またはy軸を中心とする円筒でスライスするか。

後者の方が見込みありそうか。

立体図形を回転させる求積問題は、極めて困難だよ。
平面図形の回転のように単純には行かないことは、
次の例からもわかる。

4点(1,1) (1.-1) (3,1) (3,-1)からなる正方形を、
x軸回りに回転の後、y軸回りに回転。

この立体が、単に正方形をy軸回りに回転させた
ものとは異なることは明らかだろう。
912132人目の素数さん:02/05/30 13:43
>>907

全部書いただけ遅れたようだ。負け惜しみ。
913132人目の素数さん:02/05/30 13:45
  >>909
 交点に使うPも、直線lとして使うベクトルも、、(2+t,-1+2t,-3+2t)
でいいんでしょうか?
>>901 帰納法の必要なし。nCm + nCm-1 を定義どおりに書いて、
分母を通分して加えてごらん。一発で出る。
>>913
なるほどそこがあいまいなのか。
APベクトルと垂直なのは直線lの『方向』ベクトル。

APベクトル=(2+t,-1+2t,-3+2t) - (1,-2,3)
直線lの方向ベクトル=(1,2,2)
916911:02/05/30 13:51
ちなみに、y=±sin xで囲まれる平面図形をy軸で回転させた
立体の体積なら、逆正弦なんか使わなくても出る。

半径xの円筒で薄切りすることを考えればいいから、

V = ∫[0,π] (2πx)・(sin x) dx = 4π^2
>>912
たまにはπさんでもいいのに
いつもAだな(w
918132人目の素数さん:02/05/30 13:58
 >>915  分かりました。 ありがとうございます。
   でも、なんだかしっくりこないなあ・・。
   直線を表す式のはずなのに・・・。
>>918
いま問題にしているベクトルは
位置(座標)ではなく方向。そのうち慣れる。
そろそろ新スレの準備をしようと思うんだけど
立てていいかな?
921899:02/05/30 14:05
すみません。
どうも問題を読み直すと、
「x軸で回転→それをさらにy軸で回転」なようなのです。
ほんとにすみませんがどうかお願いします。
922132人目の素数さん:02/05/30 14:05
>>919
なんにしろ、ありがとうございます。 がんばって早くなれます!!
923132人目の素数さん:02/05/30 14:08
方向と向きって今いちわからないです。
>>918
座標で攻めればしっくりくるのか?

Aを通って直線lに垂直な平面πの式を考えると
lの方向ベクトルがπの法線ベクトルになるから
平面π:1(x-1)+2(y+2)+2(z-3)=0
これと
直線l:(x,y,z)=(2,-1,-3)+t(1,2,2)
を連立してtを求めれば交点P(x,y,z)が出る。
925895:02/05/30 14:17
>>898
↓この続きか
>>829,>>832,>>833,>>851,>>852,>>856

はい、そうです。この問題は先生が黒板にちらっと書いた物なので、
はっきりと書かれていないのですが、範囲は0≦A≦90, 0≦B≦90, 0≦C≦90,
0≦D≦90 に関する不等式だったと思います。
926132人目の素数さん:02/05/30 15:53
(x-a)(x-b)(x-c)の微分
の方法教えて
927132人目の素数さん:02/05/30 15:57
(fgh)'=f'gh+fg'h+fgh'
928132人目の素数さん:02/05/30 15:59
ありがとう!
それ知らなかった
929132人目の素数さん:02/05/30 16:00
>>926
(x-a)(x-b)(x-c)=x~3+(-a-b-c)x~2+(ab+bc+ca)x-abc
xで微分すると
3x~2+2(-a-b-c)x+(ab+bc+ca)となります。
930929:02/05/30 16:01
ををを
私のタコっぷりが発揮されてしまった(泣)
931132人目の素数さん:02/05/30 16:18
ある曲線の極大値の接点を必ず通る直線があるとして、
その直線のいずれかが、極小値を越えない範囲内で、必ず曲線との交点
を持ち、交点から極大値の範囲内では、曲線の値>直線の値である
という命題は正しいですか?
>>931
>ある曲線の極大値の接点

ここですでに意味不明。
全体的にもチンプンカンプン
>>895
D=(A+B+C)/3 とは限らないけど
D=(A+B+C)/3 のときも(1)は成り立つんじゃないの?
934132人目の素数さん:02/05/30 16:35
3次関数の極大値の接線とその3次関数との交点という意味です。
935132人目の素数さん:02/05/30 16:36
個数の処理の問題で質問です

6色の玉が各一個ずつある。次の各場合について、玉の分け方が何通りあるかを調べよ。

なお2つの群に分けるとは、個数0と6のようなわけ方は除くということ

(8)
6球をA,B二つの群に分ける
(9)
6球を二つの群に分ける

(8)はすぐわかります
2^6−2=62
です
ただ(9)がよく分かりません
自分は
62/2!=31
だと思うのですが
友人は
41といってました
どっちでしょうか?
>>931
全部書き直せ。
937132人目の素数さん:02/05/30 17:04
f(x)=x^3 - ax^2 - bx - 1とする
(1)a≧1, b≧1のとき、方程式f(x)=0の正の実解はただひとつであり、
しかもその実解は1より大きいことを示せ。
(2)a≧1a=bのとき、方程式f(x)が相異なる3実解をもつようなaの例をあげよ。

どういう方針で解けばよいのかわかりません。答えだけでなくその考え方を
体系的に教えてください。お願いします。

再びお願いします

0≦A≦90, 0≦B≦90において、
(tanA + tanB)/2 ≧ tan(A+B)/2・・・@ を使って、次のことを示せ。
(1)(tanA+tanB+tanC+tanD)/4 ≧ tan(A+B+C+D)/4
(2)(tanA+tanB+tanC)/3 ≧ tan(A+B+C)/3
という問題なのですが、(1)は@を繰り返し使えばいいと思うのですが、
(2)は(1)を使うためにD=(A+B+C)/3 を代入すると思うんですが、
勝手にDを決めてしまって良いのですか?
(A+B+C)/3じゃないかもしれないですよね。
939132人目の素数さん:02/05/30 17:07
>>937
y=f(x)のグラフを書くことを考えてみよう。
fはxの3次整式だから、極大極小での様子が分かれば、
設問に答えるに十分な情報が得られるはずだよ。
940数字神:02/05/30 17:09
>>938
レスしたぞ
>>933
942132人目の素数さん:02/05/30 17:10
>>935
友人になぜ41なのか聞いてみるべし。
943934:02/05/30 17:18
書き直します。まず正の3次関数F(x)をイメージしてください。きょくしょう
3次関数F(x)の極大値の接線とその3次関数F(x)との交点Xを通る直線があるとします。この直線をf(x)=bxとします。
さらに、その交点Xを通る3次関数F(x)の接線があります。
極大値の接線の傾きをa、直線の傾きb、Xを通る3次関数F(x)の接線をcとします。
で、bを変数として、その範囲をa>b>c(不等号逆かも?とにかく傾きbがaとcの間にある)として、
その範囲内では、直線y=bxが必ず3次関数との交点Yを持ち、
極大値のx座標の値<x<交点Yのx座標の値では必ず、F(x)-f(x)>0
である。
で、交点Xを通る3次関数F(x)の接線とF(x)の交点から右のグラフはないということにしてください。
よって交点は一つしかありません。
944938:02/05/30 17:18
>>941
どうもすみません。見落としてました。
>D=(A+B+C)/3 のときも(1)は成り立つんじゃないの?
これはどういうことでしょうか?Dを(A+B+C)/3になるように取ると
題意が成立するという意味ですか?Dが(A+B+C)/3以外の値をとるとき
にも成立することを証明しなきゃいけないと思うんですけど。 
>>944
(1)が成り立つときのA〜Dの範囲は?
946934:02/05/30 17:21
申し訳ないんですが、f(x)=abじゃなくて+nを追加してください
947934:02/05/30 17:21
nは変数です
948937:02/05/30 17:23
>>939
fを微分してみると、3x - 2ax - b になりますよね。
ここから、極大極小を求めるためにどうするんですか?
解の公式を使うのでしょうか?汚くなりますよね?
>>944
基本的なことをいうと
(1)→(2)でしょ。
あなたの考え方は
(2)→(1)
950944:02/05/30 17:24
>>945
範囲は0≦A≦90, 0≦B≦90, 0≦C≦90,
0≦D≦90 に関する不等式だったと思います。
そろそろ次スレ立てるね
952132人目の素数さん:02/05/30 17:27
>>950
(1)(tanA+tanB+tanC+tanD)/4 ≧ tan(A+B+C+D)/4

0≦A≦90, 0≦B≦90, 0≦C≦90,0≦D≦90
のみたすA〜Dなら成り立つんでしょ。
0°≦(A+B+C)/3≦90°
だから・・・
>>951
御願いします
ありがとう
955934:02/05/30 17:33
あと意味がわからなかったら、どこがわからないか言ってください。
>>943
なんか複雑かつ曖昧でいまいちわかんないんだけど、
とりあえず、

極 大 値 に お け る 接 線 の 傾 き は 0 だ
957934:02/05/30 17:38
そうでした。ええと傾きが0>b>cということでお願いします。
新スレが立ちましたので現在の話題が終了したら
移動をお願いします
◆ わからない問題はここに書いてね 34◆
http://science.2ch.net/test/read.cgi/math/1022747441/
>>943
正の3次関数F(x) って何?
極大値の接線の傾きをa=0 でしょ?
極大値の接線=Xを通る3次関数F(x)の接線 でしょ?
>>958
お疲れ様

質問者がレスしないと暇
>>959
極大値の接線=Xを通る3次関数F(x)の接線 でしょ?

これ間違ってたね
スマソ
962950:02/05/30 17:53
>>953
ありがとう!やっとわかりました!うれしいです。
963934:02/05/30 17:55
>>正の3次関数F(x) って何?
F(x)をax^3+b^2+cとするとaが正ってことです。
>>極大値の接線の傾きをa=0 でしょ?
そうです
>>極大値の接線=Xを通る3次関数F(x)の接線 でしょ?
ちょっと違います・・・
ええと、F'(x)に極大値X(c,dとでもおく。当然これは定数です)を代入した式で、それは二本でますが、極大値の接線ではなく
もう一本の線のことを言っております
>>939
誰かお願いします。
fを微分してみると、3x - 2ax - b になりますよね。
ここから、極大極小を求めるためにどうするんですか?
解の公式を使うのでしょうか?汚くなりますよね?
見通しをお願いします。
965934:02/05/30 18:00
代入じゃなかった。ええと、y-d=F'(X)(x-c)でいいのかな?
966934:02/05/30 18:06
極大値を通る接線は2本ありますが、極大値における接線じゃない方ってことです。
965も違ってます。すいません。
>>964
f'(x)=3x^2 - 2ax - b
f’(x)=0の判別式をDとし、実数解があるときα,βとする。(α<β)

f(x)=0が実数解を1つしか持たないとき、次の3つが考えられる
1
f(x)は単調増加→D≦0
2
極大値<0→f(α)<0
3
極小値>0→f(β)>0
>>964
>>967です。
確か2・3はまとめて
f(α)・f(β)>0
でよかったと思います。
つまり極小値・極大値が同じ符号なので
掛け合わせると+になる。
>>966
まとめなおして次スレで再依頼したら。
正直読む気しないです。
970934:02/05/30 18:34
そうします
971964:02/05/30 18:37
>>96
「正」の実解がひとつですよ。問題文の条件を読み違えてませんか?
知識で解ける単純な問題ではないと思います。ある程度の知識は
あるので具体的にどうすればよいのか方針をお願いします。
972964:02/05/30 18:44
>>968
すいません、「正」の実解がひとつですよ。問題文の条件を読み違えて
ませんか? それだと全然違う問題になってしまうと思うのですが・・・
>>971
飯食ってた
ちょっと待ってね
974ken:02/05/30 20:13
初項a、公比rの等比数列のはじめのn項までの和をSnとし、
Tn=1/n(S1+S2+…Sn)とおく。0<r<1のとき、
limTnを求めよ。っていう問題です。よろしくお願いします。
n→∞  
>>974
新スレが立ちましたので現在の話題が終了したら
移動をお願いします
◆ わからない問題はここに書いてね 34◆
http://science.2ch.net/test/read.cgi/math/1022747441/
>>971-972
正の実数解を1つだけ持つときは次の2つがあります。
(1) f(x)=0 の実数解が1つで、その実数解が正
(2) f(x)=0 は3つの実数解を持ち、そのうち正のものは1つ。
f(x)の解と係数の関係より正の実数解があることは判断できます。
(説明いりますか?)
なので、(1)は>>967-968と同じです

(2)仮に3つの実数解を持つとしたら、解と係数の関係に反しますよ。
977971−972:02/05/30 21:04
>f(x)の解と係数の関係より正の実数解があることは判断できます。
(説明いりますか?)
いりますいります!

>(2)仮に3つの実数解を持つとしたら、解と係数の関係に反しますよ。
解と係数の関係で実数解の個数がわかるんですか?f(α)・f(β)
を調べるんですか?これも説明いります!

>なので、(1)は>>967-968と同じです
「正」という条件はどこに出てくるのですか?具体的にやってみてくれませんか。
今のままだとよくわかりません。

>>977
ちょっと待ってね。
>>977
まず>>976の(1)から
>f(x)の解と係数の関係より正の実数解があることは判断できます。
>「正」という条件はどこに出てくるのですか?

↓の通りです。

f(x)=x^3 - ax^2 - bx - 1=(x-A)(x−B)(x-C)
とおくと
A・B・C=1>0
A,B,Cが全て実数解の場合、その1つは必ず正(全部負だったらA・B・C<0)
A,B,Cのうち実数解が1つだけの場合、その実数解正(残り2つの虚数解の積は正なので)
>>977
>>976の(2)はこちらの間違いです。

(2)仮に3つの実数解を持つとしたら、解と係数の関係に反しますよ。
  ↓
(2)仮に3つの正数解を持つとしたら、解と係数の関係に反しますよ。
981132人目の素数さん:02/05/30 21:47
f(0)=-1<0, f(1)=-a-b<-1 と変曲点のx座標が 0<a/3
より自明
982901:02/05/30 21:55
>>914
かなり遅れましたが、ありがとうございます。
確かにできました。
もっと勉強しなきゃ!
983977:02/05/30 22:32
>>979>>980
ありがとうございます。
>A・B・C=1>0
A,B,Cが全て実数解の場合、その1つは必ず正(全部負だったらA・B・C<0)
これでは「正の実数解はただ一つであることを示せ」という題意に合致して
ませんよね。やはり、a≧1, b≧1という条件をつかわないと証明できないかと
思うのですが・・・。

>>981
どーもです!かなり高級な解答ですね。まねできそうにないです。
979>>983
>a≧1, b≧1という条件をつかわないと証明できないかと思うのですが・・・。

当たり前。
まずレス読み直せ。
985983:02/05/30 22:40
>>984
「仮に3つの正数解を持つとしたら、解と係数の関係に反する」の理由がどのレス見ても
書かれていないのですが。どのレスを読み直すんですか?ないですよ。
>>985
読み直せ
>>985
それ書いてないよ。
考えてみ。
988985:02/05/30 23:39
>>987
わかりました。AB+BC+CA=-b≦-1<0ですか?確かにそうですね。
そうすると、b≦0という条件だけでも示せるから、a≧1, b≧1という条件を
完全には使ってないですよね?これで良いのですか?
たいがい条件は完全に使い切らないと解けないのが多数だと思うんですが、
これは問題があまり練られていなかったということでしょうか?
989132人目の素数さん:02/05/30 23:39
電卓にa(>0,≠1)の初期値をいれて、
√キーを押し続けていると必ず1になることを示す。
友達も聞いたんですが
もっとわかりやすく教えていただけるとありがたいです。
証明ですから当たり前ですが記述なので。
>>988
>どういう方針で解けばよいのかわかりません。
これにしか答えてないよ。
解答はしてない。
自分でも考えよう。
>>989
a→a^(1/2)→a^{(1/2)^2}→a^{(1/2)^3}
n回すると
a^{(1/2)^n}
になります。
n→∞のとき
{(1/2)^n}→0
電卓の場合表示できる桁数に限りがあるので
ある程度nが大きくなると
1になってしまいます。

OK?
992132人目の素数さん:02/05/31 00:07
n→∞のとき
{(1/2)^n}→0
0になるのがわかりません。
>>992
{(1/2)^n}=1/(2)^n

これでもわからない?
994988:02/05/31 00:18
>>990
すいません、意地悪しないで教えてもらえませんか。
がんばって考えた上で質問してるんですから。998=997ですか?
>>994
意地悪してないよ。
俺ははじめから答えは教えない方針。
996988:02/05/31 00:32
>>995
995=976=979ですか?
>>996
そうですよ。
>>996
もうレス少ないから次スレ行きましょう。
999988:02/05/31 00:40
>>997
答えは976などで書かれていると思うのですが、これは「答え」ではなくて
「方針」なんですか?解答の一部分だけは教えて他のところは教えて
いただけないなんて気になります。合ってるか合ってないかも答えて
くれないんですか?
1000132人目の素数さん:02/05/31 00:40
             ∧         ∧
              / ヽ        ./.∧
           /   `、     /  ∧
          /       ̄ ̄ ̄    ヽ
          l:::::::::              l
         |::::::::::  -=・=-    -=・=- |
         .|:::::::::::::::::   \___/    |
          ヽ:::::::::::::::::::  \/     丿    1000
          ヽ             ./
            ヽ      __ /
             ヽ    /
              >  <
              / \/\
             (  ○―○チリンチリン
             =\\ー\
          / ̄ヽ///ヽ ̄ヽ
          |l(ニ |l .l/=  |ヽ ||
          .ゝ ノ` '     ゝ ノ
10011001
このスレッドは1000を超えました。
もう書けないので、新しいスレッドを立ててくださいです。。。